You are on page 1of 252

Item: lof41 ~. , .

M k <:] t> al ~· ~
QIO: 5071 .l. ar Previous Next lab 'Vfl1ues Notes Calculator

•1 •
A 74-year -old nursing home resident is brought to the emergency department after she develops difficulty speaking . Her past medical history is
•2 notable for atrial fibrillation, long-standing diabet es, hypertension, and smoking. Her vital signs are within normal limits and she is able to follow
commands, but she has gross impairment in her articulation .
•3
•4 Which of the following vessels is most likely to have been occluded in this patient?
•5
•6 A. Anterior cerebral artery
•7
B. Anterior communicating artery
•8
c. Left middle cerebral artery
•9
• 10 D. Posterior communicating artery

· 11 F. Right mirlrliP. C.P.I'P.hr'l l 'l r'tP.ry


• 12
• 13
• 14
• 15
• 16
• 17

• 18
• 19
• 20
• 21

Q
lock
s
Suspend
0
End Block
Item: lof41 ~. , . M k <:] t> al ~· ~
QIO: 5071 .l. ar Previous Next lab 'Vfl1 ues Notes Calculator

1
The co rrect a nswer is c. 73% cho se this.
. 2
This patient is likely suffering from Broca's (expressive) aphasia, a neurologic insult to the motor speech area of the brain. Atrial fibrillation, diabet es, and
•3 smoking are risk factors for strokes, secondary to cardiac emboli. Patients with Broca's aphasia have difficulty enunciating words, although they can
understand both written and verbal language. Unlike patients with Wernicke's (receptive) aphasia, those with Broca's aphasia are usually aware of their
.4 problem, resulting in frustration . In > 9 5 % of the right-handed population (and approximat ely 90% of the left -handed population), the speech center is
•5 locat ed in the left cerebral hemisphere, in a vascular area supplied by the left middle cerebral artery.
Cerebral hemisphere Atrial fibrillation Middle cerebral artery Aphasia Expressive aphasia Diabetes mellitus Embolism Atrium (heart) Neurology Human brain Brain
•6 Fibrillation Cerebral arteries Blood vessel Receptive aphasia
.7
A is no t co rrect. 11% cho se this.
•8 The anterior cerebral artery supplies the medial surface of the cerebral hemispheres. An infarction of this t erritory secondary to an embolus would likely
result in motor and/or sensory loss of the contralat eral leg and foot . Expressive aphasia would not be a likely result .
•9 Anterior cerebral artery Aphasia Expressive aphasia Anatomical terms of location Embolism Contralateral Cerebral hemisphere Embolus Cerebral arteries
• 10 B is no t co rrect. 6 % cho se this.
· 11 The anterior communicating artery connects the left and right anterior cerebral arteries, and forms the most anterior pole of the circle of Willis. This
vessel is the most common site of aneurysms in the circle of Willis. Because of its proximity to the optic chiasm, aneurysms of the anterior communicating
• 12 artery often result in visual field deficits. Rupture of the anterior communicating artery can characteristically result in mutism.
Anterior communicating artery Circle of Willis Optic chiasm Anterior cerebral artery Homonymous hemianopsia Visual field Aneurysm Cerebral arteries Artery
• 13
• 14 D is no t co rrect. 4 % cho se this •
The posterior communicating arteries are also common sites of aneurysms in the circle of Willis. Because of their proximity to cranial nerve (CN) III,
• 15 aneurysms in this location are likely to cause CN III palsies. Pupillary dilation is a common finding, as the aneurysm typically impinges on the superior
aspect of the nerve, compressing the parasympathetic fibers. Aphasia would not be a likely result .
• 16
Circle of Willis Posterior communicating artery Aphasia Cranial nerves Parasympathetic nervous system Aneurysm Pupillary response Mydriasis Artery
• 17 Vasodilation
• 18
E is no t co rrect. 6 % cho se this.
• 19 The right middle cerebral artery is equally likely to be occluded as the left middle cerebral artery in a patient with known cardioembolic risk factors .
However; a stroke in this vessel is unlikely to cause aphasia, as only a small portion of the population has right -hemisphere speech centers. An infarction
• 20 in this t erritory is likely to cause left trunk-arm-face weakness and/or sensory loss with hemispatial neglect.
• 21 Hemispatial neglect Middle cerebral artery Aphasia Stroke Cerebral arteries

6
lock
s
Suspend
0
End Block
Item: lof41 ~. , . M k <:] t> al ~· ~
QIO: 5071 .l. ar Previous Next lab 'Vfl1 ues Notes Calculator

1
. 2 Bottom Line:
In the majority of patients, the speech center is located in the left hemisphere, and is served by the left middle cerebral artery. Patients with Broca's
•3 (expressive) aphasia are aware of their deficits and can comprehend verbal commands, but forming words and sentences requires significant effort .
.4 Aphasia Middle cerebral artery lateralization of brain function Cerebral arteries Expressive aphasia

•5
•6
.7 I iii I;fi 1!1 I•J for year:l 2017 ..
FI RST AID FAC T S

•8
•9 FA17 p486.1
• 10 Aphasia phasia- higher-order language deficit (inabil ity to understand/speak/read/write).
· 11 Dysarth ria-motor inabil ity to spea k (movement defi cit).
• 12 TYPE SPEECH FLUENCY COMPREHENSION COMMENTS

• 13 Repetition impaired
• 14 Broca {expressive) on Auent Intact Broca = Broken Boca (boca = mouth in Span ish).
• 15
Broca area in inferior fronta l gyrus of frontal lobe. Patient
appears frustrated, insight intact.
• 16
Wernicke {receptive) Fluent Impaired Wernicke is Wordy but makes no sense. Patients do not
• 17
have insight.
• 18 Wern icke area in superior temporal gyrus of temporal
• 19 lobe .
• 20 Conduction Fluent Intact Can be caused by damage to arC uate fasciculus.
• 21 Global on fluent Impaired Arcuate fasciculus; Broca and Wernicke areas affected

6
lock
s
Suspend
0
End Block
Item: lof41 ~. , . M k <:] t> al ~· ~
QIO: 5071 .l. ar Previous Next lab 'Vfl1 ues Notes Calculator

1 • •
FA17 p484.1
•2
Effects of strokes
•3
ARTERY AREA OF LESION SYMPTOMS NOTES
•4
Anterior circulation
•5
Middle Motor and sensory cortices fJ - upper Contralateral paralysis and sensory Wernicke aphasia is associated
•6 cerebral limb and face. loss-face and upper limb. with right superior quadrant
•7 artery Temporal lobe (Wernicke area); Aphasia if in dominant (usually visua 1 field defect due to
•8 frontal lobe (Broca area). left) hemisphere. I lemineglect temporal lobe involvement.
if lesion affects nondominant
•9
(usually right) side.
• 10
Anterior Motor and sensory cortices-lower Contralateral paralysis and sensory
· 11 cerebral loss- lower limb.
limb.
• 12 artery
• 13 Lenticulo- Striatum, internal capsule. Contralateral paralysis and/or Common location of lacunar
• 14 striate sensory loss-face and body. infarctsrn, due to hyal ine
• 15 artery Absence of cortical signs arteriosclerosis zoto
(eg, neglect, aphasia, visual field unmanaged hypertension.
• 16
loss).
• 17
Posterior circulation
• 18
Anterior Lateral corticospinal tract. Contralateral paralysis-upper and Medial medullary syndrome-
• 19
spinal lower limbs. caused by infarct of
• 20 artery Medial lemniscus. l contra lateral proprioception. paramedian branches of ASA
• 21 Caudal medulla-hypoglossal nerve. Ipsilateral hypoglossal dysfunction and/or vertebral arteries. •

6
lock
s
Suspend
0
End Block
Item: 1 of 41 ~ 1 • M k -<:J 1>- Jil ~· !:';-~
QIO: 5 0 71 ..L ar Pre v ious Next Lab~lues Notes Calcula t o r

1
&
S) mpathetic fibers lpsilateral llorner S) ndrome &

.2 Iiddle and inferior cerebellar taxia, dysmetria


•3 peduncles
·4 Basilar artery Pons, medulla, lower mjdbrain RAS spared, therefore preserved "Locked-in S)11drome."
C011SCIOIISI1CSS
•5
Corticospinal and corticobulbar Quadriplegia; loss of' oluntary
•6
tracts facial, mouth, and tongue
.7 mo,·ements
·8 Ocular cranial nerYe nuclei, Loss of hori1ontal, but not vertical,
.9 paramedian pontine reticular C\ e mo' emenls
'
formation
• 10
• 11
Posterior Occipital lobe 1!]. Contralateral hemianopia with
cerebral macular sparing.
• 12
artery
• 13
• 14
• 15
• 16
• 17
. 18
• 19
• 20
• 21

a
Lock
s
Suspend
8
End Bl ock
Item: 2 of 41 ~. , . M k <:] t> al ~· ~
QIO: 3772 .l. ar Previous Next Lab 'Vfll ues Notes Calculator

1 •
A 67-year -old man presents to the emergency department complaining of sudden loss of sensation, including t aste, over the left side of his face and
•2 the right side of his body. Furthermore, his voice is hoarse and he is having trouble swallowing. Neurologic examination reveals loss of pinprick and
t emperature sensation over the left face and right body from below the neck all the way down to the right foot . Light touch, vibration, and joint
•3 position sense are all intact. His motor examination is normal, except for an unstea dy gait and consistent falling to the left .
•4
•5 Where might a single lesion account for this clinical picture?

•6
A. Left medulla
•7
•8 B. Left midbrain
•9 c. Left pons
• 10
D. Right medulla
· 11
• 12 E. Right midbrain

• 13 F. Right pons
• 14
• 15
• 16
• 17

• 18
• 19
• 20
• 21

Q
lock
s
Suspend
0
End Block
Item: 2 of 41 ~. , . M k <:] t> al ~· ~
QIO: 3772 .l. ar Previous Next lab 'Vfllues Notes Calculator

1 The co rrect a nswer is A. 51% cho se this.


This patient has lat eral medullary syndrome, also known as Wallenberg's syndrome. This is the most common brain stem stroke. Thrombosis of the
2
vertebral artery supplying the lat eral t egmentum of the medulla is the most common cause. An infarct in the distribution of the posterior inferior
•3 cerebellar artery is another common cause of this syndrome. Resulting anterolat eral system (spinothalamic tract) damage lea ds to decrea sed pain and
t emperature sense of the contralat eral body, because these fibers decussat e near the innervat ed region. Trigeminal nucleus and tract damage lea ds to
.4 ipsilat eral facial decrea sed pain and t emperature senses. Given these two facts, the lesion can be localized to the left medulla, as the patient's left face
and right body are affected. Damage to the inferior cerebellar peduncle lea ds to the ipsilat eral at axia and falling to the left . Damage to the nucleus
•5
ambiguus lea ds to hoarseness and dysphagia. Damage to the nucleus solitarius lea ds to ipsilat eral decrea sed t aste.
•6 Spinothalamic tract Nucleus ambiguus lateral medullary syndrome Solitary nucleus Dysphagia Inferior cerebellar peduncle Posterior inferior cerebellar artery
Vertebral artery Thermoreceptor Brainstem Ataxia Trigeminal nerve nuclei Medulla oblongata Decussation Anatomical terms of location Infarction Tegmentum
.7
Cerebellum Contralateral Ipsilateral lesion Dysphonia Stroke Brain Thrombosis Brainstem stroke syndrome Human brain Trigeminal nerve
•8
Superior cerebellar peduncle
•9
B is no t co rrect. 12% cho se this •
• 10 The lesion is on the left side of the body, but in the medulla, not in the midbrain. The anterolat eral system is very close to the dorsal column, medial
lemniscus fibers at this level, so a lesion here would most likely affect position and vibration sense as well.
· 11
Spinothalamic tract Midbrain Medial lemniscus Posterior column lesion Medulla oblongata Dorsum (biology) lemniscus (anatomy) Vibration
• 12 Anatomical terms of location
• 13
c is no t co rrect. 21% cho se this •
• 14 The biggest clues helping to localize the lesion to the medulla are the presence of hoarseness and loss of t aste, which result from damage to the nucleus
ambiguus and nucleus solitarius, respectively. These two nuclei are in the medulla, not the pons or the midbrain. Pontine lesions often cause horizontal
• 15 gaze paralysis, as they can damage the medial longitudinal fasciculus.
• 16 Nucleus ambiguus Solitary nucleus Medial longitudinal fasciculus Pons Midbrain Medulla oblongata Nucleus (neuroanatomy) lesion Dysphonia Paralysis

• 17 D is no t co rrect. 8 % cho se this •


The medulla is the correct area of the brain stem, but the lesion is on the left side, not the right .
• 18 Brainstem Medulla oblongata lesion Human brain Brain
• 19
E is no t co rrect. 4 % cho se this •
• 20 The anterolat eral system is very close to the dorsal column, medial lemniscus fibers at the level of the midbrain, so a lesion here would most likely affect
position and vibration sense as well. A right-sided lesion would not match the findings in the anterolat eral system based on early decussation of these
• 21 fibers in the spinal cord prior to rea ching the brain stem .

6
lock
s
Suspend
0
End Block
Item: 2 of 41 ~. , . M k <:] t> al ~· ~
QIO: 3772 .l. ar Previous Next lab 'Vfll ues Notes Calculator

1 F is not correct. 4 % chose this .


2 A pons lesion cannot account for the hoarseness and decreased ipsilat eral t aste. A right-sided lesion does not match the findings in the anterolateral
system based on early decussation of these fibers in the spinal cord prior to reaching the brain stem.
•3 Spinothalamic tract Pons Brainstem lesion Spinal cord Anatomical terms of location Ipsilateral Decussation Dysphonia Brain
.4
•5 Bottom Line:
•6 Wallenberg's syndrome is a brain stem stroke that leads to loss of sensation (including t aste) on the ipsilateral side of the face and the contralateral
.7 side of the body. It also causes the patient to have a hoarse voice .
lateral medullary syndrome Brainstem Anatomical terms of location Brainstem stroke syndrome Contralateral Ipsilateral Stroke Brain
•8
•9
• 10 I iii I;fi 1!1 I•J for year:l 2017 ..
FIRST AID FAC T S
· 11
• 12
FA17 p484.1
• 13
Effects of strokes
• 14 ARTERY AREA OF LESION SYMPTOMS NOTES
• 15 Anterior circulation
• 16 Middle Motor and sensory cortices fJ - upper Contralateral paralysis and sensory Wernicke aphasia is associated
• 17 cerebral limb and face. loss-face and upper limb. with right superior quadrant
• 18 artery Temporal lobe (Wernicke area); Aphasia if in dominant (usually visual field defect due to
• 19
frontal lobe (Broca area). left) hemisphere. I lemineglect temporal lobe involvement.
if lesion affects nondominant
• 20 (usually right) side.
• 21
Anterior Motor and scnsorv cortices-lower Contralateral oaraksis and sensorv
6
lock
s
Suspend
0
End Block
Item: 2 of 41 ~ 1 • Ma rk -<:J 1>- Jil ~· !:';-~
QIO: 3772 ..L Pre v ious Next Lab~lues Notes Calcula t o r
& &
1
Posterior circulation
2
Anterior Lateral corticospinal tract. Contralateral paralrsis-upper and Medial medullary syndrome-
•3 spinal lo" er limbs. caused by infarct of
·4 artery ~l edia llemniscus. l contralatera l proprioception. paramedian branches of ASA
•5 Caudal medulla-hypoglossa I nerve. Ipsilateral h) poglossal dysfunction and/or vertebral arteries.
(tongue de,·iates ipsilaterally).
•6
•7
Posterior Lateral medulla: Lateral medullary (Wallenberg)
inferior Nucleus ambiguus (C1 IX, X, XI) O~'iphagia , hoar~cncss, l gag syndrome.
·8 cerebellar 'ucleus ambiguus effects arc
Vestibular nuclei rcflc\
.9 artery Lateral spinothalamic tract, spinal Vomiting, ' ertigo, nystagmus specific to PICA lesions
• 10 trigeminal nucleus l pain and temperature sensation "Don't pick a (PICA) horse
• 11
• 12
• 13
Sympathetic fibers
Inferior cerebellar peduncle
from contralateral body,
ipsilateral face
Ipsilatera l Horner syndrome
taxia, d}smctria
(hoarseness) that can' t cat
(dysphagia)."
Also supplies inferior cerebellar
peduncle (part of cerebellum).
I
• 14
Anterior Lateral pons Lateral pontine syndrome .
• 15
inferior Facia l nucleus Paraly~i~ of face, l h1crimation , Facial nucleus effects arc
• 16 cerebellar l salivation, l taste from anterior specific to AICA lesions.
• 17 artery 7'1 of tongue "Facial droop means AICA's
• 18 Vestibular nuclei Vomiting, vertigo, nystagmus pooped."
Spinothalamic tract, spinal l pain and temperature sensation Also supplies middle and
• 19
trigeminal nucleus from contra latera l body, inferior cerebellar peduncles
• 20
ipsilateral face (part of cerebellum).
• 21 S) mpathetic fibers lpsilateralllorner S) ndrome

a
Lock
s
Suspend
8
End Bl ock
Item: 2 of 41 ~ 1 • M k -<:J 1>- Jil ~· !:';-~
QIO: 3772 ..L ar Pre v ious Next Lab~lues Notes Calcula t o r
& &
1 FA17 p 4 79.1
2
Spinal tract anatomy Ascending tracts synapse and then cross.
•3 and functions
·4 TRACT FUNCTION 1ST-ORDER NEURON SYNAPSE 1 2ND-ORDER NEURON SYNAPSE 2+PROJECTIONS
•5 Ascend ing tracts
•6 Dorsal column Pressure, Sensory ncr"e '\ucleus Decussates
•7 \'ibration, ending - b) pass graci li~. in medu lla
·8 fine touch, pseudounipolar cell nucleus - ascends
proprioception body in dorsal root cuncatus contra laterally
.9
ganglion - enter (ipsilateral in medial
• 10
spinal cord - asce nd medulla) lemniscus
• 11 ipsilaterally in dorsal
VPL (thalam us)
• 12 columns
- sensory cortex
• 13 Spinothalamic tract Lateral: pain, Sensory nerve Ipsilateral gray Decussates at
• 14 temperature ending (AO and C maller (spinal anterior ,,-h
ite
Anterior: fibers) - bypass cord) commtssure
• 15

I
crude touch, pseudounipolar cell - ascends
• 16
pressure body in dorsal root contra laterall y
• 17 ganglion - enter
• 18 spinal cord
• 19 Descending trac t
• 20 Lateral corticospinal oluntary UM t :cell body in Cell body of LM1 : lea,·es 1 \!1 J - muscle
• 21 tract mo\·emcnt of Io motor cortex - an terior horn spinal cord fibers

a
Lock
s
Suspend
8
End Bl ock
Item: 2 of 41 ~ 1 • M k -<:J 1>- Jil ~· !:';-~
QIO: 3772 ..L ar Pre v ious Next Lab~lues Notes Calcula t o r

1
&
. . &

2
•3 FA17 p 474.2

·4 Brain stem- ventral


Optic chiasm ~- Olfactory bulb (CN I)
•5 view
_,----- Olfactory tract
•6 ~- CNII
Infundibulum
•7 ~- OptiC tract

w=
CNIII
·8

r
CN lV(ansesdorsally
.9 and immechately
decussates)
• 10
Pons ....----,- CN V
• 11 ~ ,~, -__:;.---+-CNVI

• 12 M1ddle cerebellar~ ?"'< • CN VII


peduncle \ - - ~ CNVIII
cJ ~~CCN XIX
• 13
• 14 Pyramid \
Pyramidal decussatioo
• 15 CNXI
0 CN~I
• 16
• 17 4 C are above pons (1, II , Il l, I ).
• 18 4 CN arc in pons (V, Vl, VI!, VIIJ).
4 C1 are in medulla (IX, X, XI, XII ).

I
• 19
• 20
4 C1 nuclei are medial (Ill, IV, V I, XI I). ''Factors of 12, except I and 2."
• 21

a
Lock
s
Suspend
8
End Bl ock
Item:3of41 ~. , . M k <:] t> al ~· ~
QIO: 5153 .l. ar Previous Next lab 'Vfl1ues Notes Calculator

1 •
A 49 -year-old man presents to his primary care physician complaining that "I can't move my face." He states the problem began sometime the night
2 before; he went to bed with no deficits and woke up with the problem. He has a history of coronary artery disease and type 2 diabetes mellitus
treated with atorvastatin and metformin, respectively. He reports no weakness or numbness. He is asked to smile during the physical exam, the
•3 results of which are shown in the video clip .
•4
OPEN MEDIA
•5
•6
Which of the following correctly describes the type and location of the lesion resulting in this patient's symptoms?
•7
•8
A. Lower motor neuron, cranial nerve XII
•9
B. Upper motor neuron, cranial nerve XII
• 10

· 11 c. Lower motor neuron, cranial nerve V

• 12 D. Upper motor neuron, cranial nerve V


• 13
E. Lower motor neuron, cranial nerve VII
• 14
F. Upper motor neuron, cranial nerve VII
• 15
• 16
• 17

• 18
• 19
• 20
• 21

Q
lock
s
Suspend
0
End Block
Item:3of41 ~. , . M k <:] t> al ~· ~
QIO: 5153 .l. ar Previous Next lab 'Vfl1ues Notes Calculator

1 The co rrect a nswer is E. 65% cho se this.


This patient is presenting with classic Bell palsy, a lower motor neuron lesion affecting the peripheral branches of the facial nerve. This can be
2 differentiat ed from an upper motor neuron lesion by the presence or absence of forehea d movement. In an upper motor lesion, the patient will be able to
3 raise his eyebrows or close his eyelids tightly, due to dual innervation from the contralat eral facial nerve nucleus.
Upper motor neuron lesion lower motor neuron Bell's palsy Neuron Upper motor neuron Facial nerve lower motor neuron lesion Facial motor nucleus
.4 Motor neuron lesion Anatomical terms of location Contralateral Nerve
•5
A is no t co rrect. 3 % cho se this •
•6 A lower motor neuron lesion of the hypoglossal nerve can cause ipsilat eral weakness of the tongue.
Hypoglossal nerve lower motor neuron Neuron lower motor neuron lesion Motor neuron lesion Anatomical terms of location Ipsilateral
.7
B is no t co rrect. 3 % cho se this •
•8
An upper motor neuron lesion of the hypoglossal nerve can cause contralat eral weakness of the tongue .
•9 Hypoglossal nerve Upper motor neuron lesion Neuron Upper motor neuron Motor neuron lesion Anatomical terms of location Contralateral
• 10 c is no t co rrect. 5 % cho se this.
· 11 A lower motor neuron lesion of the trigeminal nerve can cause ipsilat eral j aw weakness.
lower motor neuron Trigeminal nerve Neuron lower motor neuron lesion Motor neuron lesion Anatomical terms of location Ipsilateral Jaw
• 12
D is no t co rrect. 4 % cho se this •
• 13
An upper motor neuron lesion of the mandibular nerve rarely affects function as the trigeminal motor nucleus receives bilat eral innervation . A bilat eral
• 14 upper motor neuron lesion above the trigeminal motor nucleus may result in a hypera ctive j aw reflex .
Upper motor neuron lesion Mandibular nerve Neuron Upper motor neuron Motor neuron Trigeminal nerve lesion Reflex Mandible
• 15
F is no t co rrect. 20 % cho se this •
• 16
In an upper motor neuron facial nerve lesion, one would expect to see movement in the ipsilat eral forehea d.
• 17 Neuron Upper motor neuron Facial nerve lesion Motor neuron Anatomical terms of location Ipsilateral

• 18
• 19 Botto m Li ne:
• 20 A peripheral facial nerve (CN VII) palsy, otherwise known as Bell palsy, results from a lower motor neuron lesion affecting the peripheral branches of the
facial nerve. It is often idiopathic and of sudden onset, and usually resolves on its own. It can be differentiat ed from va scular causes (eg, stroke) by the
• 21 presence of ipsilat eral forehea d paralysis; if the ipsilat eral forehea d is moving normally, it should suggest a cerebrova scular event.

6
lock
s
Suspend
0
End Block
Item: 3 of 41 ~ 1 • M k -<:J 1>- Jil ~· !:';-~
QIO: 5153 ..L ar Pre v ious Next Lab~lues Notes Calcula t o r
& &
1

3
141;fi1Ji•J f o r yea r : 2017 ...
FIRST AID FACTS

·4
•5 FA17 p 502.2

•6
Facial nerve lesions

.7 Upper motor neuron Destruction of motor corte;~. or connect ion

-
lesion between motor cortex and fa cial nucleus in focum
·8
.9
pons - contralateral paralysis of lo\\cr muscles "'""*"
of facial expression. Forehead is spared due to
• 10 its bilateral U~ I N innervation .
• 11 Lower motor neuron Destruction offacia I nucleus or C II
• 12 lesion anywhere along its course - ipsilateral
• 13 paralysis of upper and lower muscles of
• 14
facial expression a, hypcracusis, loss of taste
sensation to anterior tongue.
• 15
When idiopathic (most common), facial nerve
• 16 palsy is called Bell palsy. May also be caused CHVII

• 17 by Lyme disease, herpes simplex, herpes r_._


(l.MN lesion •

pobyl
. 18 zoster (Ramsay Hunt syndrome), sarcoidosis,
• 19
tumors (eg, parotid gland), diabetes mellitus.
Treatment is corticosteroids, acyclo' ir. 'vlost
• 20
patients graduallr recover function .
• 21

a
Lock
s
Suspend
8
End Bl ock
Item:3of41 ~. , . M k <:] t> al ~· ~
QIO: 5153 .l. ar Previous Next lab 'Vfl1ues Notes Calculator

1 • •
FA17 p 4 76.2
2 Cranial nerves
3 NERVE CN FUNCTION TYPE MNEMONIC

•4 Olfactory 1 Smell (only CN without thalamic relay to cortex) Sensory Some


•5 Optic TJ Sight Sensory Say
•6 Oculomotor Ill Eye movement (SR, IR, MR, 10), pupi llary constriction Motor l\ larry
•7 (sphincter pupillae: Edinger-Westphal nucleus, musca rinic
receptors), accommodation, eyelid opening (levator palpebrae)
•8
Trochlear IV Eye mo,·ement (SO) Motor l\Ioney
•9
• 10
Trigeminal v Mastication, facial sensation (ophthalmic, maxil lary, mandibular Both But
divisions), somatosensation from anterior 213 of tongue
· 11
Abducens VI Eye movement (LR) l\totor l\Iy
• 12
Facial VII Facial movement, taste from anterior 1/ ; of tongue, lacrimation, Both Brother
• 13
sal ivation (submandibula r and sublingual glands), eyelid closing
• 14 (orbicularis oculi), auditory volume modulation (stapedius)
• 15
Vestibulocochlear VIII Hearing, balance Sensory Says
• 16
Glossopharyngeal IX Taste and sensation from posterior 113 of tongue, swallowing, Both Big
• 17 sal ivation (parotid gland), monitoring carotid body and sinus
• 18 chcmo- and baroreceptors, and elevation of pharynx/larynx
• 19 (stylopharyngeus)
• 20 Vagus X Taste from supraglottic region, swallowing, soft palate elevation, Both Brains
midline uvula, talking, cough reflex, parasympathetics to
• 21 •
• thnr!lf"'l'\o:a l,rlnmin !ll "i~f"f'l rn mnn itnrino •..vu-1-il" -:arf"h ,..hPn'1A.. ~nrl

6
lock
s
Suspend
0
End Block
Item:4of41 ~. , . M k <:] t> al ~· ~
QIO: 3236 .l. ar Previous Next Lab 'Vfll ues Notes Calculator

1 •
A 45 -year-old woman who recently underwent a left mastectomy and axillary dissection for breast cancer now presents to her physician with a chief
2 complaint that she feels like her shoulder blade sticks out sometimes. She denies any pain.

3
This patient's injured nerve originates at which spinal levels?
•4
•5
A. C3, C4, and CS
•6
•7 B. cs and C6
•8 c. cs, C6, and C7
•9
D. C7 and C8
• 10
E. C7, C8, and T1
· 11
• 12
• 13
• 14
• 15
• 16
• 17
• 18
• 19
• 20
• 21

Q
lock
s
Suspend
0
End Block
Item: 4 of 41 ~ 1 • M k -<:J 1>- Jil ~· !:';-~
QIO: 3236 ..L ar Prev ious Next Lab~lues Notes Calculat o r

& &
1
2 Th e correct an sw er i s c . 560/o chose this.
3 As a complication of her surgery, this woman has sustained an injury to her long thoracic
nerve, which innervates the serratus anterior muscle. The function of the serratus anterior
4 muscle Is to anchor the scapula against the thoracic cage. When the long thoracic nerve is
•5 damaged, the scapula moves away from the thoracic cage, resulting in what Is referred to
as " winging" of the scapula. Long thoradc nerve injury is an occasional complication of
•6 mastectomy. The long thoradc nerve originates from the brachial plexus, specifically from
CS, C6, and C7. A helpful mnemonic for this is "C5- C7 keep the wing s fl yi ng to heaven. •
•7 This Image demonstrates the cl inica l presentation of a w inged scapula.
Long tho• 1cic nerve Winged scapula Serratus anterior muscle Brachial pie us Mastectomy
·8
Scapula Rib cage Spinal nerve Thorax Cervical spinal nerve 6 Muscle
.9
Anatomical terms of location
• 10
Image copyright ©2012 Adriaenssens eta/.; licensee
• 11 BioMed Central Ltd.
• 12
A is not co rrect. 90/o chose this •
• 13
C3, C4, and CS are the origins of the phrenic nerve, which innervates the diaphragm. Unilateral injury to the phrenic nerve as a postoperative
• 14 complication Is more likely with cardiac cases, but there are many other possible causes. Patients may have exertional dyspnea and/or orthopnea but are
often asymptomatic. If an underlying lung disea se is present, dyspnea at rest may occur.
• 15 Orthopnea Phrenic nerve Dyspnea Thoracic diaphragm Asymptomatic Respiratory disease Lung
• 16 B is not co rrect . 170/o ch ose this.
• 17 CS and C6 join together to form the upper trunk and do pa1ticipate In the formation of the long thoracic nerve, but C7 is also Involved . upward traction
during difficult childbirth (dystocia, for example) can lead to Erb palsy (CS-C6 nerve roots), which manifests with adduction and internal rotation of the
• 18 arm, forearm extension, but with preservation of hand and wrist movement.
Long thoracic nerve Spinal nerve Cervical spinal nerve 6 Anatomical terms of motion Adduction Forearm Nerve root
• 19
0 i s n ot correct. 8 0/o ch ose this•
• 20
The long thoracic nerve does not originate from C7 and CB, but instead from CS-C7 .
• 21 Long thoracic nerve Spinal nerve Thorax Cervical spinal nerve 8

a
Lock
s
Suspend
8
End Block
Item:4of41 ~. , . M k <:] t> al ~· ~
QIO: 3236 .l. ar Previous Next lab 'Vfllues Notes Calculator

1 E is not correct. 10% chose this.


2 C8 and T1 join together to form the lower trunk of the brachial plexus, wheras C7 is a part of the middle trunk. Neither C8 nor T1 contributes to the long
thoracic nerve.
3 long thoracic nerve Brachial plexus Spinal nerve Thoracic spinal nerve 1 Cervical spinal nerve 7 Cervical spinal nerve 8 Thorax
4

•5
Bottom Line:
•6
Long thoracic nerve (CS, C6, C7) injury can occur in mastectomies, impairing anchoring of the scapula against the thoracic cage .
.7 long thoracic nerve Scapula Rib cage Spinal nerve Thorax Thoracic vertebrae Mastectomy Cervical spinal nerve 6

•8
•9
• 10 I ill ;fi 1!1 I•J for year:[ 2017 ..
FIRST AID FAC T S

· 11
• 12 FA17 p428.1
• 13 Brachial plexus lesions
• 14 0 Erb palsy ("waiter's tip') cs Lateral Randy
• 15
E) Klumpke palsy {claw hand) Musculocutaneous T rm·is
E) Wrist drop C6 D rinks
• 16 Axillary
0 Winged scapula Middle Posterio' 0 Cold
• 17 0 Deltoid paralysis C7 E) (;)(Extensors) 0 Median {flexors)
(;) 'Saturday night palsy' {wrist drop)
Beer
• 18
0 Difficulty flexing elbow. variable CB Radial
• 19 sensory loss Lower Medial
• 20 0 Decreased thumb function• a I I
a Ulnar
'Pope's blessing' T1 I I I
• 21 Trunks Divisions Cords Branches
• e Intrinsic muscles of hand. A •

6
lock
s
Suspend
0
End Block
Item: 4 of 41 ~ 1 • M k -<:J 1>- Jil ~· !:';-~
QIO: 3236 ..L ar Pre v ious Next Lab~lues Notes Calcula t o r
& &
1 CONDITION INJURY CAUSES MUSCLE DEFICIT FUNCTIONAL DEFICIT PRESENTATION
2 Erb palsy ("waiter's Traction or Infants-lateral Deltoid, Abduction (arm
3 tip" ) tear of upper traction on neck supraspinatus hangs by side)
4
("Erb-er") trunk: during delhcry Infraspinatus Lateral rotation (arm
C5-C6 roots Adults-trauma medially rotated)
•5
•6 Biceps brachii Flexion, supination
.7 (ann extended and
pronated)
·8
.9
Klumpke palsy Traction or tear Infants-upwarcl Intrinsic hand Total claw hand:
• 10
of lower trunk: force on arm muscles: lumbricals normally
• 11 C8-TI root during deli, cry lumbricals, Aex NfCP joints and
• 12 Aclults-trauma interossei, extend Dl P and PIP
• 13 (eg, grabbing a thenar, joints
• 14
tree branch lo hypothenar
break a fall)
• 15
Thoracic outlet Compression CcrviC<ll rib Same as Klumpkc Atrophy of intrinsic
• 16
syndrome of lower trunk (arrows in r.J), palsy hand muscles;
• 17 Pancoast tumor
and subclavian ischemia, pain,
. 18 ,·essels and edema
• 19 clue to vascular
• 20 compressiOn
• 21

Winged scapula Lesion of long
.. Axillary node
'. 1".
Serratus anterior
.
lnabilitr to anchor
.' .tM . .
a
Lock
s
Suspend
8
End Bl ock
Item: 4 of 41 ~ 1 • M k -<:J 1>- Jil ~· !:';-~
QIO: 3236 ..L ar Pre v ious Next Lab~lues Notes Calcula t o r
& &
1
FA17 p427.1
2 Upper extremity nerves
3 NERVE CAUSES Of INJURY PRESENTATION
4 Axillary (CS-C6) Fractured surgical neck of humerus; anterior F'lallened deltoid
•5 dislocation of humerus l.oss of arm abduction at shoulder (> 15°)
•6
Loss of sensation over deltoid muscle and lateral
arm
.7
Musculocutaneous Upper trunk compression Loss of forearm Aexion and supination
·8 (CS-C7) Loss of sensation o\·er lateral foream1
.9
Radial (CS-Tl ) ~l idshaft fracture of humerus; compression of Wrist drop: loss of elbow, wrist, and finger
• 10 axilla, eg, due to crutches or sleeping " ith arm extension
• 11 O\·er chair ("Saturday night palsy") l grip strength (wrist extension n ecess;~ ry for
• 12 maximal action of Aexors)
Loss of sensation over posterior arm/forearm and
• 13
dorsal hand
• 14
M edian (CS-Tl) Supracondylar fracture of humerus { prox i me~ l "Ape hand" and "Pope's blessing"
• 15
lesion); carpal tunnel syndrome and wrist Loss of wrist Aexion, Aexion of lateral fingers,
• 16 laceration (distal lesion) thumb opposition , lumbrieals of 2nd and 3rcl
• 17 digits
. 18
Loss of sensation over thenar eminence and
dorsal and palmar aspects of lateral 3\1:! fingers
• 19
with proximal lesion
• 20
Ulnar (C8-T1 ) Fracture of medial epicondyle of humerus "Ulnar claw" on digit extension
• 21 "funny bone" (proximal lesion); fractmed Radial deviation of wrist upon Aexion {proximal

a
Lock
s
Suspend
8
End Bl ock
Item: 4 of 41 ~ 1 • M k -<:J 1>- Jil ~· !:';-~
QIO: 3236 ..L ar Pre v ious Next Lab~lues Notes Calcula t o r

1
&
actions of medial 2 lumbrical muscles &

2 Loss of sensation over medial 11h fingers


including hypothenar eminence
3
Recurrent branch of Superficial laceration of palm "Ape hand"
4
median nerve (C5-T1) Loss of thenar muscle group: opposition,
•5
abduction, and flexion of thumb
•6 'o loss of sensation
•7 llumcrus fmcturcs, proximall) to distally, folio\\ the AR\1 (.\\illar) - Radial - \lcdian)
·8
.9 Axlilaryne.ve~ ~
• 10 "" Tl ~ Med.an nerve
Axillary nerve
• 11
Musculocutaneous ne.ve-------.:. 'I? Ulnar nerve --._
_r Intercostobrachial
• 12 " ' - Radial ne.ve .-" nerve
Radial nerve ~ ......_Radial nerve
/ Medial brachial
• 13 cutaneous nerve Palm of hand

..,"""~ '--um"'""
• 14
• 15 • Musculocutaneoos nerve~ Medial antebrachial
cutaneousnerve Median nerve-....._
• 16
Radial nerve_/,
_.....-Ulnar nerve
• 17 Radial nerve----.........
Recurrent branch"""'
of median nerve Radial nerve -....
• 18
• 19 Dorsum of hand

• 20
• 21

a
Lock Suspend
s 8
End Bl ock
Item: 5 of 41 ~ 1 • M k -<:J 1>- Jil ~· !:';-~
QIO: 3377 ..L ar Pre v ious Next Lab~lues Not es Calcula t o r
& &
1
A 47-year-old woman presents to the clinic because of difficulty walking . A galt Imbalance is evident on physical examination, and the patient
2 describes an occasional feeling as though a shock has run down her spine when moving her neck. The patient's fluid-attenuated inversion-recovery
MRI Is shown in the image.
3
4
•5
•6
•7

·8
.9
• 10
• 11
• 12
• 13
• 14
• 15
• 16
• 17
• 18
• 19
Image courtesy of Dooley eta/. J List J Ophthalmic Vis Resv.5(3); 2010 Ju/0
• 20
• 21
• Which of the follow1nq additional symptoms would be most expected in this patient?

a
Lock
s
Suspend
8
End Bl ock
Image courtesy of Dooley et at. J List J Ophthalmic Vis Resv.S( J); 2010 Ju/0
· 11
• 12
Which of the following additional symptoms would be most expected in this patient?
• 13
• 14 A . Aphasia
• 15
B. Bladder incontinence
• 16
• 17 c. Complete paralysis

• 18 D. Seizures
• 19
E. Tremor at rest
• 20
• 21

Q
lock
s
Suspend
0
End Block
Item: 5 of 41 ~ 1 • M k -<:J 1>- Jil ~· !:';-~
QIO: 3377 ..L ar Prev ious Next Lab~lues Not es Calculat o r

& &
1
2
Th e correct an sw er i s B. 5 4 0/o chose this.
3 This patient has a gait imbalance likely caused by impairment of the cerebellar pathways, as well as Lhermitte phenomenon, a transient shock-like
4 sensation most often triggered by flexion of the neck. Her MRI shows multiple hyperintense signal abnormalities in the periventricular and subcortical
white matter. These fmdings suggest a diagnosis of multiple sclerosis (MS). The simplest defmition of MS is an autoimmune disease of the central nervous
5 system causing multiple white matter lesions separated in both time and space. The class1c patient is a young woman. Problems can be arranged Into
broad categories such as cranial nerves (optic neuritis, relative afferent pupillary defect, internuclear ophthalmoplegia, and nystagmus), motor system
•6 (wea kness, spastiCity, and abnormal reflexes), sensory system (paresthesias), cerebellar system (ataxia, dysarthria, and intention tremor), and autonomic
•7 system (bladder dysfunction). Scanning the answer choices, bladder dysfunction is an additional symptom encountered in patients with MS.
Optic neo i •s Intenbon tremor Multiple sclerosis Dysarthria Nystagmus Autoimm 1ne disease Central nervous system Spasticity Cranial nerves Ophthal'lloparesos
·8 Cerebellum Paresthesia White matter Sensory system Tremor Autonomic nervous system Nervous system Symptom Gait Magnetic resonance imaging
.9 Afferent nerve fiber Autoimmunity Flexion Gait (human) Neuritis Urinary bladder

• 10 A i s n ot co rrect. 9 0/o chose this.


• 11 Although patients with multiple sclerosis may have irregularities of speech (scanning speech and dysarthria) resulting from cerebellar involvement,
aphasia is usually not present. Aphasia generally represents a cortical lesion and manifests with impairment of some aspect of language processing or
• 12 communication .
Multiple sclerosis Dysarthria Aphasia Lesion Cerebellum Cortex (anatomy) Cerebral cortex
• 13
C is not co rrect. 70/o chose this •
• 14
Patients with multiple sclerosis often have pronounced weakness but Incomplete paralysis; complete paralysis is rare. Complet e paralysis may be seen In
• 15 locked-in syndrome, with lesion to the basilar a1tery.
Locked-in syndrome Multiple sclerosis Basilar artery Lesion Paralysis
• 16
D is not co rrect. 150/o ch ose this .
• 17
Seizures are not a common symptom of multiple sclerosis. Seizures can manifest In a variety of ways, including tonic and clonic contractions .
• 18 Multiple sclerosis Clonus Symptom Epileptic sei zure

• 19 E i s n ot correct. 150/o ch ose this.


• 20 Patients with multiple sclerosis (MS) can suffer from intention tremor, but tremor at rest is a symptom of Parkinson disease (PO), not MS. Classically,
tremor at rest in PO is "pill-rolling" and occurs with no engagement of purposeful activities.
• 21 Intention tremor Multiple sclerosis Parkinson's disease Symptom Tremor

a
Lock
s
Suspend
8
End Block
Item:5of41 ~. , . M k <:] t> al ~· ~
QIO: 3377 .l. ar Previous Next Lab 'Vfll ues Notes Calculator

1
Bottom Line:
2
Gait abnormalities and the Lhermitte sign suggest MS, which can interfere with the autonomic system (including bladder function) .
3 lhermitte's sign Gait (human) Urinary bladder Autonomic nervous system Gait Gait abnormality

4
5
•6 lijj ;fi IJ l•l for year:l 2017 ..
FI RST AID FAC T S
.7
•8 FA17 p 513.1
•9
Internuclear Medial longitudinal fasciculus (MLF): pair of l\ILF in MS.
• 10 ophthalmoplegia tracts that allows for crosstalk between C VI When looking left, the left nucleus of CN VI
· 11 and CN Ill nuclei. Coordinates both eyes to fires, which contracts the left lateral rectus and
• 12 move in same horizontal direction. Highly stimulates the contralateral (right) nucleus of
• 13
myelinated (must commun icate quickly so eyes C II I via the right M LF to contract the right
move at same time). Lesions may be unilateral mediaI rectus.
• 14
or bilateral (latter classically seen in multiple Directional term (eg, right I 0, left It 0 ) refers
• 15 sclerosis). to which eye is paralyzed.
• 16 Lesion in MLF = internuclear ophthalmoplegia
• 17 (l 0), a conjugate horizontal gaze palsy. Lack
• 18
of communication such that when CN VI
nucleus activates ipsilateral lateral rectus,
• 19
contralateral C III nucleus does not stimulate
• 20 medial rectus to contract. Abducting eye
• 21 gets nystagmus (CN I overfires to stimulate

6
lock
s
Suspend
0
End Block
Item: 5 of 41 ~ 1 • M k -<:J 1>- Jil ~· !:';-~
QIO: 3377 ..L ar Pre v ious Next Lab~lues Not es Calcula t o r

fires, which contracts the left lateral rectus and


& &
1 and C I Ill nuclei. Coordinates both C\'CS•
to
2 move in same horizontal direction. l lighly stimulates the contralateral (right) nucleus of
3 myelinated (must communicate quickly so eyes CN Il l via the rightl\ ILF' to contract the right
mo,·e at same time). Lesions may be unilateral media l rectus.
4
or bilateral (latter classically seen in mult iple Di rectional term (eg, right 1'\0, left I 0 ) refers
5
sclerosis). to ''hich eye is paralyzed.
•6 Lesion in ~ lLF = internuclear ophthalmoplegia
•7 (It 0 }, a conjugate hori7ontal ga7e palsy. Lack
·8 of communication such that when C ' VI
.9
nucleus activates ipsilateral lateral rectus,
contralateral C • III nucleus docs not stimulate
• 10
medial rectus to contract. bducting eye
• 11 gets nystagmus (C ' VI ovcrfires to stimulate
• 12 CN Ill ). Convergence normal.
• 13 lateral Medial re<:ti lateral Right INO (right MLF lesion}
• 14 ( Rv ectus

• 15
Left gaze
• 16
y
• 17
• 18
rLefi~LF_. _.. Impaired adduction
(convergence normal}
Nystagmus

• 19 "T
Medial rectus Nuclei
• 20 subnucleus ofCN VI
of CN Ill
• 21

a
Lock
s
Suspend
8
End Bl ock
Item: 5 of 41 ~ 1 • M k -<:J 1>- Jil ~· !:';-~
QIO: 3377 ..L ar Pre v ious Next Lab~lues Not es Calcula t o r
& &
1 FA17 p 4 93.1
2
Multiple sclerosis Autoimmune inAammation and dem)clination of CNS (brain and spinal cord). Patients can
3 present with optic neuritis (sudden loss of vision resulting in Ylareus Gunn pupils), !NO,
4 hemiparesis, hemisensory symptoms, bladder/bowel dysfunction. Srmptoms may exacerbate with
5 increased body temperature (eg, hot bath, exercise). Relapsing and remitting is most common
•6 clinical course. lost often affects women in their 20s and 30s; more common in Caucasians
Ji,·ing farther from equator. 'eck Aexion may precipitate sensation of electric shock nmning
.7
down spine (Lhermitte phenomenon).
·8 Charcot triad of I\ IS is a SI:\:
.9 Scanning speech
• 10 Intention tremor (also Incont inence and Internuclear ophthalmoplegia)
• 11
• "\lystagmus
• 12 FINDINGS f IgG level and myelin basic protein in CSJi'. Oligoclonal bands are diagnostic. ~1rRI is gold
standard. Periventricular plaques fJ (areas of oligodendrocyte loss and reactive gliosis) with
• 13
preservation of axons. Multiple white matter lesions disseminated in space and time.
• 14
• 15
• 16
• 17
. 18
• 19 TREATMENT Slow progression with disease-modifying therapies (eg, ~-interferon , glatiramer, nataliw mab). Treat
• 20 acute flares with IV steroids. S) mptomatic treatment for neurogenic bladder (catheterization,
• 21 muscarinic antagonists), spasticit) (baclofen, CASAR receptor agonists), pain (TCAs,
• • • 1 • \

a
Lock
s
Suspend
8
End Bl ock
Item: 6 of 41 ~. , . M k <:] t> al ~· ~
QIO: 3778 .l. ar Previous Next Lab 'Vfll ues Notes Calculator

1 •
A 77 -year -old man with a history of hypertension and prior stroke presents to the emergency department with a hea dache of 4 hours' duration . His
2 vital signs are currently stable. His neurologic examination is nearly normal, except upon finger -to-nose t esting, the patient overshoots and
undershoots with his right arm . CT of the hea d demonstrat es no acute hemorrhages. He is referred to the neurology service, which decides to do an
3 MRI to further evaluat e for brain lesions.
4
5 Which of the following is most likely to be observed on brain MRI?

•6
A. Cerebellar vermis infarct
•7
•8 B. Left cerebellar hemisphere infarct

•9 c. Left cerebral hemisphere infarct


• 10
D. Right cerebellar hemisphere infarct
· 11
E. Right cerebral hemisphere infarct
• 12
• 13
• 14
• 15
• 16
• 17

• 18
• 19
• 20
• 21

Q
lock
s
Suspend
0
End Block
Item: 6 of 41 ~ 1 • M k -<:J 1>- Jil ~· !:';-~
QIO: 3778 ..L ar Prev ious Next Lab~lues Notes Calculat o r

& &
1
2
Th e correct an sw er i s D. 660/o chose this.
3 This patient likely infarcted his right cerebellar hemisphere. The cerebellar hemispheres are involved in t he
control of distal appendicular muscles. When damaged, appendicular ataxia results, with poor performa nce
4
on finger-to-nose and heel-to-shin tests (dysmetria) and impairment of rapid alternating movement s
5 (dysdiadochokinesia). The pa t ient will also have ocular ata xia wit h overshoot on saccades to the right.
Cerebellar coordination crosses t wice as it communicates down the spine: (1) the first midline crossing
6 being before reaching the red nucl eus and (2) t he second crossing after emerging from the red nucl eus t o
project to sp1nal neurons. Therefore, cerebellar lesions are ipsilateral to the observed deflc1t. As this
0 7 patient's right side is affected, th e lesion is likely on th e right, similar to that shown in the image. Notably,
o8 cerebellar communication with cortical ce nte rs is contralateral.
Dysdiadocho inesia Dysmetria Ataxia Red nucleus Cerebellum Saccade Lesion Anatomical terms of location
.9 Neuron Ipsilateral Cerebral hemisphere Infarction Cortex (anatomy) Contralateral Cerebral cortex
• 10
• 11
• 12
• 13
0 14
• 15
0
16 I mage courtesy of Dr. Laughlin Dawes
Osborn A. Diagnostic Imaging: Brain Amirsys
0
17 2004
• 18
A i s n ot co rrect. s oto ch ose this .
• 19
Damage to the vermis of t he cerebellum results in t runcal ata xia with a broad-based gait that is unsteady and " drunk-like." Because facial and vocal
• 20 control are also localized in t he vermis, lesions in t his area may result In a slurring and slowing of speech . The pat ient described above has appendicular;
not trunca l, ataxia .
• 21 Cerebe lum Ata 1a Gait (human) Cerebellar vermis Gait Horse gait

a
Lock
s
Suspend
8
End Block
Item: 6 of 41 ~. , . M k <:] t> al ~· ~
QIO: 3778 .l. ar Previous Next lab 'Vfllues Notes Calculator

1 B is not correct. 19% chose this.


2 Lesions of the left cerebellar hemisphere would lead to the symptoms described in the stem, only on the left side of the body. Dysmetria is ipsilatera l to
the side of the cerebellar lesion.
3 Dysmetria Cerebellum lesion Anatomical terms of location
4 c is not correct. 6 % chose this.
5 Lesions of the left cerebral hemisphere, if they involve the motor cortex, would result in motor deficits on the right side of the body. This patient's
symptoms do involve his right arm; however; his problem is one of coordination, not paresis.
6 Cerebral hemisphere Motor cortex Paresis Cortex (anatomy)
.7 E is not correct. 1 Ofo chose this.
•8 Lesions of the right cerebral hemisphere, if they involve the right motor cortex, would result in motor deficits on the left side of the body; however; these
deficits are usually paretic (lack of movem ent or hyperreflexia ). What is described in this patient is clearly a lack of coordination, not the absence of
•9 movem ent, which is under the control of the cerebellar hemispheres.
Cerebral hemisphere Motor cortex Cerebellum Hyperreflexia Cerebral cortex Cortex (anatomy) Paresis
• 10

· 11
• 12 Bottom Line:

• 13 Dysmetria, or lack of coordination, localizes to a deficit in the ipsilateral cerebellar hemisphere .


Dysmetria Cerebellum Anatomical terms of location
• 14
• 15
• 16
I ill ;fi 1!1 I•J for year:[ 2017 ..
FI RST AID FA CTS
• 17

• 18
FA17 p481 .1
• 19
Common brain lesions
• 20
AREA Of LESION CONSEQUENCE EXAMPLES
• 21 Frontal lobe Disinhibition and deficits in concentration,

6
lock
s
Suspend
0
End Block
Item: 6 of 41 ~ 1 • M k -<:J 1>- Jil ~· !:';-~
QIO: 3778 ..L ar Pre v ious Next Lab~lues Notes Calcula t o r
& &
1 FA17 p481 .1
2 Common brain lesions
3 AREAOFLESION CONSEQUENCE EXAMPLES

4 Frontal lobe Disinhibition and deficits in concentration,


orientation, judgment; may have reemergence
5
of primiti,·e reflexes.
6
Frontal eye fields Eves look toward lesion.
0 7 '
Paramedian pontine Eyes look away from side of lesion.
o8
reticular formation
.9
Medial longitudinal Internuclear ophthalmoplegia (impaired l\ lultiple sclerosis.
• 10
fa sciculus adduction of ipsilateral eye; nystagmus of
• 11 contralateral eye with abduction).
• 12 Dominant parietal Agraph ia, acalculia, finger agnosia, left-right Cerstmann srndrome.
• 13 cortex disorientation .
0 14 Nondominant parietal Agnosia of the contra latera I side of the world. 1-lemispatial neglect syndrome.
• 15 cortex
0
16 Hippocampus Anterograde amnesia-inability to make new
0
17 (bilateral) memoncs.
. 18 Basal ganglia l\ lay result in tremor at rest, chorea, athetosis. Park inson disease, Huntington disease.
• 19 Subthalamic nucleus Contralateral hemiballismus.
• 20 Mammillary bodies Wernicke-Korsakoff syndrome- C onfmion, Wernicke problems come in a C.\ N 0 ' beer.
• 21 (b ilateral) .\ taxia, ~ystagmus, O phthalmoplegia,

a
Lock
s
Suspend
8
End Bl ock
Item: 6 of 41 ~ 1 • M k -<:J 1>- Jil ~· !:';-~
QIO: 3778 ..L ar Pre v ious Next Lab~lues Notes Calcula t o r
& &
1
Hippocampus Anterograde amnesia-inability to make new
2 (bilateral) memones.
3 Basal ganglia Parkinson disease, Huntington disease.
l\1ay result in tremor at rest, chorea, athetosis.
4
Subthalamic nucleus Contralateral hemiballismus.
5
Mammillary bodies Wernicke-Korsakoff syndrome- Confusion, Wernicke problems come in a C.\ N 0 ' beer.
6 (bilateral) Ataxia, :--.ystagmus, Ophthalmoplegia,
0 7 memory loss (anterograde and retrograde
o8 amnesia), confabulation, personalit) changes.
.9 Amygdala (bilateral) KIOver-Bucy syndrome-disinhibited behavior II V-1 encephalitis.
• 10 (eg, hyperphagia, hypersexuality, h) perorality).
• 11 Superior colliculus Parinaud syndrome-paralysis of conjugate Stroke, hydrocephalus, pinealoma.
• 12
vertical gaze (rostral interstitial nucleus also
involved).
• 13
Reticular activating Reduced levels of arousal and wakefulness
0 14
system (midbrain) (eg, coma).
• 15
Cerebellar hemisphere Intention tremor, limb at·axia, loss of balance; Cerebellar hemispheres are laterally loc;~ted ­
0
16
damage to cerebellum - ipsilatera l deficits; <lffect lateral limbs.
0
17 fall toward side of lesion.
. 18 Cerebellar vermis Truncal ataxia, dysarthria. Vermis is centrally located-affects central body.
• 19 Degeneration associated with ch ronic alcohol
• 20 USC .

• 21

a
Lock
s
Suspend
8
End Bl ock
Item: 7 of 41 ~. , . M k <:] t> al ~· ~
QIO: 5014 .l. ar Previous Next lab 'Vfl1ues Notes Calculator

2

A 59 -year -old man presents to the emergency department after waking up in the middle of the night with a very severe hea dache. When asked about
the intensity of pain, the patient exclaims, " I feel like my hea d is going to explode." Emergent CT of the hea d demonstrat es blood tracking down the
lA
"' AI
sulci and following the contours of the pia.
3
4 What is the most likely underlying pathophysiologic mechanism for this patient's condition?
5
6 A. Arteriovenous malformation
•7
B. Atherosclerosis
•8
c. Cerebral contusion
•9
• 10 D. Hypertension

· 11 F. RupturP. of 'l rtP.ri'l l 'l n P.urysm


• 12
• 13
• 14
• 15
• 16
• 17

• 18
• 19
• 20
• 21

Q
lock
s
Suspend
0
End Block
Item: 7 of 41 ~ 1 • M k -<:J 1>- Jil ~· !:';-~
QIO: 5 0 14 ..L ar Previous Next Lab~lues Not es Calculat or
& &
1 Th e co rrect a nsw er i s E. 6 9 0/o chose this.
2 The patient is likely suffering from a subarachnoid hemorrhage (SAH). Subarachnoid hemorrhages begin abruptly, occurring at night in 30% of cases, and
are classically described as the "worst headache of my life." The headache Is unilateral in approximately one-third of patients. The onset of the headache
3 may or may not be associated with a brief loss of consciousness, seizure, nausea, vomiting, focal neurologic deficit, or stiff neck. There are usually no
Im portant foca l neurologic signs at presentation unless bleeding occurs Into the bra in and cerebrospinal fluid (CSF) at the same time (menlngocerebral
4 hemorrhage). Rupture of arterial aneurysms represents the most common cause of non-traumatic SAH. Upon rupture, aneurysms release blood directly
5 Into the CSF under arterial pressure. The blood spreads quickly within the CSF, rapidly increasing intracranial pressure. Death or deep coma ensues If the
bleeding continues. The bleeding usually lasts only a few seconds, but rebleedlng Is common. Risk factors for intracranial aneurysm include atherosclerotic
6 disease, arteri ovenous malformations (AVMs), adult polycystic kidney d isease, and connective tissue disease. When present, risk factors for aneurysmal
rupture Include hypertension, smoking, a lcohol use, and situations causing sudden elevations in blood pressure. Other less common causes of SAH,
7 incl uding vascular malformations, bleed ing d iatheses, amylo id ang iopathy, and Illicit drug use, feature bleed ing that is less abrupt and may continue ove r
a longe r pe riod.
·8
Bleeding diathesis Subarachnoid hemorrhage Cerebrospinal fluid Intracranial ane~... ysm Intracranial pressure Polycystic kidney disease Connective tissue disease
.9 Hypertension Blood pressure Headache Aneurysm Arteriovenous malformation Vomiting Atherosclerosis Nausea Syncope (medicine) Connective tissue
• 10 Focal neurologic signs Unconsciousness Coma Kidney Bleeding Epileptic seizure Kidney disease Amyloid Neurology Human brain Tobacco smoking Brain
• 11 Cerebral arteriovenous malformation Blood vessel

• 12 A is not co rrect . 1 0 0/o chose this •


Arteriovenous malformation (AVM) is the cause of subarachnoid hemmorrhage In approximately 4% -5% of cases. AVMs are congenital anomalies In which
• 13 there are abnormal direct connections between arteries and veins. Hemorrhage secondary to an AVM is usually intraparenchymal, but it can extend to the
• 14 Intraventricular or subarachnoid space at times.
Arteriovenous malformation Subarachnoid space Congenital disorder Bleeding Artery
• 15
B is no t co rrect . 30fo chose this .
• 16
Atherosclerosis is a risk factor for the development of intracranial aneurysms. However, atherosclerosis is associat ed more with the cause of Ischemic
• 17 stroke, rather than subarachnoid hemmorrhage (SAH). The "most likely" nontraumatic cause of SAH is aneurysm rupture .
Atherosclerosis Ischemic stroke Stroke Aneurysm Risk factor Ischemia
. 18
C i s not co rrect. 6 % ch ose this .
• 19
Traumatic subarachnoid hemmorrhage (SAH) is most often caused by bleeding Into the CSF from damaged blood vessels associated with cerebral
• 20 contusion. Although traumatic SAH is more common than nontraumatlc (spontaneous) SAH, there is no evidence of trauma in this case. Indeed, the
patient's pa in began during sleep, a relatively common finding in cases of nontraumatic aneurysma l SAH .
• 21 Cereb a contus1on Bruise Blood vessel

a
Lock
s
Suspend
8
End Block
Item: 7 of 41 ~. , . M k <:] t> al ~· ~
QIO: 5014 .l. ar Previous Next lab 'Vfl1ues Notes Calculator

1 D is not correct. 12% chose this.


2 Although hypertension is a risk factor for aneurysm rupture, hypertensive hemorrhage is the most common cause of nontraumatic intra parenchymal
hemorrhage, not subarachnoid hemmorrhage. The pathogenesis of hypertensive hemorrhage is unknown, but is believed to be related to chronic
3 pathologic effects of hypertension on the small penetrating blood vessels.
4 Hypertension Intraparenchymal hemorrhage Aneurysm Risk factor Bleeding Pathogenesis Blood vessel

5
6 Bottom Line:

7 Rupture of arterial aneurysms represents the most common cause of nontraumatic subarachnoid hemorrhage.
Subarachnoid hemorrhage Bleeding Aneurysm
•8
•9
• 10
l i j l ; f i i J I • l toryear:[ 2017 • ]
· 11 FI RST AID FAC T S

• 12
FA17 p483.1
• 13
Intracranial hemorrhage
• 14
Epidural hematoma Rupture of middle meningeal artery (branch
• 15
of maxillary artery), often zoto skull
• 16 fracture fJ involving the pterion (thinnest
• 17 area of the lateral skull). Lucid interva l. Rapid
• 18 expansion under systemic arterial pressure
-+ transtentorial herniation, CN Ill palsy.
• 19
CT shows biconvex (lentiform), hyperdense
• 20
blood collection : not crossing suture lines .
• 21

6
lock
s
Suspend
0
End Block
Item: 7 of 41 ~ 1 • M k -<:J 1>- Jil ~· !:';-~
QIO: 5 0 14 ..L ar Pre v ious Next Lab~lues Not es Calcula t o r
& &
1
Subdural hematoma Rupture of bridging ,•cins. Can be acute
2
(traumatic, high-energy impact - h) pcrdcnse
3 on CT) or chronic (associated with mild
4 trauma, cerebral atrophy, elderly, alcoholism
5 - hrpodense on CT). Also seen in shal..en
babies. Predisposing factors: brain atrophy,
6
trauma.
7 Crescent-shaped hemorrhage (red arro\\S in
·8 and • ) that crosses suture lines. Can cause
.9 midline shift (yellow arro" in ), findin gs of
• 10 "acute on chronic" hemorrhage (blue arrows
in [!]).
• 11

• 12
Subarachnoid Bleeding D I] due to trauma, or rupture of
hemorrhage an aneurrsm (such as a saccular aneurysm
• 13
) or arteriovenous malformation. Rapid
• 14
time course. Patients complain of""orst
• 15 headache of my Iife." Bloody or yellow
• 16 (xanthochromic) spinal tap. 4- 10 days aft er
• 17 hemorrhage, vasospasm (narrowing of blood
vessels) can occur due to blood breakdown
. 18
or rebleed - ischemic infarct; nimodipine
• 19
used to prc,·cnt/rcducc "asospasm. t risk of
• 20 de, -eloping communicating and/or obstructive
• 21 hydrocephalus.

a
Lock
s
Suspend
8
End Bl ock
Item: 7 of 41 ~ 1 • M k -<:J 1>- Jil ~· !:';-~
QIO: 5 0 14 ..L ar Pre v ious Next Lab~lues Not es Calcula t o r
& &
1
lntraparenchymal lost commonly caused by systemic
2
hemorrhage hypertension. Also seen wilh amyloid
3 angiopathy (recurrent lobar hemorrhagic
4 stroke in elderly), vasculitis, neoplasm. \!lay
5 zo
be to reperfusion injury in ischemic
6
stroke. Typically occurs in basal ganglia
and internal capsule (Charcot-Bouchard
7
microaneurysm of lenticulostriate vessels),
·8 but can also occur in cerebral hemispheres,
.9 brainstem, and cerebellum
• 10
• 11 FA17 p 482.1
• 12 Ischemic brain Irreversible damage begins after 5 minutes of hypoxia. \t1ost vulnerable: hippocampus, neocortex,
• 13 disease/stroke cerebellum, watershed areas. lrrc, ersible neuronal injury. Hippocampus is most vulnerable to
• 14 ischemic hypoxia ("vulnerable hippos") .
• 15
Stroke imaging: noncontrast CT to exclude hemorrhage (before tPA can be given). CT det·ects
ischemic changes in 6-24 hr. Diffusion-weighted lRl can detect ischemia within 3- 30 min.
• 16
• 17 TIME SINCE ISCHEMIC 12-24 HOURS 24-72 HOURS 3-S DAYS 1-2WEEKS > 2WEEKS
EVENT
. 18
Histologic Red neurons 1ccrosis
+ ~ l ac ropha ges Reactive gliosis Glial scar
• 19
features (eosinophilic neutroph ils (microglia) +vascular
• 20 cytoplasm proliferation
• 21 with

a
Lock
s
Suspend
8
End Bl ock
Item: 7 of 41 ~ 1 • M k -<:J 1>- Jil ~· !:';-~
QIO: 5 0 14 ..L ar Pre v ious Next Lab~lues Not es Calcula t o r

1 g g g l• •g )
ischemic changes in 6-24 hr. Oiffusion-\\eighted ~lRl can detect ischemia within 3-30 min.
2

3 TIME SINCE ISCHEMIC 12-24 HOURS 24-72 HOURS 3-5 DAYS HWEEKS > 2WEEKS
4 EVENT
Histologic Red neurons Necrosis+ ~ lacrophages Reactive gliosis Glial scar
5
features (eosinophilic neutrophils (microglia) + \'ascular
6
C)1oplasm pro! iferation
7 with
·8 pyknotic
.9 nuclei)
• 10 Ische mic stroke Acute blockage of vessels .... disruption of blood no\\ and subsequent ischemia .... liquefacti\e
necros1s.
• 11
3 types:
• 12
• Thrombotic-due to a clot forming directly at site of infarction (commonly the MCA rJ),
• 13 usually 0\·er an atherosclerotic plaque.
• 14 • Embolic- embolus from another part of the body obstructs ,·essel. Can affect multiple vascular
• 15 territories. Examples: atri<d Fibril lation; DVT with patent foramen ovale.
• 16
• Hypoxic- due to hypoperfusion or hypoxemia. Common during cardiovascular surgeries, lends
to affect watershed areas.
• 17
Treatment: tPA (if within 3-4.5 hr of onset and no hemorrhage/risk of hemorrhage). Reduce risk
. 18 with medical therapy (eg, aspirin, clopidogrel); optimum control of blood pressure, blood sugars,
• 19 lipids; and treat conditions that t ris~ (eg, atrial fibri llation).
• 20 Transient ische mic Brief, re,·ersible episode of focal neurologic dysfunction" itJ10ut acute infarction (8 ,\IRI), with the
• 21 attack majority resolving in < 15 minutes; deficits due to focal ischemia .

a
Lock Suspend
s 8
End Bl ock
Item:8of41 ~. , . M k <:] t> al ~· ~
QIO: 3542 .l. ar Previous Next lab 'Vfl1ues Notes Calculator

1
2

A 54 -year -old white man presents for a routine preoperative assessment. Physical examination reveals that the man is unable to close his left eye
complet ely or to seal the corner of his mouth on the left side. ECG reveals new first -degree atrioventricular heart block. The patient denies any recent
lA
"' AI
changes in his health, but notes that he recently traveled to New England for a camping trip . The patient is given a common medication to trea t the
3 underlying cause of his symptoms.
4
5 What is a common adverse effect of this trea tment in adults?

6
A. Acute cholestatic hepatitis
7
•8 B. Discoloration of t eeth

•9 c. Nephrotoxicity
• 10
D. Ototoxicity
· 11
E. Photosensitivity
• 12
• 13
• 14
• 15
• 16
• 17

• 18
• 19
• 20
• 21

Q
lock
s
Suspend
0
End Block
Item:8of41 ~. , . M k <:] t> al ~· ~
QIO: 3542 .l. ar Previous Next lab 'Vfl1 ues Notes Calculator

2 The co rrect a nswer is E. 44% cho se this.


This patient presents with stage 2 Lyme disea se, which is marked by facial nerve palsy and evidence of a new heart block. Fa cial nerve palsy results from
3 a lower motor neuron lesion of the facial nerve and manifests as an ipsilat eral facial paralysis with the inability to close the eye or seal the corner of the
mouth on the affected side. The patient most likely acquired Lyme disea se from an Ixodes tick bite during his recent camping trip to New England. The
4
most common antibiotic used to trea t Lyme disea se as an outpatient is doxycycline. Photosensitivity, gastrointestinal distress, and the discoloration of
5 t eeth in children are common adverse effects of doxycycline.

6 If this patient had severe neurologic manifestations (eg, meningitis or encephalitis) and/or severe carditis (eg, 2nd- or 3rd-degree AV block or a PR
interval > 300 msec), then he would require hospitalization and trea tment with ceftriaxone.
7
A is no t co rrect. 6 % cho se this.
8
Acute cholestatic hepatitis is an adverse effect of macrolides.
•9
B is no t co rrect. 24% cho se this.
• 10 Discoloration of the t eeth from doxycycline occurs among children who are < 8 years old rather than in adults.

· 11 c is no t co rrect. 1 3 % cho se this.


Nephrotoxicity is an adverse effect associat ed with aminoglycosides and vancomycin .
• 12
D is no t co rrect. 1 3 % cho se this •
• 13 Ototoxicity is an adverse effect of aminoglycosides, vancomycin, and high doses of aspirin .
• 14
• 15 Botto m Li ne:
• 16 Lyme disea se can result in facial nerve palsy. Lyme disea se is most commonly trea t ed with doxycycline, which is associat ed with photosensitivity,
gastrointestinal issues, and discoloration in children's t eeth . Ceftriaxone is warranted in cases of severe neurologic or cardiac pathology.
• 17

• 18
• 19
lijj ;fi IJ l•l f o r yea r:l 2017 ..
• 20 FIRST AID FACTS

• 21

6
lock
s
Suspend
0
End Block
Item:8of41 ~. , . M k <:] t> al ~· ~
QIO: 3542 .l. ar Previous Next Lab 'Vfl1ues Notes Calculator

2 Bottom Line:

3 Lyme disease can result in facial nerve palsy. Lyme disease is most commonly treated with doxycycline, which is associated with photosensitivity,
gastrointestinal issues, and discoloration in children's teeth. Ceftriaxone is warranted in cases of severe neurologic or cardiac pathology.
4 Lyme disease Doxycycline Ceftriaxone Facial nerve Facial nerve paralysis Photosensitivity Pathology Gastrointestinal tract

5
6
7 I ill ;fi 1!1 I•J for year:[ 2017 ..
FIRST AID FAC T S
8
•9 FA17 p 188.1
• 10 Tetracyclines Tetracycline, doxycycl ine, minocycline.
· 11 MECHANISM Bacteriostatic; bind to 30S and prevent attachment of aminoacyl-tRl A; limited CNS penetration.
• 12 Doxycycline is fecally eliminated and can be used in patients with renal failure. Do not take
• 13 tetracycl ines with milk (Ca 2+), antacids (Ca 2+ or Mg 2+), or iron-containing prepa rations because
divalent cations inhibit drugs' absorption in the gut.
• 14
• 15
CLINICALUSE Borrelia burgdor{eri, M p11eumoniae. Drugs' ability to accumulc1te intracellularly makes them very
effective against Rickettsia and Chlamydia. Also used to treat acne. Doxycycline effective against
• 16
MRSA .
• 17
ADVERSE EFFECTS Gl distress, discoloration of teeth and inhibition of bone growth in children, photosensitivity.
• 18 Contraindicated in pregnancy.
• 19
MECHANISM OF RESISTANCE ! uptake or t efAux out of bacterial cel ls by plasmid-encoded transport pumps.
• 20
• 21
-·..... - ....,.. ..
6
lock Suspend
s 0
End Block
Item: 8 of 41 ~ 1 • M k -<:J 1>- Jil ~· !:';-~
QIO: 3542 ..L ar Pre v ious Next Lab~lues Notes Calcula t o r
& &
1
FA17 p 142.4
2
Lyme disease Caused by Borrelia burgdor(eri, "h ich is A Key L) me pie to the FACE:
3
transmitted by the Ixodes deer tick rJ {also Facial nen·e palsy (typically bilateral)
4
vector for Anaplasma spp. and protozoa Arthritis
5 Babesia). Natural rescn·oir is the mouse {and Cardiac block
6 important to tick life cycle). Er) thema migrans
7 Common in northeastern United States. Treatment: doxycycline (lst line); amoxicillin
Stage 1-early localized: err thema migrans and cefuroxime in pregnant women and
8
(typical "bulls-eye" configuration 1s children.
.9
pathognomonic but not always present),
• 10 Au-like sp11ptoms.
• 11 Stage 2- early disseminated: secondary lesions,
• 12 carditis, AV block, fa cial ncn c {13cll) palsy,
• 13
migratory myalgias/transient arthrit is .
Stage 3- late disseminated: encephalopathies,
• 14
chronic arthritis.
• 15
• 16
• 17
FA17 p 502.2
. 18
Facial nerve lesions
• 19
Upper motor neuron Destruction of motor cortex or connection
• 20
lesion between motor cortex and facial nucleus in
• 21 pons ..... contralateral paralrsis of lower muscles

a
Lock
s
Suspend
8
End Bl ock
2
3
4 FA17 p 502.2
5 Facial nerve lesions
6 Upper motor neuron Destruction of motor corte:. or connection
7 lesion between motor cortex and facial nucleus in
8
pons ..... contralateral paralysis of lower muscle
of facial expression. Forehead is spared due to
.9
its bilateral U\ II innenation .
• 10
l ower motor neuron Destruction offacia I nucleus or C II
• 11
lesion any" here along its course ..... ipsilateral
• 12 paralysis of upper and lower 111Uscles of
• 13 facial expression · , hyperacusis, loss of taste
• 14 sensation to anterior tongue.
• 15
When idiopathic (most common), fe~cialnerl'e
palsy is called Bell palsy. May also be caused
• 16
by Lyme disease, herpes simplex, herpes
• 17 zoster (Ramsay Hunt syndrome), sa rcoidosis,
• 18 tumors (eg, parotid gland), diabetes mel l itu~.
• 19 Treatment is corticosteroids, acyclo' ir. Most
• 20 patients gradually recover function .
• 21

a
Lock
s
Suspend
8
End Block
Item:9of41 ~. , . M k <:] t> al ~· ~
QIO: 4592 .l. ar Previous Next lab 'Vfl1ues Notes Calculator

1 •
A 59 -year -old, morbidly obese man with a history of uncontrolled hypertension and smoking suffers a stroke that damages his left subthalamic
2 nucleus.

3
Which of the following is the most likely result of the stroke?
4
5
A. Broca's aphasia
6
7 B. Hemiballismus

8 c. Huntington's chorea
•9
D. Internuclear ophthalmoplegia
• 10
E. Parkinson's disea se
· 11
• 12 F. Werdnig- Hoffmann disea se

• 13
• 14
• 15
• 16
• 17

• 18
• 19
• 20
• 21

Q
lock
s
Suspend
0
End Block
Item: 9 of 41 ~ 1 • M k -<:J 1>- Jil ~· !:';-~
QIO: 4592 ..L ar Prev ious Next Lab~lues Notes Calculat o r

& &
1
Th e correct an sw er i s B. 710/o ch ose this.
2
Hemiballismus is a condition that results from injury to the subthalamic nucleus, which is a basal ganglia nucleus. The injury leads to sudden
3 uncontrollable flailing movements of the contralateral arm due to loss of Inhibit ion of the thalamus via the globus pallidus.
Hem1ballbmus Basal ganglia Globus pallidus Subthalamic nucleus Thalamus Contralateral Ganglion Anatomical terms of location
4
A i s not correct. 4% chose t his.
5
Broca's aphas1a is a language disorder caused by damage to Broca's area of the brain, which is located in the frontal lobe. The injury leads to a problem
6 with language fluency, while sparing language comprehension.
Broca's a ea F o otal lobe Aphasia Expressive aphasia Sentence processing Language o.sorder Human bram Brain
7
C i s not correct. 9% chose this.
8
Huntington's chorea is an autosomal dominant genetic disease that leads to chorea and dementia due to atrophy of the caudate nucleus.
9 Caudate nudeus Dominance genetics) Huntington's disease Chorea Dementia At·ophy Autosome Genetic disorder Cell nucleus

• 10 D is n ot co rrect. 5% chose this •

• 11 Internuclear ophthalmoplegia is caused by a lesion in the medial longitudinal fasciculus, which is responsible for coordinating lateral eye gaze. The Injury
leads to nystagmus in the abducting eye. The lesion is often seen in multiple sclerosis.
• 12 Multiple sclerosis Medial longitudinal fasciculus Nystagmus Internuclear ophthalmoplegia Ophthalmoparesis Lesion

• 13 E is not co rrect. 70/o chose this •

• 14 Parkinson's disease is a result of damage to the substantia nigra, not the subthalamic nucleus. The injury leads to a hypokinetic disorder due to lack of
Inhibition of the indirect pathway of the basal ganglia, which normally Inhibits movement.
• 15 Parkinson's disease Basal ganglia Substantia nigra Subthalamic nucleus Ganglion Hypokinesia Indirect pathway of movement Cell nucleus

• 16 F is not co rrect . 4 0/o ch ose this .

• 17 Werdnlg-Hoffman disease is an autosomal recessive genetic disease that leads to degeneration of the anterior horns of the spinal cord . It presents as
"floppy baby" syndrome. It is not associat ed with a subthalamic nucleus lesion.
. 18 Subthalamic nucleus Autosomal recessive Spinal cord Dominance (genetics) Lesion Autosome Lateral ventricles Cell nucleus Genetic disorder
Anatomical terms of location
• 19
• 20
• 21 Bottom Line:

a
Lock
s
Suspend
8
End Block
Item:9of41 ~. , . M k <:] t> al ~· ~
QIO: 4592 .l. ar Previous Next lab 'Vfl1 ues Notes Calculator

1
Bottom Line:
2 Hemiballismus is characterized by sudden uncontrollable flailing movem ents of the contralateral arm due to loss of inhibition of the thalamus via the
3 globus pallidus.
Hemiballismus Globus pallidus Thalamus Contralateral Anatomical terms of location
4
5
6
I iii I;fi 1!1 I•J for year:l 2017 ..
FI RST AI D FA CT S
7

8
FA17 p489.1
9
Movement disorders
0 10
DISORDER PRESENTATION CHARACTERISTIC LESION NOTES
o ll
Akathisia Restlessness and intense urge Can be seen with neuroleptic
0
12 to move use or in Parkinson disease.
0
13 Asterixis Extension of wrists causes Associated with hepatic
0 14 "Rapping" motion encephalopathy, Wilson
0 15 disease, and other metabolic
derangements.
0
16
0 17
Athetosis Slow, snake-!ike, writhing Basal ganglia
movements; especially seen in
0
18
the fin gers
0 19
Chorea Sudden, jerky, purposeless Basal ganglia Chorea = dancing.
20
0
movements Sydenham chorea seen in acute
0
21 rheumatic fever. •

6
lock
s
Suspend
0
End Block
Item: 9 of 41 ~ 1 • M k -<:J 1>- Jil ~· !:';-~
QIO: 4592 ..L ar Pre v ious Next Lab~lues Notes Calcula t o r

2
Dystonia Sustained, involuntary muscle Writer's cramp, blepharospasm,
contractions torticollis.
3
Essential tremor High-frequency tremor Often familial. Patients often
4
with sustained posture self-medicate with alcohol,
5
(eg, outstretched arms}, which l tremor amplitude.
6 worsened with mo,·ement or Treatment: nonselective
7 when anxious P-blockers (eg, propranolol),
8 primidonc.
9 Hemiballismus Sudden, wild Aailing of I arm Coni ra latera I subtha Iamie Pronounce "Half-of-body
• 10
+1- ipsilateral leg nucleus (eg, lacunar stroke) ball istic."
Contralateral lesion .
• 11
Intention tremor Slow, zigzag motion when Cerebellar d)sfunction
• 12
pointing/extending toward a
• 13
target
• 14
Myoclonus Sudden, brief, uncontrol led Jerks; hiccups; common in
• 15 muscle contraction metabolic abnormalities such
• 16 as renal and liver fai lure.
• 17 Resting tremor Uncontrolled movement of distal Subst·anlia nigra (Parkinson Occurs at rest; "pill-rolling
• 18 appendages (most noticeable disease) tremor" of Parkinson disease.
• 19 in hands}; tremor alleviated b)' When you park )'Our car, it is
intentionaJ movement at rest.
• 20
• 21

a
Lock
s
Suspend
8
End Bl ock
Item: 10 of 41 ~ 1 • M k -<:J 1>- Jil ~· !:';-~
QIO: 5230 ..L ar Pre v ious Next Lab~lues Notes Calcula t o r
& &
1
A 56-year-old man presents to his primary care physician because of 3 months of "clumsiness." When he walks, he says he feels unsteady, as If he Is
2 going to fall on his right side. He also has started writing with his left hand; whenever he tries to use his right hand, he has a tremor and difficulty
with coordination. The video clip shows a portion of the patient's neurologic examination.
3
OPEN MEDIA
4
5
6 Which of the following locat ions in the man's brain is most likely affected?

7 :
8 A. Left basal ganglia

9 B. Left cerebellar hemisphere


• 10
c. Left motor cortex
• 11
o. Right basal ganglia
• 12
• 13 E. Right cerebellar hemisphere
• 14 F• Right motor cortex
• 15
• 16
• 17
• 18
• 19
• 20
• 21

a
Lock
s
Suspend
8
End Bl ock
going to fall on his right side. He also has started writing with his left hand; whenever he tries to use his right hand, he has a tremor and difficulty
with coordination . The video clip shows a portion of the patient's neurologic examination.
2
3 OPEN MEDIA

4
5
6
7
8
9
• 10
• 11
• 12
• 13
• 14
• 15
• 16
• 17
• 18
• 19
• 20
• 21

a
Lock
s
Suspend
8
End Block
going to fall on his right side. He also has started writing with his left hand; whenever he tries to use his right hand, he has a tremor and difficulty
with coordination . The video clip shows a portion of the patient's neurologic examination.
2
3 OPEN MEDIA

4
5
6
7
8
9
• 10
• 11
• 12
• 13
• 14
• 15
• 16
• 17
• 18
• 19
• 20
• 21

a
Lock
s
Suspend
8
End Block
• 12
Which of the following locations in the m an's brain is most likely affected?
• 13
• 14 :
A . Left basal ganglia
• 15
• 16 8 . Left cerebellar hemisphere

• 17 c. Left motor cortex

. 18
o. Right basal ganglia
• 19
E. Right cerebellar hemisphere
• 20
• 21 F. Right motor cortex

a
Lock
s
Suspend
8
End Block
Item: 10 of 41 ~ 1 • M k -<:J 1>- Jil ~· !:';-~
QIO: 5230 ..L ar Prev ious Next Lab~lues Notes Calculat o r

& &
1

2 Th e correct an sw er i s E. 550/o chose this.


3 Whereas the cerebral hemispheres sense and move the contralateral side of a person's body, the cerebellum receives sensory and motor input from the
Ipsilateral side of the body. Cerebellar lesions, therefore, manifest as Ipsilateral deficits, including loss of coordination (ataxia), hypotonia, Intention
4 tremor, and Instability (patients feel they are going to fa ll toward the side of their lesion). In the video the patient demonstrates dysmetria during the
finger-to-nose test, primarily when using t he right hand. Thus the lesion Is most likely involving the patient's right cerebellar hemisphere.
5 Cerebel um Dysmetr;a Intention tremor Hypotonia Anatomical terms of location Ce1 ebra hemisphere Tremor Lesion Contralateral Ipsilateral Ata: ,a
6
7
8
A i s not correct. 10% chose t his.
The basal ganglia have been assodated with the fine-tuning of motor activities, such as force control and timing. Effects of unilateral lesions are typically
minor, with the contralateral structure providing suffident processing. A unilateral lesion of the left basal ganglia might produce minor motor problems In
the contralateral limbs, but not the ataxia, tremor, and instability seen In this patient.
Basa ganglia Ata ia Ganglion Tremor Anatomical terms of location Lesion Contra late• a•
I
9
10 B is n ot co rrect . 1 g % chose this.
A left cerebellar hemisphere would manifest with left-sided ataxia, hypotonia, Intention tremor, and instability.
• 11
Intention tremor Hypotonia Ataxia Tremor Cerebellum
• 12
C is not co rrect. 70fo chose t his.
• 13 The left motor cortex is responsible for planning and executing motor activity for the right side of the patient's body. A lesion of the left motor strip would
result In right-sided paralysis, not at axia, tremor, and instability.
• 14
Ataxia Motor cortex Lesion Tremor Paralysis Primary motor cortex Cortex (anatomy) Cerebral cortex
• 15
D is no t co rrect . 60/o ch ose this .
• 16 The basal ganglia have been associated with the fine-tuning of motor activities, such as force control and timing . The effects of unilateral lesions are
typically minor, with the contralateral structure providing sufficient processing. A unilateral lesion of the right basal ganglia might produce minor motor
• 17 problems In the contralateral limbs, but not the at axia, tremor, and Instability seen in this patient.
. 18 Basal ganglia Ataxia Ganglion Tremor Anatomical terms of location Lesion Contralateral

• 19 F i s n ot correct. 3 0/o ch ose this .


Although the right motor cortex is responsible for planning and executing motor activity for the left side of the patient's body, a lesion of this processing
• 20 area would result in left-sided para lysis, not at axia, tremor, and instability.
• 21 Ata: a Motor corte· Tremor Lesion Paralysis Cortex (anatomy) Cerebra cortex

a
Lock
s
Suspend
8
End Bl ock
Item: 10 of 41 ~. I • M k <:] t> al ~· ~
QIO: 5230 .l. ar Previous Next lab 'Vfl1 ues Notes Calculator

1
Bottom Line:
2
The cerebellum is responsible for the coordination of movem ent on the ipsilateral side of the body. A lesion of a cerebellar hemisphere would result in
3 ataxia, instability, and dysmetria on the same side of the body as the lesion.
Dysmetria Cerebellum Ataxia Anatomical terms of location lesion Cerebellar hemisphere
4
5
6
7
lijj ;fi IJ l•l for year:l 2017 ..
FIRST AID FACTS

8
9
10
· 11
Common brain lesions
AREA Of LESION CONSEQUEN CE EXAMPLES
FA17 p481 .1

I
Frontal lobe Disinhibition and deficits in concentration,
• 12
orientation, judgment; may have reemergence
• 13
of primitive reflexes.
• 14
Frontal eye fields Eyes look toward lesion.
• 15
Paramedian pontine Eyes look away from side of lesion.
• 16
reticular formation
• 17
Medial longitudinal Internuclear ophthalmoplegia (impaired Iultiple sclerosis.
• 18 fasciculus adduction of ipsilateral eye; nyst<1gmus of
• 19 contra lateral eye with abduction) .
• 20 Dominant parietal Agraphia, acalculia, finger agnosia, left-right Ccrstmann srndrome.
• 21 cortex disorientation.

6
lock
s
Suspend
0
End Block
Item: 10 of 41 ~ 1 • M k -<:J 1>- Jil ~· !:';-~
QIO: 5230 ..L ar Pre v ious Next Lab~lues Notes Calcula t o r
&
. - - ·· -- &
1
Nondominant parietal Agnosia of the contra latera I side of the world. Hemispatial neglect syndrome.
2
cortex
3
Hippocampus Anterograde amnesia-inability to make new
4 (bilateral) memones.
5
Basal ganglia lay result in tremor at rest, chorea, athetosis. Parkinson disease, Huntington disease.
6
Subthalamic nucleus Contralateral hemiba II ismus.
7
Mammillary bodies Wernicke-Korsakoff syndrome- Confusion, Wernicke problems come in a C \~ 0 ' beer.
8 (bilateral) Ataxia, ~ystagmus, Ophthalmoplegia,
9 memory loss (anterograde and retrograde
10 amnesia), confabulation, personalit} ch;mges.
• 11

• 12
• 13
Amygdala (bilateral)

Superior colliculus
KIOver-Bucy syndrome-disinhibited behavior
(eg, hyperphagia, hypcrscxua Iity, hypcrora Iit y).
Parinaud syndrome-paralrsis of conjugate
vertical gaze (rostral interstitial nucleus also
IISY-1 encephalitis.

Stroke, hydrocephalus, pinea loma.


I
• 14
involved).
• 15
Reticular activating Reduced levels of arousa l <111d wakeful ness
• 16
system (midbrain) (eg, coma).
• 17
Cerebellar hemisphere Intention tremor, limb ataxia, loss of balance; Cerebellar hemispheres are laterally located-
. 18
damage to cerebellum - ipsilatera l deficits; affect lateral limbs.
• 19
fall toward side of lesion .
• 20
Cerebellar vermis Truncal ataxia, drsarthria. crmis is centrally located-affects central body.
• 21 Degeneration associated with chronic alcohol

a
Lock
s
Suspend
8
End Bl ock
Item: 10 of 41 ~ 1 • M k -<:J 1>- Jil ~· !:';-~
QIO: 5230 ..L ar Pre v ious Next Lab~lues Notes Calcula t o r

1
&
FA17 p 484.1 &

2 Effects of strokes
ARTERY AREA OF LESION SYMPTOMS NOTES
3
Anterior circulation
4
5
Middle ~lotor and sensory cortices rJ-uppcr Contralateral para I) sis and sensor~ Wernicke aphasia is associated
cerebral limb and face. loss-face and upper limb. with right superior quadrant
6
artery Temporal lobe (Wernicke area); ph asia if in dominant (usually visual field defect due to
7 frontal lobe (Broca area). left) hemisphere. llemineglecl temporal lobe involvement.
8 if bion affects nondominanl
9 (usually right) side.
10 Anterior Ialor and sensory cortices-lower Contralateral paralysi~ and sensory
cerebral limb. loss-lower limb.
• 11
artery
• 12
Lenticulo- Striatum, internal capsule. Contralateral para I) sis and/or Common location of lacunar
• 13
striate sensory loss-face and body. infarcts rn,due to hyaline
• 14
• 15
• 16
artery Absence of cortical signs
(eg, neglect, aphasia, visual field
loss).
arteriosclerosis zoto
unmanaged hypertension. I
• 17 Posterior circulation
• 18 Anterior Lateral corticospinal tract. Contralateral paralrsis-upper and Medial medullary syndrome-
spinal lo11er limbs. caused by infarct of
• 19
artery ~lediallemniscus. l contralateral proprioception. paramedian branches of ASA
• 20
Caudal medulla-hypoglossal nerve. Ipsilateral h) poglossal dysfunction and/or vertebral arteries.
• 21 (longue de1·iates ipsilaterally).

a
Lock
s
Suspend
8
End Bl ock
Item: 10 of 41 ~ 1 • Ma rk -<:J 1>- Jil ~· !:';-~
QIO: 5230 ..L Pre v ious Next Lab ~lues Notes Calcula t o r
& &
1 Posterior Lateral medulla: Lateral medullary (Wallenberg)
2 inferior 'ucleus ambiguus (C1 IX, X, XI ) Dy~ ph agia , hoarseness, l gag syndrome.
3
cerebellar Vestibular nuclei rcflc\ Nucleus ambiguus effects arc
artery Lateral spinothalamic tract, spinal Vomiti ng, vertigo, nystagmus specific to PICA lesions
4
trigeminal nucleus l pain and temperature sensation "Don't pick a (PICA) horse
5 (hoarseness) that can' t cat
from contra latera I body,
6 ipsilateral face (dysphagia)."
7 ympathetic fibers Ipsilateral Horner S) ndrome Also supplies inferior cerebellar
Inferior cerebellar peduncle Ataxia, d)Smetria peduncle (part of cerebellum).
8
9 Anterior Lateral pons Lateral pontine syndrome.
inferior Facial nucleus Paraly'i' of face, l lacrimation, Facial nucleus effects are
10
cerebellar l salin 1tion, l taste from anterior specific to AICA lesions.
• 11
artery ~ of tongue "Facial droop means AICA's
• 12 Vestibular nuclei Vomiting, vertigo, nystagmus pooped."
• 13 Spinothalamic tract, spinal l pain and temperature sensation Also supplies middle and
• 14 trigem ina! nucleus from conira lateral body, inferior cerebellar peduncles
ipsilatera I face (part of cerebellum).
• 15
Sympathetic fibers Ipsilatera I llorner sync! rome

I
• 16
Middle and inferior cerebellar taxia, dysmetria
• 17
peduncles
• 18
Basilar artery Pons, medulla, lower midbrain RAS spared, therefore prcscr"cd "Locked-in syndrome."
• 19 COnSCIOIISnCSS
• 20 Corticospinal and corticobulbar Quadriplegia; loss of ' olnntary
• 21 tracts facial, mouth, and longue

a
Lock
s
Suspend
8
End Bl ock
Item: 10 of 41 ~ 1 • M k -<:J 1>- Jil ~· !:';-~
QIO: 5230 ..L ar Pre v ious Next Lab~lues Notes Calcula t o r
& &
1 cerebral macular sparing.
2 artery
3
4
5
6
7
8
9
10
• 11

• 12
• 13 FA1 7 p 503.5
• 14 Vertigo Sensation of spinning while actua lly stationary. Subtype of "dizziness," but distinct from
• 15 "lightheadedness."
• 16 Peripheral vertigo More common. Inner car etiology (cg, semicircu lar canal debris, vestibular nerve infection,
• 17 Meniere disease, benign paroxysmal positional vertigo). Positional testing - delayed horizontal
. 18
nystagmus.
• 19 Central vertigo Brain stem or cerebellar lesion (eg, sl roke affecting' estibular nuclei or posterior fossa tumor).
Findings: directional or purely 'ertical n~stagmus, skew deviation, diplopia, dysmetria. Posit ional
• 20
testing - immediate nrstagmus in an) direction; may change directions. Focal neurologic
• 21
• findings .

a
Lock
s
Suspend
8
End Bl ock
Item: 11 of 41 ~ 1 • M k -<:J 1>- Jil ~· !:';-~
QIO: 3237 ..L ar Pre v ious Next Lab~lues Notes Calcula t o r
& &
1
A 15-year-old boy presents to his physician for a follow-up visit 2 months after he fell from a ladder and broke a bone. on physical examination, the
2 physician notes that the contour of the right shoulder is lost and appears flattened. Furthermore, the patient stat es that the lateral side of his
proximal arm feels like "it has fallen asleep." On strength testing, the left arm scores 5/5 on abducting against resistance, whereas the right arm
3 scores only 2/5. The rest of the examination results are normal.
4
5 What Is the most likely location of the fracture?

6 :
A. Acromion
7
8 B. Clavicle

9 c. Distal humerus
10
o. Proximal humerus
• 11
E. T1
• 12
• 13
• 14
• 15
• 16
• 17
• 18
• 19
• 20
• 21

a
Lock
s
Suspend
8
End Bl ock
Item: llof41 ~. , . M k <:] t> al ~· ~
QIO: 3237 .l. ar Previous Next Lab 'Vfll ues Notes Calculator

2 The co rrect a nswer is D. 60% cho se this.


3 The lost contour and flattening of this boy's shoulder are indicative of atrophy of the deltoid muscle. The deltoid is the main abductor of the glenohumeral
joint and is innervat ed by the axillary nerve. The axillary nerve winds around the surgical neck of the humerus, which is why it can be ea sily damaged
4 during fracture of the proximal end of the humerus. Deltoid muscle atrophy t akes weeks to occur. Axillary nerve damage can also produce an area of
numbness over the lat eral side of the proximal part of the arm .
5 Deltoid muscle Shoulder joint Humerus Axillary nerve Surgical neck of the humerus Muscle atrophy Atrophy Axilla Bone fracture Hypoesthesia
6 Anatomical terms of location Muscle

7 A is no t co rrect. 1 9 % cho se this.


8 The acromion is part of the scapula and just superior to the point at which the humerus articulat es with the glenoid fossa of the same bone. A fracture
here is usually the result of a downward-directed blow onto the shoulder. Although some fibers of the deltoid muscle originat e from the acromion, a
9 fracture here may explain only some pain-relat ed underuse atrophy; it does not explain the loss of sensation .
Deltoid muscle Acromion Scapula Humerus Glenoid fossa Glenoid cavity Bone fracture Bone Atrophy Fossa (anatomy) Muscle
10
B is no t co rrect. 1 2 % cho se this.
11
The axillary nerve is not locat ed near the clavicle. Fracture of the clavicle is most likely to occur between the middle and lat eral thirds, as this is the
• 12 weakest area . Fractures at this location do not typically injure any nerves of the brachial plexus.
Axillary nerve Clavicle Brachial plexus Fracture Bone fracture Axilla
• 13
• 14 c is no t co rrect. 6 % cho se this •
The axillary nerve is locat ed near the proximal humerus, not the distal humerus. Injuries to the midshaft-to -distal area s of the humerus can injure the
• 15 radial nerve. Remember that motor and sensory deficits due to a nerve injury are always distal to the point at which the patient experiences symptoms
Humerus Axillary nerve Radial nerve Anatomical terms of location Axilla Distal
• 16
E is no t co rrect. 3 % cho se this •
• 17
The axillary nerve arises from the posterior cord, and ultimat ely received its nerve fibers from the CS and C6 nerve roots. The T1 root contributes only to
• 18 the medial cord, and a fracture at this location could potentially affect the ulnar and median nerves, not the axillary nerve.
Axillary nerve Medial cord Posterior cord Axilla Cervical spinal nerve 6 Thoracic spinal nerve 1 Nerve root
• 19
• 20
• 21 Botto m Li ne:

6
lock
s
Suspend
0
End Block
Item: llof41 ~. , . M k <:] t> al ~· ~
QIO: 3237 .l. ar Previous Next Lab 'Vfll ues Notes Calculator

6
lock
s
Suspend
0
End Block
Item: 11 of 41 ~ 1 • M k -<:J 1>- Jil ~· !:';-~
QIO: 3237 ..L ar Pre v ious Next Lab~lues Notes Calcula t o r
& &
1
CONDITION INJURY CAUSES MUSCLE DEFICIT FUNCTIONAL DEFICIT PRESENTATION
2 Traction or Abduction (arm
Erb palsy ("waiter's Infants-lateral Deltoid,
3 tip") tear of upper traction on neck supraspinatus hangs by side)
4 ("Erb-er") trunk: during deli,·ery Infraspinatus Lateral rotation (arm
5 C5-C6 roots Adults-trauma medially rotated)
6 Biceps brachii Flexion, supination
7 (ann ex1ended and
8
pronated)
9
10 Klumpke palsy Traction or tear Infants-upward lntrinsic hand Total claw hand:
of lower trunk: force on arm muscles: lumbricals normallv
11
C8-TI root during delivery lumbricals, Aex YICP joints and
• 12
Adults-trauma interossei, extend DIP and PIP
• 13 (eg, grabbing a thenar, joints
• 14 tree branch to hypothenar
• 15 break a fall )
• 16 Thoracic outlet Compression Cervical rib Same as Klumpkc Atrophy of intrinsic
• 17
syndrome of lower trunk (arrows in fJ), palsy hand muscles;
and subclavian Pancoast tumor ischemia, pain,
. 18
''essels and edema
• 19 due to vascular
• 20 compressiOn
• 21

Winged scapula Lesion of long Axillary node

a
Lock
Serratus anterior Inability to anchor ... - s
Suspend
8
End Bl ock
Item: llof41 ~. , . M k <:] t> al ~· ~
QIO: 3237 .l. ar Previous Next Lab 'Vfll ues Notes Calculator

1 • •
Winged scapula Lesion of long Axillary node Serratus anterior Inability to anchor
2 thoracic nerve dissection after scapula to thoracic
3 mastectomy, cage -+ cannot
4 stab wounds abduct arm above
5
horizontal position

6
7
FA17 p 4 27.1
8 Upper extremity nerves
9 NERVE CAUSES OF INJURY PRESENTATION

10 Axillary (CS-C6) Fractured surgical neck of humerus; anterior Flattened deltoid


11 dislocation of humerus Loss of arm abduction at shoulder (> 15°)
Loss of sensation over deltoid muscle and lateral
• 12
arm
• 13
Musculocutaneous Upper trunk compression Loss of forearm Aexion and supination
• 14
(CS-C7) Loss of sensation over lateral forearm
• 15
Radial (C5-T1) Midshaft fracture of humerus; compression of Wrist drop: loss of elbow, wrist, and finger
• 16 axilla, eg, due to crutches or sleeping with arm extension
• 17 over chair ("Saturday night pa lsy") l grip str ength (wrist extension necessary for
• 18 maxima l action ofAexors)
Loss of sensation over posterior ann/forearm and
• 19
dorsal hand
• 20
M edian (CS-T1) Supracondylar fracture of humerus (proximal "Ape hand" and "Pope's blessing"
• 21
• lesion); carpal tunnel syndrome and wrist Loss of wrist Aexion, Aexion of lateral fingers, •

6
lock
s
Suspend
0
End Block
Item: llof41 ~. , . M k <:] t> al ~· ~
QIO: 3237 .l. ar Previous Next Lab 'Vfll ues Notes Calculator

1 • •
FA17 p 4 27.1
2 Upper extremity nerves
3 NERVE CAUSES OF INJURY PRESENTATION

4 Axillary (CS-C6) Fractured surgical neck of humerus; anterior Flattened deltoid


dislocation of humerus Loss of arm abduction at shoulder(> 15°)
5
Loss of sensation over deltoid muscle and lateral
6 arm
7 Musculocutaneous Upper trunk compression Loss of forearm Aexion and supination
8 (CS-C7) Loss of sensation over lateral forearm
9 Radial (CS-T1) Midshaft fracture of humerus; compression of Wrist drop: loss of elbow, wrist, and fin ger
10 axilla, eg, due to crutches or sleeping with arm extension
11 over chair ("Saturday night pa lsy") l grip strength (wrist extension necessary for
maxima l action ofAexors)
• 12
Loss of sensation over posterior ann/forearm and
• 13 dorsal hand
• 14 Median (CS-T1) Supracondylar fracture of humerus (proximal "Ape hand" and "Pope's blessing"
• 15 lesion); carpal tunnel syndrome and wrist Loss of wrist Aexion, Aexion of lateral fingers,
• 16 laceration (distal lesion) thumb opposition, lumbricals of 2nd and 3rd
• 17
digits
Loss of sensation over thenar eminence and
• 18
dorsal and palmar aspects of lateral 3Y2 lingers
• 19 with proximal lesion
• 20 Ulnar (C8-T1 ) Fracture of med ial epicondyle of humerus "Ulnar claw" on digit extension
• 21 "funny bone" (proximal lesion); fractured Radial deviation of wrist upon Aexion (proximal •

6
lock
s
Suspend
0
End Block
Item: 11 of 41 ~ 1 • M k -<:J 1>- Jil ~· !:';-~
Lab~lues
..L ar
.. . ..
QIO: 3237 Pre v ious Next Notes Calcula t o r

1
. .:
abduction and adduction of fingers (interossei),
2 actions of medial 2 lumbrical muscles
3 Loss of sensation over medial 11h fingers
4 including hypothenar eminence
5 Recurrent branch of Superficial laceration of palm " pe hand"
median nerve (CS-T1) Loss of thenar muscle group: opposition,
6
ahduct ion, and flexion of thumb
7 No loss of sensation
8
llumerus fractures, proximally to distally, folio" the \R\1 ( h iliar} .... Radial .... \ledian)
9
cs
10 Ax1llary nerve ~ g
~ Tl
~ Median nerve

nerve~
11
Axillary nerve~
• 12 Musculocutaneous '17 Ulnar nerve -....,...
/"Intercostobrachial
• 13 ~ Radial nerve ~ nerve
Rad1al nerve~ .._Radial nerve
• 14 /'"'"Medial brach~al
'!'io cutaneous nerve Palm of hand
• 15
• 16 Median nerve _A " ' -ulnar nerve
Musculocutaneous nerve~ Medial antebrachial
• 17 cutaneous nerve
Radial nerve /
• 18 ~ Ulnar nerve
Recurrent branch ..........._ Radial nerve-......,
• 19 of med1an nerve Radial nerve --....

• 20 Dorsum of hand
• 21

a
Lock
s
Suspend
8
End Bl ock
Item: 12 of 41 ~ 1 • M k -<:J 1>- Jil ~· !:';-~
QIO: 3380 ..L ar Pre v ious Next Lab~lues Notes Calcula t o r
& &
1
A 67-year-old man has a history of high cholesterol and hypertension and has smoked cigarettes for the past so years. While undergoing a stressful
2 divorce, the man suffers a massive stroke and collapses.

3
An Injury to the superior division of the left middle cerebral artery could be expected to produce which of the following symptoms?
4
5 :
A. Ataxia
6
B. Blindness
7
8 c. Expressive aphasia

9
D. Medial rectus palsy
10
E. Unilateral leg paralysis
11
• 12
• 13
• 14
• 15
• 16
• 17
• 18
• 19
• 20
• 21

a
Lock
s
Suspend
8
End Bl ock
Item: 12 of 41 ~ 1 • M k -<:J 1>- Jil ~· !:';-~
QIO: 3380 ..L ar Previous Next Lab~lues Notes Calculat or
& &
1
Th e correct a n sw er i s c. 700/o chose this.
2
The middle cerebral artery arises from the internal carotid artery and the circle of Willis and supplies the lateral aspect of the brain. If the right
3 hemisphere is compromised, you would expect to see left-sided weakness and sensory loss, gaze deviation to the right, and neglect of the left visual
space. If the left hemisphere is compromised, the findings are similar but on the opposite side. Furthermore, on the left the language centers are at risk.
4 Because the middle cerebral artery supplies both Broca and Wernicke speech areas, an injury to the middle cerebral artery can produce either expressive
aphasia (if Broca area is injured) or sensory aphasia (if Wernicke area Is Injured). Additional impairments may include difficulty with reading, writing, and
5 ca lculation.
6 Circle of W s Middle cerebral artery Aphasia Expressive aphasia Internal carotid a• te '( Wermcke's area Receptive aphasia carotid artery Cerebral hemisphere
Broca's a e Human brain Brain Lateralization of brain function Common carotid artery Cerebral arteries
7
A i s not correct. 6 0/o chose this.
8
Ataxia would result from a cerebellar injury, which could be produced by blockage of the cerebellar arte ri es or the posterior arte ries in the circle of Willis,
9 but not typically by injury to the middle cerebra l artery.
Circle of Willis Middle cerebral artery Ataxia Cerebellar artery Artery Cerebellum Cerebral arteries
10
B i s n ot co rrect . 50/o chose this.
11
Blindness could be the result of injury to the ophthalmic artery or anterior communicating artery, both of which would damage the optic nerve.
12 Alternatively, blindness could result from damage to parts of the visual cortex supplied by the posterior cerebral artery.
Anterior communicating artery Ophthalmic artery Posterior cerebral artery Optic nerve Visual cortex Visual impairment Cortex (anatomy) Ophthalmology
• 13
Cerebral arteries
• 14
D is not co rrect . 50fo chose this .
• 15
Medial rectus palsy, or gaze palsy, would be due to a lesion or occlusion of any artery supplying the gaze center in the frontal lobe, oculomotor nucleas In
• 16 the midbrain, paramedian pontine reticular formation in the pons, or anywhere along the medial longitudinal fasciculus. None of these structures are
supplied by the middle cerebral a1tery.
• 17 Paramedian pontine reticular formation Medial longitudinal fasciculus Frontal lobe Midbrain Middle cerebral artery Reticular formation Medial rectus muscle Pons
• 18 Conjugate gaze palsy Lesion Oculomotor nerve

• 19 E i s n ot correct. 14 0/o ch ose this .

• 20 Unilateral leg para lysis could be the result of an injury to the medial aspect of the motor cortex or to the internal capsule, which are supplied by the
anterior cerebra l artery and the latera l striate artery, respectively. Remember the motor homunculus.
• 21 Anterior cerebral artery Internal capsule Motor cortex Cortical homunculus Paralys•s Homunculus Cortex (anatomy)

a
Lock
s
Suspend
8
End Bl ock
Item: 12 of 41 ~ 1 • M k -<:J 1>- Jil ~· !:';-~
QIO: 3380 ..L ar Pre v ious Next Lab~lues Notes Calcula t o r
& &
1 Botto m Line:
2 Injury to the superior division of the middle cerebral artery can produce either expressive aphasia (if Broca area is injured) or sensory aphasia (if
Wernicke area is injured). It may also impair rea ding, writing, and calculation.
3 Expressive aphasia Middle cerebral artery Aphasia Wernicke's area Receptive aphasoa Broca's area Cerebral arteries
4
5
6 lilhfil!1j•) for year: 2017
F IRST AIO FACTS
"'
7
8 FA17p 486.1
9
Aphasia Aphasia-higher-order language de ~ cit (inabilit) to understand/speak/read/write).
10 Dysarthria-motor inability to spea ~ (movement de ~ cit).
11 TYPE SPEECH FLUENCY COMPREHENSION COMMENTS
12 Repetition impaired
• 13 Broca (expressive) t\'onAuent Intact Broca =Broken Boca (boca =mouth in Spanish).
• 14 Broc<l area in inferior frontal gyrus of frontal lobe. Patient
• 15 appears frustrated, insight intact.
• 16 Wernicke (receptive) F'luent lmpa ired Wernicke is Wordy but makes no sense. Patients do not
• 17
have in~ighl.
Wernicke area in superior temporal gyrus of temporal
. 18
lobe .
• 19
Conduction F'luent Intact Can be caused by damage to arC uate fasciculus.
• 20
Global 'onAuent Impaired Arcuate fasciculus; Broca and \Vemicke areas affected
• 21
• (a ll arcasl.

a
Lock
s
Suspend
8
End Bl ock
Item: 12 of 41 ~ 1 • M k -<:J 1>- Jil ~· !:';-~
QIO: 3380 ..L ar Pre v ious Next Lab~lues Notes Calcula t o r
& &
1 Re petition intact
2 Transcortical motor NonAuent Intact Affects frontal lobe around Broca area, but Broca area is
3 spared.
4 Transcortical sensory Fluent Impaired Affects tcmporallobc around Wernicke area, but
5 Wernicke area is spared.
6 Transcortical, mixed 'onAuent Impaired Broca and Wernicke areas and arcuate fasciculus remain
7 intact; surrounding watershed areas affected.
8
9 FA17 p 481 .1
10 Common brain lesions
11 AREA OF LESION CONSEQUENCE EXAMPLES

12 Frontal lobe Disinhibition and defi cits in concentration,


• 13
orientation, judgment; mar ha\'e reemergence
of primitive reflexes.
• 14
Frontal eye fields Eyes look toward lesion.
• 15
Paramedian pontine Eyes look away from side of lesion.
• 16
reticular formation
• 17
Medial longitudinal Internuclear ophthalmoplegia (impaired Multiple sclerosis.
. 18
fasciculus adduction of ipsilateral eye; nystagmus of
• 19 contralateral eye with abduction).
• 20
Dominant parietal Agraphia, acalculia, fin ger agnosia, left-right C erstmann syndrome.
• 21 cortex disorientation.

a
Lock Suspend
s 8
End Bl ock
Item: 12 of 41 ~ 1 • M k -<:J 1>- Jil ~· !:';-~
QIO: 3380 ..L ar Pre v ious Next Lab~lues Notes Calcula t o r
& &
1
Nondominant parietal Agnosia of the contralatera I side of the world. Hemispatial neglect syndrome.
2 cortex
3 Hippocampus Anterograde amnesia-inability to make new
4 (bilateral) memones.
5 Basal ganglia lay result in tremor at rest, chorea, atheto is. Parkinson disease, Huntington disease.
6 Subthalamic nucleus Contralateral hemiballismus.
7 Mammillary bodies Wernicke-Korsakoff syndrome- Confusion, Wernicke problems come in a C \:\ 0 ' beer.
8 (bilateral) Ataxia, :-..ystagmus, Ophthalmoplegia,
9 memory loss (anterograde and retrograde
10 amnesia), confabulation, persom1lit} changes.
11 Amygdala (bilateral) KIOver-Bucy syndrome-disinhibited behavior IISV-1 encephalitis.
(eg, hyperphagia, hypersexuality, hypcrorality).
12
• 13
Superior colliculus Parinaud syndrome-paralysis of conjugate Stroke, hydrocephalus, pinea loma.
vertical gaze (rostral interstitial nucleus also
• 14
involved).
• 15
Reticular activating Reduced levels of arousal and wakeful ness
• 16
system (midbrain) (eg, coma).
• 17
Cerebellar hemisphere Intention tremor, limb at·axia, loss of balance; Cerebellar hemispheres are laterally located-
. 18 damage to cerebellum - ipsilatera l deficits; affect lateral limbs.
• 19 fall toward side of lesion.
• 20 Cerebellar vermis Truncal ataxia, dysarthria. ermis is centrally located-affects central body.
• 21 Degeneration associated with chronic alcohol

a
Lock
s
Suspend
8
End Bl ock
Item: 12 of 41
QIO: 3380
~
..L
1 • Ma r k -<:J
Prev ious
I>
Next Lab
fJ
lues
£!1}>'

Notes
!!":-~
Cal culat o r

& &
1 FA17 p473.1
2 Circle of Willis ystem of anastomoses between anterior and posterior blood suppl ies to brain.
3
4
I
ACorn Anterior
commun~catmg - - -- - - --, ~-- Optoc chiasm

5
6
ACA
7
8
0
Antenot
ClfCUUtlon I
MCA Middle
ce.rebral ----.:::...
CIRCLE
./ ~MCA
ACA
9 OICA / / PCA
OF
10
D MCA
Posterior
I
PC 001 Posterior
communicating - - - - 4
WILLIS
~----BA
11 D Clrculat•on External

12 I
PCA Posterior_----:;
cerebral
ECA carotid
artery
"---..,....--PCom
~~~---!--ICA
Common " ' - - -Jii>.:l
• 13
CCA carotid
• 14 artery - - - _ _ ,11: I

• 15 I
SCA Superior
cerebellar _ __..) Pontine

• 16
• 17 I
AICA Anterior inferior
cerebellar -----4 Aorta
. 18
• 19 I
PICA Posterior Inferior
cerebellar ----~""\ OBLIQUE-LATERAL VIEW
• 20 INFERIOR VIEW
• 21

a
Lock
s
Suspend
8
End Bl ock
Item: 13 of 41 ~ 1 • M k -<:J 1>- Jil ~· !:';-~
QIO: 5 0 12 ..L ar Pre v ious Next Lab~lues Notes Calcula t o r
& &
1
A 71-year-old woman is brought into the emergency department by her daughter after she suddenly developed weakness in her left leg and began
2 having trouble ambulating. Three hours prior to admission, the patient was walking to get the mail and suddenly felt that she could not support her
weight. The patient has a medical history significant for hypertension, which Is currently treated with hydrochlorothiazide (HCTZ), and has smoked 1.5
3 packs per day for 25 years. On neurologic examination the patient is alert and oriented, but does not understand or remember that her left leg Is weak.
Cranial nerve examination is unremarkable. Motor examination revea ls 2/ 5 strength In the left leg and 5/ 5 strength in the right leg, right arm, and left arm.
4 Reflexes are 2+ throughout. A sensory examination is not performed.
5
6 Which of the following is the most likely cause of the patient's symptoms?

7 :
8 A. Basilar stroke

9 B. Left anterior cerebral artery infarct


10
c. Left middle cerebral artery infarct
11
o. Left posterior cerebral artery infarct
12
• 13 E. Right anterior cerebral artery infarct

• 14 F. Right middle cerebral artery infarct


• 15
G. Right posterior cerebral artery infarct
• 16
• 17
• 18
• 19
• 20
• 21

a
Lock
s
Suspend
8
End Bl ock
Item: 13 of 41 ~ 1 • M k -<:J 1>- Jil ~· !:';-~
QIO: 5 0 12 ..L ar Pre v ious Next Lab~lues Notes Calcula t o r
& &
1 Th e co rrect an sw er i s E. 680/o chose this.
2 Anterior cerebral artery (ACA) infarcts produce contralateral cortical-type leg paresis and weakness, amnesia, and frontal lobe behavioral abnormalities.
Because weakness presented in the left leg, the pathology is most likely seen In the right (contralateral) ACA . Given the sudden onset of symptoms, the
3 patient's age, and history of smoking and hypertension, the most likely diagnosis is right-sided ACA stroke.
Antenor cerebral artery Frontal lobe Hypertension Anatomical terms of ocation Contralateral Paresis Stroke Amnesia Infarction Pathology Cerebral arteries
4
5 A i s not correct. 2% chose t his.
Thrombosis of the basilar artery causes locked -in syndrome. However, basilar artery Infarctions usually involve its paramedian branches, leading to
6 Ischemia of the medial basis pontis and tegmentum; clinically, these patients may present with contralateral face, arm, and leg weakness, dysarthria, or
contralateral ataxia.
7
Loc ed-in synd orne Bas' lar artery Dysarthria Ataxia Ischemia Tegmentum Anatomi~ao terms of location Thrombosis Contralateral
8
B i s not correct. 5% chose this.
9 Anterior cerebral artery (ACA) infarcts produce contralateral cortical-type leg paresis and weakness, amnesia, and fronta l lobe behavioral abnormalities.
Although this patient's symptoms are consistent w ith an ACA stroke, the symptoms are localized to the left side, wh ich indicates a right-sided ACA Infarct.
10
Anterior cerebral artery Frontal lobe Paresis Infarction Contralateral Stroke Anatomical terms of location Amnesia Cerebral arteries
11
C is n ot co rrect. 3% ch ose t his.
12 Middle cerebral artery (MCA) Infarcts typically Involve aphasia, hemlneglect, hemianopia, and contralateral (right sided for the left MCA) face-arm or face-
arm-leg sensorimotor loss, depending on the regions Involved. This patient has sensorimotor deficit mainly of the leg, which more closely resembles
13
Involvement of the ACA .
• 14 Middle cerebral artery Hemispatial neglect Aphasia Contralateral Hemianopsia Infarction Anatomical terms of location Stroke Cerebral arteries

• 15 D is no t co rrect . 20/o ch ose this .

• 16 Posterior cerebral artery (PCA) infarcts often cause a contralateral homonymous hemianopsia . At times, small penet ra ting vessels originating from the
PCA are also Involved, which results in contra lat eral sensory loss, contralateral hemiparesis, or thalamic aphasia.
• 17 Homonymous hemianopsia Posterior cerebral artery Hemiparesis Aphasia Thalamus Contralateral Hemianopsia Anatomical terms of location Cerebral arteries
Infarction Stroke
• 18
• 19 F i s n ot correct. 160/o ch ose this .
Middle cerebral artery (MCA) infarcts are the most common type of intracranial infarctions. These infarcts typically involve aphasia, hemineglect,
• 20 hemianopsia, and contralatera l (left sided for the right MCA) face-arm or face-arm-leg sensorimotor loss, depending on the regions of the MCA Involved.
• 21 This patient has sensorimotor deficits mainly of the leg, which more closely resembles involvement of the ACA .
• Middle ce eb a arterv Hemisoatial neolect Aohasia Contralateral Hemianoos•a Infa1 ctlon Stro .e Anatomical terms of location

a
Lock
s
Suspend
8
End Bl ock
Item: 13 of 41 ~. I • M k <:] t> al ~· ~
QIO: 5012 .l. ar Previous Next lab 'Vfl1 ues Notes Calculator

1 F is not correct. 16% chose this .


2 Middle cerebral artery (MCA) infarcts are the most common type of intracranial infarctions. These infarcts typically involve aphasia, hemineglect,
hemianopsia, and contralat eral (left sided for the right MCA) face -arm or face -arm-leg sensorimotor loss, depending on the regions of the MCA involved.
3 This patient has sensorimotor deficits mainly of the leg, which more closely resembles involvement of the ACA .
4 Middle cerebral artery Hemispatial neglect Aphasia Contralateral Hemianopsia Infarction Stroke Anatomical terms of location

5 G is not correct. 4 % chose this .


Posterior cerebral artery (PCA) infarcts often cause a contralat eral homonymous hemianopsia. At times small penetrating vessels originating from the PCA
6 are also involved, which results in contralat eral sensory loss, contralat eral hemiparesis, or thalamic aphasia.
7 Homonymous hemianopsia Posterior cerebral artery Hemiparesis Aphasia Thalamus Contralateral Hemianopsia Anatomical terms of location Cerebral arteries
Infarction Stroke
8
9
10 Bottom Line:
Anterior cerebral artery infarcts produce contralat eral leg paresis and weakness, amnesia, and frontal lobe behavioral abnormalities.
11
Anterior cerebral artery Frontal lobe Anatomical terms of location Paresis Contralateral Amnesia Infarction Stroke
12
13
• 14 I iii I;fi 1!1 I•J for year:l 2017 ..
FI RST AI D FA CTS
• 15
• 16
FA17 p473.1
• 17
Circle of Willis System of anastomoses between anterior and posterior blood supplies to brain.
• 18
I
ACorn Anterior

l.
• 19 communicating ~-- Optrc chrasm

• 20
ACA I cerebral
Anterior ·-
-~
• 21
• ~ ~ Internal carotid I ICA

6
lock
s
Suspend
0
End Block
Item: 13 of 41 ~ 1 • M k -<:J 1>- Jil ~· !:';-~
QIO: 5 0 12 ..L ar Pre v ious Next Lab~lues Notes Calcula t o r
& &
1 FA17 p473.1
2 Circle of Willis System of anastomoses between anterior and posterior blood supplies to brain.
3 I
ACorn Anterior . .
communacating - - -- - - -..., ~-- Optic chiasm
4
5 ..

~'"'-:'1 "
6
ACA
7
8
0
Anterior
ClfCUIMJOI! I
MCA Middle
cerebral ----.:::.
CIRCLE

,.L stna e
./ ~ MCA
O ACA
/ /PCA
O ICA OF ~
9
10
O MCA
Posterior
I
PC 001 Posterior
communicating - - - - 4
WILLIS
~ ~~
D Clrcui.Jt•on External t&-----BA
11 ECA carotid l f'- -:---PCom
12 artery ·~.-----:.--ICA
Common '---~
13 CCA carotid
artery ---!.......11iJ
• 14
• 15
I
SCA Superior
cerebellar _ __, Pont1ne Brachio·
cephalic _.:.._ __;,~"
• 16
• 17 I
AICA Anterior inferior
cerebellar - - - - -
Aota
. 18
• 19 I
PICA Posterior 1nfenor
cerebellar ---~ OBLIQUE-LATERAL VIEW
• 20 INFERIOR VIEW
• 21

a
Lock
s
Suspend
8
End Bl ock
Item: 13 of 41 ~ 1 • M k -<:J 1>- Jil ~· !:';-~
QIO: 5 0 12 ..L ar Pre v ious Next Lab~lues Notes Calcula t o r
& &
1 FA17 p 484.1
2 Effects of strokes
ARTERY AREA OF LESION SYMPTOMS NOTES
3
4
Ante rior circulation
5 Middle ~lotor and sensory cortices rJ- upper Contralateral para I) sis and sensOr) Wernicke aphasia is associated
cerebral limb and face. loss-face and upper limb. with right superior quadrant
6
a rtery Temporal lobe (Wernicke area); ph asia if in dominant (usually visual field defect due to
7 frontal lobe (Broca area). left) hemisphere. llemineglecl temporal lobe involvement.
8 if bion affects nondominant
9 (usually right) side.
10 Anterior l'.Jotor and sensory cortices-lower Contralateral paralysis and sensory
cerebral limb. loss-lower limb.
11
artery
12
Lenticulo- Striatum, internal capsule. Contralateral para I) sis and/or Common location of lacunar
13
striate sensory loss-face and body. infarcts rn,due to hyaline
• 14 artery Absence of cortical signs arteriosclerosis zoto
• 15 (eg, neglect, aphasia, visual field unmanaged hypertension.
• 16 loss).
• 17 Posterior circulation
• 18 Ante rior Lateral corticospinal tract. Contralateral paralysis-upper and Medial medullary syndrome-
• 19
spinal lo"er limbs. caused by infarct of
a rtery t\ ledial lemniscus. ~ contra l ateral proprioception. paramedian branches of /\SA
• 20
Caudal medulla-hypoglossal nerve. Ipsilateral h) poglossal dysfunction and/or "ertebral arteries.
• 21

(longue de,·iales ipsilaterally).

a
Lock
s
Suspend
8
End Bl ock
Item: 13 of 41 ~ 1 • Ma rk -<:J 1>- Jil ~· !:';-~
QIO: 5 0 12 ..L Pre v ious Next Lab ~lues Notes Calcula t o r
& &
1 Posterior Lateral medulla: Lateral medullary (Wallenberg)
2 inferior 'ucleus ambiguus (C l IX, X, XI ) Dy~ ph agia , hoarseness, l gag syndrome.
3 cerebellar Vestibular nuclei rcflc\ lucleus ambiguus effects are
artery Lateral spinothalamic tract, spinal Vomiting, vertigo, nystagmus specific to PICA lesions
4
trigeminal nucleus l pain and temperature sensation "Don't pick a (PICA) horse
5 (hoarseness) that can' t eat
from contralateral body,
6 ipsilateral face (dysphagia)."
7 ympathetic fibers Ipsilateral Horner S) ndrome Also supplies inferior cerebellar
Inferior cerebellar peduncle Ataxia, d)Smetria peduncle (part of cerebellum).
8
9 Anterior Lateral pons Lateral pontine syndrome.
10 inferior Facial nucleus Pa ra lr~i\ uf face, l lacrimation, Facial nucleus effects are
cerebellar l sali,·e~tion, l te~ste from anterior specific to AICA lesions.
11
artery ¥1 of tongue ''Facial droop mee~ns AICA's
12 pooped."
Vestibular nuclei Vomiting, vertigo, nystagmus
13 Spinothalamic tract, spinal l pe~ in and temperature sensation Also supplies middle and
• 14 trigeminal nucleus from conira lateral body, inferior cerebellar peduncles
ipsilatera I face (part of cerebellum).
• 15
• 16
Sympathetic fibers Ipsi latera I llorner synd rome
liddle and inferior cerebellar Ataxia, dysmetria
• 17
peduncles
• 18
Basilar artery Pons, medulla, lower midbrain RAS spared, therefore presen·ed "Locked-in syndrome."
• 19
COnSCIOIISI1CSS
• 20 Corticospinal and corticobulbar Que~driplcgia; loss of' olnntary
• 21 tract~ facial, mouth, and tongue

a
Lock
s
Suspend
8
End Bl ock
Item: 13 of 41 ~ 1 • M k -<:J 1>- Jil ~· !:';-~
QIO: 5 0 12 ..L ar Pre v ious Next Lab~lues Notes Calcula t o r
&
.. .. .. - ~ ~ -
&
1
peduncles
2
Basilar artery Pons, medulla, lower midbrain RAS spared, therefore preserved "Locked-in syndrome."
3 eonSCIOIISness
4 Corticospinal and corticobulbar Quadriplegia; loss of' oluntary
5 tracts facial, mouth, and longue
6
1110\·ements
Ocular cranial nerve nuclei, Loss of horizontal, but not vertical,
7
paramedian pontine reticular e\e 1110\ements
8 '
formation
9
Posterior Oecipitallobe [!]. Contralateral hemianopia with
10 cerebral macular sparing.
11 artery
12
13
• 14
• 15
• 16
• 17
. 18
• 19
• 20
• 21

a
Lock
s
Suspend
8
End Bl ock
Item: 14 of 41 ~ 1 • M k -<:J 1>- Jil ~· !:';-~
QIO: 5227 ..L ar Pre v ious Next Lab~lues Notes Calcula t o r

1
&

A neurologic examination is performed on a 40-yea r-old man complaining of decreased sensation in his lower right extremity. Cranial nerve
examination is normal. Muscle strength is intact throughout . The patient has not noticed trouble with balance, but an abnormality is noted In one part
IA•A] &

2
of the examination, as shown in the video clip .
3
OPEN MEDIA
4
5
6 Which neurologic structure is most likely affected in this patient?

7 :
8 A. Basal ganglia

9 B. Cerebellum
10
c. Medial lemniscus
11
o. Spinothalamic tract
12
13 E. Tectospinal tract

• 14
• 15
• 16
• 17
• 18
• 19
• 20
• 21

a
Lock
s
Suspend
8
End Bl ock
Item: 14 of 41 ~. I • M k <:] t> al ~· ~
QIO: 5227 .l. ar Previous Next lab 'Vfl1 ues Notes Calculator

1 The co rrect a nswer is c. 40% cho se this.


2 The video shows the patient undergoing the Romberg t est, which eva luat es the patient's balance in the absence of visual cues. A positive Romberg sign
indicat es a problem with proprioception or the vestibular apparatus. In this case, the patient's sensory deficits in addition to his positive Romberg sign
3 suggest a lesion in the medial lemniscus, the pathway arising from the dorsal columns of the spinal cord that carries information about position sense,
4
5
6
vibration, and light touch .
Proprioception Medial lemniscus Romberg's test Vestibular system Spinal cord Posterior column lesion Vibration Anatomical terms of location

A is no t co rrect. 8% cho se this.


The basal ganglia is a deep structure consisting of the striatum, pallidum, substantia nigra, and subthalamic nucleus. It plays a role in motor control and
motivation and would not affect somatic sensation or balance.
I
7
Basal ganglia Substantia nigra Subthalamic nucleus Striatum Ganglion Motor control Somatosensory system Globus pallidus
8
B is no t co rrect. 32 % cho se this.
9 The cerebellum is a posterior structure essential for motor coordination . Although a lesion in the cerebellum would disrupt the ability to maintain balance,
this impairment would be evident both before and after the patient closed his eyes for the Romberg t est. Moreover; a lesion in the cerebellum would not
10 explain the patient's somatosensory deficits.
11 Cerebellum Romberg's test lesion Somatosensory system Motor coordination

12 D is no t co rrect. 1 2 % cho se this.


The spinothalamic tract carries information about pain and t emperature, and a lesion to it would not impair balance.
13 Spinothalamic tract lesion
14
E is no t co rrect. 8% cho se this •
• 15 The t ectospinal tract is responsible for hea d movements in response to visual signals. A lesion in this t ra ct would not lea d to a positive a Romberg sign.
Tectospinal tract Romberg's test
• 16
• 17

• 18 Botto m Li ne:

• 19 A positive Romberg sign may be due to impaired proprioception, suggesting a lesion in the dorsal column-medial lemniscus tract, the pathway
responsible for carrying information on proprioception, vibration, and light touch .
• 20 Proprioception Dorsal column-medial lemniscus tract Romberg's test lesion Vibration Anatomical terms of location Dorsum (biology)

• 21

6
lock
s
Suspend
0
End Block
Item: 14 of 41 ~ 1 • M k -<:J 1>- Jil ~· !:';-~
QIO: 5227 ..L ar Pre v ious Next Lab~lues Notes Calcula t o r
& &
1 FA17 p 500.1
2 Spinal cord lesions
AREAAFFECTED DISEASE CHARACTERISTICS
3
4
Poliomyelitis and Werdnig-Hoffmann Congenital degeneration of anterior horns of spinal
disease cord. LM ' lesions only. "Floppy baby" with marked
5
6
•• hypotonia and tongue fasciculations. Infantile type
has median age of death of 7 months. Autosomal
7
8
9
recessive inheritance.
Poliomyelitis - asymmetric weakness.
\Verclnig-1-loffmann disease - symmetric weakness.
I
Amyotrophic lateral sclerosis Combined U \1 1\ and L~ I N deficits with no sensor\•
10
11

12
13
•• • • or bowel/bladder deficits (due to loss of cortical and
spina I cord motor neurons, respectively).
Can be caused by defect in superoxide dismutase I.
Commonly presents with asymmetric limb weakness
(hands/feet), fascicu lations, eventual atrophy. Fatal.
14
Commonly known as Lou Gehrig disease.
• 15 Treatment: riluzole .
• 16 Complete occlusion of anterior Spares dorsal columns and Lissauer tract; mid-
• 17 spinal artery thoracic ASA territory is watershed area, as artery
. 18 of Adamkiewicz supplies ASA below- T8. Presents
with UM deficit below the lesion (corticospinal
• 19
tract), L 1 deficit at the b ·el of the lesion (anterior
• 20 horn), and loss of pain and temperature sensation
• 21 below the lesion (spinothalamic tract).

a
Lock
s
Suspend
8
End Bl ock
Item: 14 of 41 ~ 1 • M k -<:J 1>- Jil ~· !:';-~
QIO: 5227 ..L ar Pre v ious Next Lab~lues Notes Calcula t o r
& &
1
Tabes dorsalis Caused by 3° syph ilis. Results from degeneration
2 (demyelination) of dorsal columns and roots
3 _. progressi,·e sensory ataxia (impaired
4
proprioception -.. poor coordination).
Associated" ith Charcot joints, shooting pain, Arg) II
5
Robertson pupils.
6 Exam" ill demonstrate absence of DTRs and
7 Et> Romberg sign.
8 Syringomyelia yrinx expands and damages anterior white

I
9 commissure of spinothalamic tract (2nd-order
neurons) - bilateral loss of pain and temperature
10
11
• sensation in cape-like distribution; seen "ith Chiari
!malformation; can expand and affect other tracts.
12
13
, Vitamin 812 deficiency Subacute combined degeneration (SCD)-
dcmyclination of Spinocerebellar tracts, lateral
14
• 15
• 16
' Cauda equina syndrome
C orticospinal tracts, and Dorsal columns. Ataxic
gait, paresthesia, impaired position/vibration sense.

Unilatera l symptoms including radicular pain, absent


• 17 knee and ankle reA ex, loss of bladder and an:~ I
. 18 sphincter control. Can cause saddle anesthesia .
• 19 Treatment: emergent surgery and steroids.
Due to compression of spinal roots from L2 and
• 20
below, often caused by intra,·ertebral disk herniation
• 21 or tumors.

a
Lock
s
Suspend
8
End Bl ock
Item: 14 of 41 ~ 1 • M k -<:J 1>- Jil ~· !:';-~
QIO: 5227 ..L ar Pre v ious Next Lab~lues Notes Calcula t o r
& &
1 FA17 p 484.1
2 Effects of strokes
3 ARTERY AREA OF LESION SYMPTOMS NOTES

4 Anterior circulation
5 Middle ~I alor and sensor) cortices rJ- upper Contralateral para I) sis and sensor~ Wernicke aphasia is associated
6
cere bral limb and face. loss-face and upper limb. with right superior quadrant
arte ry Temporal lobe (Wernicke area); ph asia if in dominant (usually 1·isual field defect due to
7
fronta l lobe (Broca area). left) hemisphere. llemineglecl temporal lobe involvement.
8 if bion affects nondominant
9 (usuall}' right) side.
10 Ante rior lotor and sensory cortices-lower Contralateral paralysis and sensory
11 cere bral limb. loss-lower limb.
12
13
14
arte ry
Le nticulo-
striate
arte ry
Striatum, internal capsule. Contralateral paralysis and/or
sensory loss-face and body.
Absence of cortical signs
Common location of lacunar
infarcts rn,due to hyaline
arteriosclerosis zoto
I
• 15 (eg, neglect, aphasia, visual field unmanaged hypertension.
• 16 loss).
• 17 Posterior circulation
• 18 Ante rior Lateral corticospinal tract. Contralateral para lrsis-upper and Medial medullary syndrome-
• 19 spinal lo11 er limbs. caused by infarct of
• 20
a rte ry tv led ial lemn iscus. l contralateral proprioception. paramedian branches of /\SA
Caudal medulla-hypoglossal nerve. lp ilateral h) poglossa l dysfunction and/or vertebral arteries.
• 21 (longue de1·iates ipsilateral h-).

a
Lock
s
Suspend
8
End Bl ock
Item: 14 of 41 ~ 1 • M k -<:J 1>- Jil ~· !:';-~
QIO: 5227 ..L ar Pre v ious Next Lab~lues Notes Calcula t o r
& &
1 FA17 p 479.1
2 Spinal tract anatomy Ascending tracts synapse and then cross.
3 and functions
4 TRACT FUNCTION 1ST-ORDER NEURON SYNAPSE 1 2ND-ORDER NEURON SYNAPSE 2+PROJECTIONS

5 Ascending tracts
6 Dorsal column Pressure, Sensory nerve "-:ucleus Oecussates
7 vibration, ending - b) pass graeili~. in medulla
fine touch, pseudounipolar cell nucleus - ascends
8
proprioception body in dorsal root cuneatus contra laterally
9
ganglion - enter (ipsilatera I in medial
10 spinal cord - ascend medulla) lemniscus
11 ipsilaterally in dorsal
columns VPL (thalamus)
12
- sensory cortex
13 Spinothalamic tract Lateral: pain, Sensory nerve Ipsilateral gray Oecussates at
14 temperature ending (AO and C mat ter (spinal anterior white
Anterior: fibers) - bypass cord) commtssure
• 15
crude touch, pseudounipolar cell - ascends
• 16
pressure body in dorsal root con tra laterally
• 17 ganglion - enter
spinal cord

I
• 18
• 19 Descending tract
• 20 Lateral corticospinal oluntarv UM1 :cell body in Cell body of LM1 : lea,·es 1 \11 J - muscle
'
• 21 tract mo,·ement of Io motor cortex - anterior horn spinal cord fibers

a
Lock Suspend
s 8
End Bl ock
Item: 15 of 41 ~ 1 • M k -<:J 1>- Jil ~· !:';-~
QIO: 2830 ..L ar Pre v ious Next Lab~lues Notes Calcula t o r
& &
1
A 60-year-old man comes to the physician beca use of a persistent headache that has worsened over the past several months, in addition to
2 worsening erectile dysfunction. He m entions that he feels "clumsy" at times, repeatedly colliding into door fram es because he "can't see out of the
corners of my eyes."
3
4 If asked specifically, what additional symptom is t he patient most likely to report?
5
:
6 A. Diminished libido
7
B. Dropp1ng things with his right hand
8
C. I nability to raise the right corner of his mouth when smiling
9
10 D. Painful swallowing

11 E. Urinary incontinence
12
13
14
0 15
0
16
0
17
0
18
• 19
0 20
• 21

a
Lock
s
Suspend
8
End Bl ock
Item: 15 of 41 ~ 1 • M k -<:J 1>- Jil ~· !:';-~
QIO: 2830 ..L ar Prev ious Next Lab~lues Notes Calculat o r

& &
1
2
The correct an sw er i s A. 5901o chose this.
3 The vignette describes a presentation of prolactinoma, the most common tumor of the
4 pituitary gland. Signs and symptoms are a consequence of excessive prolactin secretion and Signs/Symptoms Signs/Symptoms Due
local mass effects on suprasellar structures. Among these structures Is the optic chiasm, Due to Mass to Increased Prolactin
5 which Is situated immediately superior to the pituitary gland. Compression of the optic
chiasm by pituitary tumors characteristically causes a bitemporal hemianopia, or tunnel Effect Secretion
6 vision (the chiasm contains crossing fibers originating in the medial retinas, which transmit Women: amenorrhea,
signals from the lateral fields of view). This lesion often causes patients to miss objects that Bitemporal
7 galactorrhea, infertility,
present In their temporal fields, such as door frames and cars in adjacent lanes. Often the hemianopsia
diminished libido
8 prolactlnoma also causes sexual adverse effects by indudng a hypogonadotroplc
hypogonadism caused by the inhibitory effects of prolactin. The classic presentation in
-
9 Men: diminished libido,
women Includes infertility, amenorrhea, and galactorrhea. I n men, the effects Include Chronic headache
diminished libido, erectile dysfunction, and gynecomastia. Prolactinoma usually affects impotence, gynecomastia
10
women <40 years old, but men who are affected are usually older. Furthermore, a chronic
11 headache that increases in severity is a sign of a brain tumor.

12 The signs and symptoms of prolactinoma are shown in the table.

13 Prolactinoma Bitemporal hemianopsia Gynecomastia Galactorrhea Optic chiasm Erectile dysfunction Prolactin Pituitary gland Hypogonadism Amenorrhoea
Sella turcica Headache Ubido Hypogonadotropic hypogonadism Brain tumor Pituitary adenoma Gland Neoplasm Infertility Lesion Brain Hemianopsia Human brain
14
8 is not co rrect. 80fo chose this.
15
Right-handed weakness that causes the affected individual to drop objects he or she Is holding can occur as the result of tumor compression or Infiltration
• 16 Into the motor cortex, or precentral gyrus, of the left frontal lobe. However, this finding is more likely in a transient ischemic attack or stroke of these
regions. A tumor of the pituitary gland generally cannot cause this finding via local compression of surrounding structures.
• 17 Transient ischemic attack Frontal lobe Precentral gyrus Pituitary gland Motor cortex Stroke Neoplasm Ischemia
• 18 C i s n ot correct. 8 0/o chose this .
• 19 Right facial droop, which may manifest as inability to raise the corner of the mouth when smiling, can occur due to compression or ischemia of the motor
cortex within the frontal lobe, as well as compression or damage to cranial nerve VII (Bell palsy). However, a tumor of the pituitary generally would not
• 20 result In this finding.
• 21 Fronta obe Be 's palsy Cramal nerves Ischemia Motor cortex Facia !lerve Pituitary gland Neoplasm Pituitary adenoma

a
Lock
s
Suspend
8
End Bl ock
Item: 15 of 41 ~- I • M k <:] t> al ~· ~
QIO: 2830 .l. ar Previous Next lab 'Vfl1 ues Notes Calculator

1 D is not correct. 6% chose this.


Painful swallowing, or odynophagia, can occur with compression or ischemia of the medulla involving cranial nerves IX and X or the motor cortex within
2 the frontal lobe. A tumor of the pituitary generally would not result in this finding .
3 Frontal lobe Cranial nerves Motor cortex Ischemia Neoplasm Medulla oblongata Pituitary gland Cortex (anatomy)

4 E is not correct. 19% chose this.


Urinary incontinence is a presenting symptom of normal-pressure hydrocephalus. The classic triad associated with this disease comprises urinary
5 incontinence, gait ataxia, and dementia . Because this patient did not report any of these symptoms but instead has other complaints that are associated
6 with a pituitary tumor; this is not the best answer.
Hydrocephalus Urinary incontinence Ataxia Symptom Fecal incontinence Gait abnormality Dementia Gait (human) Pituitary gland Neoplasm Gait
7

8
Bottom Line:
9
Prolactinomas cause bitemporal hemianopia due to compression of the optic chiasm . They also cause erectile dysfunction, decreased libido,
10 galactorrhea, and amenorrhea due to prolactin hypersecretion.
Bitemporal hemianopsia Galactorrhea Optic chiasm Erectile dysfunction Prolactin Amenorrhoea Hyperprolactinaemia libido
11

12
13
lijj ;fi IJ l•l for year:l 2017 ..
14 FI RST AID FAC T S

15
• 16 FA17 p 332.3

• 17 Pituitary adenoma Benign tumor, most commonly prolactinoma (arises from lactotrophs). Adenoma rl1 may be
• 18 funct ional (hormone producing) or nonfunctional (silent). !on functional tumors present with
mass effect (bitemporal hemianopia, hypopituitarism, headache). Functional tumor presentation
• 19
is based on the hormone produced.
0 20
Prolaclinoma in women classically presents as galactorrhea, amenorrhea, and ' bone density due to
0
21 suppression of estrogen. Prolactinoma in men classically presents as low libido and infertility.

6
lock
s
Suspend
0
End Block
Item: 15 of 41 ~. I • M k <:] t> al ~· ~
QIO: 2830 .l. ar Previous Next lab 'Vfl1 ues Notes Calculator

1 • •
FA17 p496.1
2 Adult primary brain tumors
3 TUMOR DESCRIPTION HISTOLOGY

4 Glioblastoma Common, highly malignant 1° brain tumor with Astrocyte origin, CFAP ® . "Pseudopalisading"
multiforme (grade IV ~ 1-year median survival. Found in cerebral pleomorphic tumor cells [lJ border central
5
astrocytoma) hemispheres rJ. Can cross corpus callosum areas of necrosis and hemorrhage.
6
("butterfly glioma").
7
Oligodendroglioma Relatively rare, slow growing. Most often in Oligodendrocyte origin. "Fried egg" cells-
8
frontal lobes ~- "Chicken-wire" capillary round nuclei with clear cytoplasm [!]. Often
9 pattern. calcified.
10
Meningioma Common, typically benign 1° brain tumor. Arachnoid cell origin. Spi ndle cells
11 Most often occurs near surfaces of brain and concentrically arranged in a whorled
12 in parasagittal region. Extra-axial (external pattern; psammoma bodies D (laminated
13 to brain pa renchyma) and may have a dural calcifications).
14
attachment ("tail" 0). Often asymptomatic;
may present with seizures or focal neurologic
15
signs. Resection and/or radiosurgery.
• 16
Hemangioblastoma Most often cerebel lar 1!1. Associated with von Blood vessel origin. Closely arranged, thin-
• 17
Hippel-Lindau synd rome when found with walled capillaries with minimal interveni ng
• 18 retinal angiomas. Can produce erythropoietin parenchyma CJ.
• 19 .... zopolycythemia .
• 20 Pituitary adenoma Adenoma may be nonfunctioning or Hyperplasia of only one type of endocrine cells
• 21 hyperfunctioning. Most commonly found in pituitary (ie, lactotroph, gonadotroph, •

6
lock
s
Suspend
0
End Block
Item: 15 of 41 ~ 1 • M k -<:J 1>- Jil ~· !:';-~
QIO: 2830 ..L ar Pre v ious Next Lab~lues Notes Calcula t o r
&
1 Pituitary adenoma Adenoma may be non functioning or llyperplasia of only one type of endocrine cel ls
&

2 hyperfunctioning. Most commonly found in pituitary (ie, lactotroph, gonadotroph,


3 from lactotrophs (prolactinoma) somatotroph, corticotroph).
- h~ perprolactinemia; less commonly
4
adenoma of somatotrophs (GH ) - acromegaly/
5
gigantism; corticotrophs ( CTH) - Cushing's
6 disease. Rare!~. adenoma of th} rotrophs (TSII )
7 and gonadotroph (FSI I, Lll ). Bitemporal
8 hemianopia due to pressure on optic chiasm
(D shows normal visual field abO\e, patient's
9
perspecti,·e below). Sequelae include hyper-
10
or hypopituitarism, which may be caused by
11 pituitary apopb.:y.
12 Schwan noma Classically at the cerebellopontine angle 13. Schwann cell origi n 0, S-100 G:>.
13 but can be along any peripheral nerve. Often
14 localized to CN VIIi in internal acoust ic
15 meatus~ vestibular sehwannoma. Bilat·eral
vestibular schwannomt~s found in W-2 .
• 16
Resection or stereotactic radiosurgery.
• 17
. 18
• 19
• 20
• 21

a
Lock
s
Suspend
8
End Bl ock
Item: 15 of 41
QIO: 2830
~
..L
1 • Ma r k -<:J
Pre v ious
I>
Next Lab
fJ
lues
£!1}>'

Notes
!!":-~
Calcula t o r
& &
1
2
3
4
5
6
7
8
9
10
11
12
13
14
15
• 16
• 17
• 18
• 19
• 20
• 21

a
Lock
s
Suspend
8
End Bl ock
2
3
4
5
FA17 p 315.2
6 Hypothalamic-pituitary hormones
7 HORMONE FUNCTION CliNICAL NOlES
8 CRH t ACTH, ~ I SH, ~-endorphin l in chronic exogenous steroid use.
9
Dopamine l prolactin, TSH Dopamine antagonists (eg, antipsychotics) can
10 cause galactorrhea due to hyperprolactinemia.
11 GHRH t CH Analog (tcsamorelin) used to treat
12 1-1 IV-<•ssociated lipodystrophr.
13 GnRH t FSH, LH Suppressed by hyperprolac tinemia.
14 Tonic Cn RH suppresses H PC axis.
15
Pulsatile CnRH leads to puberty, fertility.

• 16 Prolactin l Cn RH Pituitary prolactinoma -+ amenorrhea,


osteoporosis, hypogonadism, galac torrhea .
• 17
• 18
Somatostatin l C H , TSH Analogs used to treat acromegaly.

• 19 TRH t TSI I, prolactin t TRI [ (eg, in l 0 /2° hypothyroidism) may


increase prolactin secretion -+ galactorrhea.
• 20
• 21

a
Lock
s
Suspend
8
End Block
Item: 16 of 41 ~ 1 • M k -<:J 1>- Jil ~· !:';-~
QIO: 3773 ..L ar Pre v ious Next Lab~lues Notes Calcula t o r

An 80-year-old man is brought to the emergency department by his daughter after his first seizure. For months, he has taken exceptionally short
&
1
steps, unable to raise his legs. He is incontinent of urine and does not "know when to go." His recent memory is also impaired, and he denies having
2 fallen despite evidence to the contrary. There is no history of stroke or Intracranial mass lesions. Cranial nerves are intact, muscle strength Is normal,
and sensorium is normal. The patient sways during Romberg's test with eyes open or closed . ACT scan of the head is shown in the image. Cerebrospinal fluid
3
pressure Is normal.
4
5
6
7
8
9
10
11
12
13
14
15
• 16
• 17
• 18
• 19
• 20
• 21

a
Lock
s
Suspend
8
End Bl ock
What Is the most likely etiology of the dilat ed ventricles?

:
A . Choroid plexus papilloma

B. Communicating hydrocephalus

c. Hydranencephaly

D. Hypervitaminosis A

E. Noncommunicating hydrocephalus

a
Lock
s
Suspend
8
End Block
Item: 16 of 41 ~. I • M k <:] t> al ~· ~
QIO: 3773 .l. ar Previous Next lab 'Vfllues Notes Calculator

1 • The co rrect a nswer is B. 58 % cho se this.


2 This patient has normal pressure hydrocephalus (NPH) which is a condition of the elderly characterized by chronically dilat ed ventricles. Patients with NPH
typically present with the following clinical triad : urinary incontinence, gait difficulties, and mental decline (remember the mnemonic: W et, W obbly, and
3 W acky) . Although the exact mechanism is not known, NPH is believed to be a form of communicating hydrocephalus in which cerebrospinal fluid
reabsorption is impaired at the arachnoid villi.
4 Normal pressure hydrocephalus Hydrocephalus Cerebrospinal fluid Arachnoid granulation Urinary incontinence Arachnoid mater Ventricular system Gait (human)
5 Ventricle (heart) Gait

6 A is no t co rrect. 8% cho se this.

7 Choroid plexus papilloma is a rare cause of hydrocephalus from excess cerebrospinal fluid production. If our patient had this lesion, we would expect to
see an enhancing mass in the lat eral ventricle. A choroid plexus papilloma usually presents in patients < 10 years old, and therefore is not the most likely
8 cause of this patient's hydrocephalus.
Hydrocephalus Cerebrospinal fluid lateral ventricles Choroid plexus Choroid plexus papilloma Ventricular system Choroid Ventricle (heart) lesion Papilloma
9
10
c is no t co rrect. 1 0 % cho se this.
Hydranencephaly is the absence of cerebral hemispheres, which have been replaced by fluid-filled sacs lined by leptomeninges. The skull and its brain
11 cavities are normal. This patient clearly has cerebral hemispheres.
Hydranencephaly Cerebral hemisphere Meninges Human skull Human brain leptomeninges Skull
12
D is no t co rrect. 5 % cho se this.
13
Excessive ingestion of vitamin A can increa se secretion of cerebrospinal fluid and/or increa se permeability of the blood-brain barrier; lea ding to
14 hydrocephalus. However; there is no rea son to believe in this case that the patient is t aking (or being given ) excess vitamin A.
Hydrocephalus Cerebrospinal fluid Vitamin A Blood-brain barrier Vitamin Secretion
15
E is no t co rrect. 1 9 % cho se this.
16
Noncommunicating hydrocephalus is caused by obstruction of cerebrospinal fluid flow within the ventricular system . Obstruction can be secondary to
• 17 tumors, intraparenchymal hemorrhage, other masses, and congenital malformations. This patients CT was negative for these abnormalities.
Intraparenchymal hemorrhage Hydrocephalus Cerebrospinal fluid Ventricular system Bleeding Congenital disorder
• 18
• 19
• 20 Botto m Li ne:
NPH, believed to be a form of communicating hydrocephalus, presents with chronically dilat ed ventricles in elderly patients exhibiting urinary
• 21
• incontinence. oait difficulties. and mental decline .

6
lock
s
Suspend
0
End Block
Item: 16 of 41 ~ .I • M k <:] t> al ~· ~
QIO: 3773 .l. ar Previous Next Lab 'Vfll ues Notes Calculator

1 FA17 p 4 92.1
2 Hydrocephalus t CSF volume ... ventricular dilation +/- t IC P.
3 Communicating
4 Communicating l CSF absorption by arachnoid granulations (eg, arachnoid scarring post-meningitis) ... t IC P,
5 hydrocephalus papilledema, herniation.
6 Normal pressure Affects the elderly; idiopathic; CSF pressure elevated onlr episodically; does not result in increased
7 hydrocephalus subarachnoid space volume. Expansion of ventricles fJ d istorts the fib ers of the corona rad iata
-+ triad of urinary incontinence, ataxia, and cognitive dysfunction (sometimes reversible). "\Vet,
8
wobbly, and wacky." Characteristic magnetic gait (feet appear stuck to Aoor).
9
Noncommunicating (obstructive)
10
Noncommunicating Caused by structural blockage of CSF circulation within ventricular system (cg, stenosis of
11
hydrocephalus aqueduct of Sylvius; colloid cyst blocking foramen of Monro; tumor (lJ).
12
Hydrocephalus mimics
13
Ex vacuo Appearance of t CSF' on imaging [!II, but is actually due to decreased brain tissue and neuronal
14
ventriculomegaly atrophy (eg, Alzheimer disease, advanced HlV, Pick disease, Huntington disease). ICP is normal;
15 triad is not seen.
16
• 17

• 18
• 19
• 20
• 21

6
lock
s
Suspend
0
End Block
Item: 17 of 41 ~ 1 • M k -<:J 1>- Jil ~· !:';-~
QIO: 3238 ..L ar Pre v ious Next Lab~lues Notes Calcula t o r

1
2
&

A 24-year-old man presents to the emergency department because of a laceration on the palmar surface of the second digit of his left hand, between
the proximal interphalangeal and distal interphalangeal joints. The wound has stopped bleeding and appears clean. The physician decides to numb
lA• A] &

the area with lidocaine before she sutures the wound using a subcutaneous local block. The physician wishes to provide anesthesia to only the area
3 that she plans to suture.
4
5 The nerve that should be targeted by the physician also provides cutaneous Innervation to which area?

6 :
A. The dorsa l surface of the digit, near the metacarpophalangea l j oint
7
8 B. The lateral aspect of the forearm, halfway between the w ri st and the elbow

9 c. The palmar surface of the hand, just superficial to the thenar muscles
10
o. The rad ial aspect of the wrist
11
E. The ulnar aspect of the wrist
12
13
14
15
16
• 17
• 18
• 19
• 20
• 21

a
Lock
s
Suspend
8
End Bl ock
Item: 17 of 41 ~. I • M k <:] t> al ~· ~
QIO: 3238 .l. ar Previous Next lab 'Vfllues Notes Calculator

1 The co rrect a nswer is c. 62% cho se this.


This patient has injured his finger in an area that receives cutaneous innervation from the median nerve. The median nerve provides sensation to most of
2
the palm of the hand, as well as the palmar aspect of the first, second, and third digits and one-half of the fourth digit. The palmar area between the
3 proximal interphalangeal and distal interphalangeal joint of the second digit falls in this domain. Suturing the skin at this site would require local
anesthesia that t argets this cutaneous innervation ; lidocaine is best injected just past the proximal interphalangeal joint (but proximal to the injury) on
4 both sides of the digit to t arget the cutaneous branch of the median nerve in this area .
lidocaine Interphalangeal articulations of hand Median nerve local anesthesia Nerve Cutaneous innervation Distal Anatomical terms of location Anesthesia
5
6 A is no t co rrect. 1 3 % cho se this.
The dorsal surface of the digit near the met acarpophalangeal joint is supplied by the radial nerve and will not provide anesthesia to the area of injury.
7 Metacarpophalangeal joint Radial nerve Anesthesia Anatomical terms of location Dorsum (biology)
8 B is no t co rrect. 8% cho se this.
9 The lat eral aspect of the forearm is supplied by the lat eral cutaneous nerve of the forearm and will not provide anesthesia to the area of injury. This
nerve does not provide cutaneous innerva tion to the superficial hand.
10 lateral cutaneous nerve of forearm Cutaneous innervation Forearm Anesthesia Nerve

11 D is no t co rrect. 11% cho se this.


12 The radial aspect of the wrist is supplied by the radial nerve and will not provide anesthesia to the area of injury. The radial nerve mainly provides
cutaneous innervation to the dorsal hand.
13 Radial nerve Cutaneous innervation Anesthesia Nerve Dorsum (biology)
14 E is no t co rrect. 6 % cho se this.
15 The ulnar aspect of the wrist is supplied by the ulnar nerve and will not provide anesthesia to the area of injury. The ulnar nerve mainly provides
cutaneous innervation to the fourth and fifth digits of the palmar hand.
16 Ulnar nerve Cutaneous innervation Anesthesia
17
• 18
Botto m Li ne:
• 19 Sensation in the palmar side between the proximal interphalangeal and distal interphalangeal joints of the second digit is provided by the median
• 20 nerve. The region is best numbed by a lidocaine injection on both sides of the digit just past the proximal interphalangeal joint, proximal to the
damaged area .
• 21 Interphalangeal articulations of hand lidocaine Median nerve Anatomical terms of location

6
lock
s
Suspend
0
End Block
Item: 17 of 41 ~. I • M k <:] t> al ~· ~
QIO: 3238 .l. ar Previous Next Lab 'Vfll ues Notes Calculator

1 • •
FA17 p 4 27.1
2 Upper extremity nerves
3 NERVE CAUSES OF INJURY PRESENTATION

4 Axillary (CS-C6) Fractured surgical neck of humerus; anterior Flattened deltoid


dislocation of humerus Loss of arm abduction at shoulder(> 15°)
5
Loss of sensation over deltoid muscle and lateral
6 arm
7 Musculocutaneous Upper trunk compression Loss of forearm Aexion and supination
8 (CS-C7) Loss of sensation over lateral forearm
9 Radial (CS-T1) Midshaft fracture of humerus; compression of Wrist drop: loss of elbow, wrist, and fin ger
10 axilla, eg, due to crutches or sleeping with arm extension
11
over chair ("Saturday night pa lsy") l grip strength (wrist extension necessary for
maxima l action ofAexors)
12
Loss of sensation over posterior ann/forearm and
13 dorsal hand
14 Median (CS-T1) Supracondylar fracture of humerus (proximal "Ape hand" and "Pope's blessing"
15 lesion); carpal tunnel syndrome and wrist Loss of wrist Aexion, Aexion of lateral fingers,
16 laceration (distal lesion) thumb opposition, lumbricals of 2nd and 3rd
digits
17
Loss of sensation over thenar eminence and
• 18
dorsal and palmar aspects of lateral 3Y2 lingers
• 19 with proximal lesion
0 20 Ulnar (C8-T1 ) Fracture of med ial epicondyle of humerus "Ulnar claw" on digit extension
0 21 "funny bone" (proximal lesion); fractured Radial deviation of wrist upon Aexion (proximal •

6
lock
s
Suspend
0
End Block
Item: 17 of 41 ~ 1 • M k -<:J 1>- Jil ~· !:';-~
QIO: 3238 ..L ar Pre v ious Next Lab~lues Notes Calcula t o r
& &
1
Ulnar (C8-T1 ) Fracture of medial epicondyle of humerus "Ulnar claw" on digit extension
2 "funny bone" (proximal lesion); fractured Radial deviation of wrist upon Aexion (proximal
3 hook of hamate (distal lesion) from fall on lesion)
4
outstretched hand Loss of wrist Aexion, Aex ion of medial fingers,
abduction and adduction of fingers (interossei),
5
actions of medial 2 lumbrical muscles
6 Loss of sensation over medial 11h fingers
7 including hypothenar eminence
8 Recurrent branch of Superficial laceration of palm " pe hand"
9 median ne rve (CS-T1) Loss of thenar muscle group: opposition,
abduction, and Aexion of thumb
10
'o loss of sensation
11
ll umerus fractures, proximally to distally, follow the ARI\1 ( \\illary - Radial - \1edian)
12
cs
13 Ax1llary nerve ~ "(7
~ C8
14 Tl
~Median nerve
Axillary nerve~
15 Musculocutaneous nerve-----.:. '17 Ulnar nerve
~Radial nerve
_,-Intercostobrachial
16 .-" nerve
Rad1al nerve --.._: .....__Radial nerve
17 ~Medial brach1al
cutaneous nerve Palmof hand
. 18
• 19 Med1an n e r v e /' -Ulnar nerve
Musculocutaneous nerve"""'- ~Medial antebrachial
• 20 .-' cutaneous nerve Median nerve --......_.,_
Radial nerve/
• 21 ~ Ulnar nerve
• o~ ;....t~ ....

a
Lock Suspend
s 8
End Bl ock
£!1}>' !!":-~
Item:
.
17 of 41 ~ 1
• Ma rk ~
Prev10us N
ext
!>- Lab«tlues Not es Calculat or

1
cs

""'""'""~
(6
2 0
C8
n
3 ~ Median nerve
Axillary nerve ~
4 Musculocutaneous nerve------.:. Ulnar nerve --.........
~ lntercostobrachlill
5 ~Radaal nerve ,r nerve
Radaal nerve ~ .......__Radial nerve
6 _,.... Medial brachaal
.-r cutaneous nerve Patmothand
7
8 Medaan nerve / ' - Ulnar nerve ~Med1al antebrachial
Musculocutaneous nerve -.......
9
r cutaneous nerve Median nerve --.......___
Rad1al nerve /
10 ~ Ulnar nerve
Recurrent branch............_ Radial nerve --.......___
11 of med1an nerve Radial nerve --.....

12 Dorsum of hand
13
14
15 FA17 p428.1
16 Brach ial plexus lesions
17 0 Erb palsy ('waiter's t1p') cs Lateral Randy
. 18 E) Klumpke palsy {claw hand) 0 Musculocutaneous Tra,·is
€) Wrist drop (6 D rinks
• 19 Axillary
0 Winged scapula Middle Postern 0 Cold
• 20 0 Deltoid paralysis C7 €) (Elrt~nson) 0 Median (flexors)
(;) 'Saturday night palsy' (wrist drop) (;) Beer
21
.... llatli.al

a
Lock Suspen d End Block
Item: 17 of 41
QIO: 3238
~
..L
1 • Ma r k -<:J
Prev ious
I>
Next Lab
fJ lues
£!1}>'

Notes
!!":-~
Cal culat o r

& &
1 FA17 p428.1
2
Brachial plexus lesio ns
3 0 Erb palsy ("walter's up') C5 Lateral Randy
4 a Klumpke palsy (tlaw hand) 0 Musculocutaneous Tra,·is
€) Wrist drop (6 D rinks
5 Axit~ry
0 Winged scapula Middle Postem 0 Cold
6 0 Delt04d parai}'Sis a €) (;)IUI.ensonl e Median (flexors) Beer
7 (;) 'Saturday mght palsy' (wrist drop)

8
0 Difficulty lieXJng elbow. vanable C8 Rac!YI
sensory loss lower Medial
9 e Decreased thumb function. a -..--11 I I
(;) Ulnar
'Pope's blesSing' T1 I
I
-I I
10 Trunks DiviSions Cords Branches
(;) lntnnSic muscles of hand, 0
11 claw hand
Long thoracic
12
I
13 Roots
14
CONDITION INJURY CAUSES MUSCLE DEFICIT FUNCTIONAL DEFICIT PRESENTATION
15 Erb palsy ("waite r's Traction or Deltoid, Abduction (arm
Infants-lateral
16 t ip") tear of upper traction on neck supraspinatus hangs by side)
17 {"Erb-er") trunk: during deli,•ery In fraspinalus Lateral rotation (arm
. 18 C5-C6 roots Adults-trauma medially rotated)
• 19 Biceps brachii Flexion, supination
• 20 (arm extended and
• 21 pronated)

a
Lock
s
Suspend
8
End Bl ock
a
Lock
s
Suspend
8
End Block
Item: 18 of 41 ~ 1 • M k -<:J 1>- Jil ~· !:';-~
QIO: 2948 ..L ar Pre v ious Next Lab~lues Notes Calcula t o r
&
1 A 70-year-old woman is having difficulty performing simple calculations that she previously was able to perform. She is mildly frustrated because she
has been Increasingly forgetful of where she places her belongings. Autopsy findings from the brain of another patient who suffered from the same
2 condition are shown in the image.
3
4
5
6
7
8
9
10
11

12
13
14
15 I mage courtesy of Wikimedia Commons

16
What Is the mechanism of action of the best trea tment?
17
. 18
A . Agonism at GABA receptors
• 19
B. Antagonism at !}-adrenergic receptors
• 20
• 21 C. Conversion into active neurotransmitter

a
Lock
s
Suspend
8
End Bl ock
Image courtesy of Wikimedia Commons
10

11 What Is the mechanism of action of the best treatment?

12 :
A. Agonlsm at GABA receptors
13
14 8. Antagonism at 13-adrenergic receptors
15
c. Conversion into active neurotransmitter
16
D. Inhibition of acetylcholinesterase
17
. 18 E. Inhibition of dopaminergic activity

• 19 F. Inhibition of neurotransmitter reuptake


• 20
G. Interaction with cation transport
• 21

a
Lock
s
Suspend
8
End Block
Item: 18 of 41 ~ 1 • M k -<:J 1>- Jil ~· !:';-~
QIO: 2948 ..L ar Previous Next Lab~lues Notes Calculat or
& &
1
2
3 Th e correct a n sw er i s D. 59% chose this.
The Image shows neurofibrillary tangles (black arrow; vanishing nucleus In red circle)
4
characteristic of Alzheimer disease (AD). AD is the most common cause of dementia in the Image courtesy of Wikimedia Commons
5 elderly. Pathophysiologically, this disease is associated with extracellular deposition of
neuritic plaques (abnormally deaved amyloid protein) and intracellular neurofibrillary tangles (hyperphosphorylated T-proteins) in the cerebral cortex. The
6 plaques and tangles damage neurons over time, resulting in progressive memory loss and cognitive impairment. Although no cure for AD exists, donepezli
(a cholinesterase inhibitor/ indirect cholinomimetic) improves cognitive measures by upregulating acetylcholine activity. Donepezil, however, does not alter
7 the course of the disease.
8 Donepez• Acetylcho ne Cerebral cortex Amyloid Cholinesterase Neurofibri .• ary tangle Dementia Alzheimer's disease Parasympathomimetic drug Neuron
Sen•le plaques Phosphorylation Cognitive deficit Protein Intracellular Hyperphosphorylation Amnesia Cognition
9
A i s n ot co rrect. 8 % chose this.
10
Lorazepam, a benzodiazepine, is used to treat status epilepticus and acute anxiety (eg, panic attacks).
11 Benzodiazepine Lorazepam Status epilepticus Panic attack Anxiety
12 B is not co rrect. 5 0/o chose t his.
13 A 13-blocker such as atenolol could be effective in controlling high blood pressure that may cause multi -infarct dementia; however, the image Is not
consistent with multi-infarct dementia. The brain of a patient with multi-Infarct dementia would show small lacunar infarcts scattered in the brain
14 parenchyma and the brain stem.
Vascular dementia Brainstem Parenchyma Dementia Hypertension Blood pressure Lacunar stroke Human brain Brain Stroke
15
C is not co rrect . go;o ch ose this.
16
Combination therapy of levodopa/carbidopa is used to treat Parkinson's disease. Parkinson disease is associat ed with Lewy bodies, which are eosinophilic
17 Intracytoplasmic Inclusions in the dam aged cells of the substantia nigra . However, this finding is not specific. Parkinson disea se is marked primarily by four
cardinal signs: resting tremor, cogwheel rigidity, bradykinesia, and postural Instability.
18
Parkinson's disease Substantia nigra Bradykinesia Carbidopa/levodopa Lewy body Balance disorder Tremor
. 19
E i s n ot correct. 7 % ch ose this .
• 20 Haloperidol, an antipsychotic agent, often is used to treat schizophrenia. Schizophrenia is a disorder commonly m arked by psychosis, and is not associated
with histopathologic changes in the brain. There is no indication that this patient is having psychotic symptoms.
. 21
• Halope• dol Schizophrenia Antipsychotic Psychosis Histopatholoqy Human brain Brain

a
Lock
s
Suspend
8
End Bl ock
Item: 18 of 41 ~. I • M k <:] t> al ~· ~
QIO: 2948 .l. ar Previous Next lab 'Vfl1 ues Notes Calculator

1 c is no t co rrect. 9 % cho se this.


Combination therapy of levodopa/carbidopa is used to trea t Parkinson's disea se. Parkinson disea se is associat ed with Lewy bodies, which are eosinophilic
2 intracytoplasmic inclusions in the damaged cells of the substantia nigra . However; this finding is not specific. Parkinson disea se is marked primarily by four
3 cardinal signs: resting tremor; cogwheel rigidity, bradykinesia, and postural instability.
Parkinson' s disease Substantia nigra Bradykinesia Carbidopa/levodopa lewy body Balance disorder Tremor
4
E is no t co rrect. 7 % cho se this.
5 Haloperidol, an antipsychotic agent, often is used to trea t schizophrenia. Schizophrenia is a disorder commonly marked by psychosis, and is not associat ed
6 with histopathologic changes in the brain. There is no indication that this patient is having psychotic symptoms.
Haloperidol Schizophrenia Antipsychotic Psychosis Histopathology Human brain Brain
7
F is no t co rrect. 9 % cho se this.
8 Fluoxetine, a selective serotonin reuptake inhibitor; is an effective antidepressant, particularly in the elderly. Depression can be confused with dementia
9 (called pseudodementia); however; depression is a mood disorder that is not associat ed with histopathologic changes in the brain. Clinically, patients can
experience anhedonia, sleep disturbances, decrea sed energy, diminished ability to concentrat e, and feelings of guilt or worthlessness.
10 Selective serotonin reuptake inhibitor Mood disorder Fluoxetine Anhedonia Serotonin Antidepressant Serotonin reuptake inhibitor Reuptake Dementia

11 Sleep disorder Major depressive disorder Depression (mood) Reuptake inhibitor Human brain Enzyme inhibitor Brain Histopathology

12 G is no t co rrect. 3 % cho se this.


Lithium is commonly prescribed for patients with bipolar disorder. This is a manic mood disorder with or without alternating symptoms of depression and
13 psychosis. Although comorbidities can occur; bipolar disorder itself does not exhibit histopathologic changes in the brain.
14 Bipolar disorder Mood disorder Psychosis Histopathology Comorbidity Major depressive disorder lithium Depression (mood) Human brain Brain

15
16 Botto m Li ne:
17 Donepezil upregulat es acetylcholine activity to provide symptomatic relief in patients with AD.
Donepezil Acetylcholine
18
• 19
• 20
I ill ;fi 1!1 I•J f o r yea r:[ 20 17 ..
• 21 FI RST AI D FA CTS

6
lock
s
Suspend
0
End Block
Item: 18 of 41 ~. I • M k <:] t> al ~· ~
QIO: 2948 .l. ar Previous Next lab 'Vfl1 ues Notes Calculator

1 • •
FA17 p 518.5
2 Alzheimer disease drugs
3
Memantine
4
MECHANISM NMDA receptor antagonist; helps prevent excitotoxicity (mediated by Ca 2+).
5
ADVERSE EFFECTS Dizziness, con fusion, hallucinations.
6
Donepezil, galantamine, rivastigmine, tacrine
7
MECHANISM AChE inhibitors.
8
9
ADVERSE EFFECTS Nausea, dizziness, insomn ia.

10
FA17 p490.1
11

12
Neurodegenerative l in cognitive ability, memory, or function with intact consciousness.
disorders
13
DISEASE DESCRIPTION HISTOLOGIC/GROSS FINDINGS
14
Parkinson disease Parkinson TRAPS your body: Loss of dopam inergic neurons (ie,
15 Tremor (pill-rolling tremor at rest) depigmentation) of substantia nigra pars
16 Rigidity (cogwheel) compacta.
17 Akinesia (or bradykinesia) Lewy bodies: composed of a -synuclein
18 Postural instability (intracellular eosi noph ilic inclusions t:i.l).
ShufAing gait
• 19
MPTP, a contaminant in illegal drugs, is
20
0
metabolized to lPP+, which can cause
• 21 parkinsonian symptoms. •

6
lock
s
Suspend
0
End Block
Item: 18 of 41 ~ 1 • M k -<:J 1>- Jil ~· !:';-~
QIO: 2948 ..L ar Pre v ious Next Lab~lues Notes Calcula t o r
& &
1 Huntington disease Autosomal dominant trinucleotide (C C) 0 Atrophy of caudate and putamen with ex vacuo
2 repeat disorder on chromosome 4. Symptoms ventriculomegaly.
3 manifest between ages 20 and 50: chorea, t dopamine, l CABA, l ACh in brain. 1 euronal
4
athetosis, aggression, depression, dementia death 'ia 1 MDA-R binding and glutamate
(sometimes initially mistaken for substance excitotoxicity.
5
abuse).
6 Anticipation results from expansion of C. \ G
7 repeats. Caudate loses \ Chand GABA.
8 Alzheimer disease ~lost common cause of dementia in elclerl). Widespread cortical atrophy (normal cortex
9 Down syndrome patients ha,·e t risk of cortex in lzheimer disease ), especially
10 developing Alzheimer disease, as APP is hippocampus (arrows in 11) and ® . arrO\\ mg
located on chromosome 21. of gyri and widening of sulci.
11
Associated with the following altered proteins: Senile plaques (!] in gray matter: extracellular
12
poE2: l risk of sporad ic form ~-amyloid core; may cause amyloid angiopathy
13 • ApoE4: t risk of sporadic form - intracranial hemorrhage; A~ (amyloid-~)
14 • APP, presenil in-1, presenilin-2: familial synthesized by cleaving amyloid precursor
15 forms (10%) with earlier onset protein (APP).
l ACh eurofibrillary tangles 0 : intracellular,
16
hyperphosphorylated tau protein = insoluble
17 cytoskeletal elements; number of tangles
18 correlates with degree with dementia.
• 19 Frontotemporal Early changes in personality and behavior Frontotemporal lobe degeneration
• 20 dementia (Pick (behavioral variant), or aphasia (primary inclusions of hyperphosphorylaled tau (round
disease) progressi,·e aphasia). Pick bodies (!!) or ubiquitinated T DP-·B.
• 21
• . . . . ..
a
Lock
s
Suspend
8
End Bl ock
Item: 18 of 41 ~ 1 • M k -<:J 1>- Jil ~· !:';-~
QIO: 2948 ..L ar Pre v ious Next Lab~lues Notes Calcula t o r

F'rontotemporallobe degeneration D.
& &
1 Frontotemporal Early changes in personality and behavior
2 dementia (Pick (behavioral variant), or aphasia (primary Inclusions of hyperphosphorylated tau (round
3 disease) progressi\·e aphasia). Pick bodies (!I) or ubiquitinated TDP-·B.
lay ha\·e associated mo\·ement disorders
4
(eg, parkinsonism, ALS-Iike Ul\ 1 '/L I '
5
degeneration).
6
l ewy body dementia Dementia and\ isual hallucinations Intracellular Lew~ bodies f'.J primaril~ in cortex.
7 ("ha Lew~ cinations") - parkinsonian features
8 Vascular dementia Result of multiple arterial infarcts and/or ~ I RI
or CT shows multiple cortical and/or
9 chronic ischemia. subcortical infarcts.
10 Step-\\ise decline in cognitive ability with late-
11
onset memory impairment. 2nd most common
cause of dementia in elderly.
12
Creutzfeldt-Jakob Rapidly progressive (weeks to months) dementia Spongiform cortex.
13
disease with myoclonus ("startle myoclonus"). Prions (Prpc - Prpsc sheet [~-pl ea ted sheet
14
Commonly see periodic sharp waves on F:F:G resistant to proteases]) C).
15 and f 14-3-3 protein in CSF.
16
17
18
• 19
• 20
• 21

a
Lock
s
Suspend
8
End Bl ock
Item: 18 of 41 ~ 1 • M k -<:J 1>- Jil ~· !:';-~
QIO: 2948 ..L ar Pre v ious Next Lab~lues Notes Calcula t o r

1
&
onset memory impairment. 2nd most common &

2 cause of dementia in elderly.


3 Creutzfeldt-Jakob Rapidly progressive (weeks to months) dementia Spongiform cortex.
4
disease with myoclonus ('"startle myoclonus"). Prions (Prpc - Prrsc sheet [13-pleated sheet
Commonly see periodic sharp ,,.a,·es on EEC resistant to proteases)) Cl
5
and t 14-3-3 protein in CSF.
6
7
8
9
10
11

12
13
14
15
16
17
18
• 19
• 20
• 21

a
Lock
s
Suspend
8
End Bl ock
Item: 19 of 41
QIO: 2816
~
..L
1 • Ma r k -<:J
Prev ious
I>
Next Lab
fJ
lues
£!1}>'

Notes
!!":-~
Cal culat o r

& &
1
A researcher develops a neurophysin I mouse knockout model.
2
3 Which of the following processes will be impaired in these animals?
4
:
5 A. Milk secretion

6 B. Milk synthesis
7
c. Ovulation
8
9
o. Salt retention

10 E. Spermatogenesis

11
12
13
14
15
16
17
18
. 19
• 20
. 21

a
Lock
s
Suspend
8
End Bl ock
Item: 19 of 41 ~. I • M k <:] t> al ~· ~
QIO: 2816 .l. ar Previous Next lab 'Vfl1ues Notes Calculator

1 The co rrect a nswer is A. 56 % cho se this.


AD H (vasopressin) and oxytocin are synthesized by the neurons of the supraoptic and para ventricular nuclei in the hypothalamus, respectively. These
2
hormones are transported to the posterior pituitary gland on direct axonal connections called the supra optic hypophyseal tract, where they are stored and
3 eventually relea sed into the capillaries draining into the hypophyseal vein. The transport of ADH and oxytocin from the hypothalamus requires carrier
proteins: neurophysin I is the transporter protein for oxytocin, wherea s neurophysin II is the transporter protein for ADH . Oxytocin facilitat es milk
4 secretion but not milk synthesis and also stimulat es uterine contractions during labor. ADH mediat es wat er absorption at the renal collecting ducts via V2 -
receptors via insertion of aquaporins, thus concentrating the urine. In contra st to the posterior pituitary, the anterior pituitary is the hypothalamic-
5 hypophyseal portal system, a capillary system that t ra nsports relea sing hormones synthesized in the hypothalamus that act on the anterior pituitary.
6 Neurophysin I Oxytocin Paraventricular nucleus of hypothalamus Hypothalamus Posterior pituitary Collecting duct system Pituitary gland Axon Aquaporin Capillary
Cell nucleus Vasopressin Supraoptic nucleus Neuron Urine Hormone Protein Anterior pituitary Nucleus (neuroanatomy) Gland Carrier protein Secretion Uterus
7
B is no t co rrect. 18% cho se this.
8
Milk synthesis is mediat ed by prolactin, which is secret ed by the anterior pituitary gland and hence would be unaffected by loss of neurophysin I. Although
9 oxytocin does not have a role in milk synthesis, it allows milk letdown in lactating women.
Oxytocin Prolactin Pituitary gland Anterior pituitary Breastfeeding lactation Gland Anatomical terms of location Milk
10
11
c is no t co rrect. 7% cho se this.
Ovulation is stimulat ed by the surge of luteinizing hormone ( LH ) just before the midpoint of the menstrual cycle. LH is secret ed by the anterior pituitary
12 gland and hence would be unaffected by loss of a carrier protein because anterior pituitary hormone relea se is mediat ed by hypothalamic signals in the
blood.
13 luteinizing hormone Menstrual cycle Pituitary gland Anterior pituitary Ovulation Protein Hormone Hypothalamus
14
D is no t co rrect. 15 % cho se this.
15 Salt ret ention is a primary action of aldosterone, acting at the renal distal tubules to increa se sodium and chloride reabsorption, as well as increa se
potassium and hydrogen secretion . Aldosterone is produced from cholesterol in a multistep pathway in response to ACTH stimulation . Serum potassium
16 and activation of the renin-angiotensin-aldosterone system ( RAAS ) are the primary regulators of aldosterone relea se. These processes would not be
17 affected by lack of neurophysin.
Aldosterone Adrenocorticotropic hormone Renin-angiotensin system Potassium Sodium Cholesterol Distal convoluted tubule Blood plasma Salt
18
Anatomical terms of location Salt (chemistry) Hydrogen Secretion Serum (blood) Kidney Chloride
19
E is no t co rrect. 4 % cho se this.
0 20 Spermatogenesis is stimulat ed by follicle-stimulating hormone, which is secret ed by the anterior pituitary gland and hence would be unaffected by
neurophysin loss.
0
21 Follicle-stimulatina hormone Soermatoaenesis Pituitary aland Anterior oituitarv Hormone Gland

6
lock
s
Suspend
0
End Block
Item: 19 of 41 ~. I • M k <:] t> al ~· ~
QIO: 2816 .l. ar Previous Next lab 'Vfl1ues Notes Calculator

1
Bottom Line:
2
ADH and oxytocin are synthesized by the hypothalamus and transported to the posterior pituitary via a direct neuronal connection . Neurophysin I is the
3 carrier protein required for oxytocin, and neurophysin II is the carrier protein for ADH .
Neurophysin I Oxytocin Hypothalamus Posterior pituitary Vasopressin Neurophysin II Pituitary gland Protein Carrier protein Neurophysins
4
5
6
I iii I;fi 1!1 I•J for year:l 2017 ..
7 FIRST AID FAC T S

8
FA17 p 313.1
9
Pituitary gland
10
Anterior pituitary Secretes FSTI, LH, ACTH, TSH, prolactin, ACTH and ISH are derivatives of
11
(adenohypophysis) C H. Melanotropin (MSII ) secreted from proopiomelanocorti n (POMC).
12 intermediate lobe of pituitary. Derived from FLAT PiG: FSH, LH, ACTH, T SH, PRL, G H.
13 oral ectoderm (Rathke pouch). B-FLAT: Basophils- FSI I, Ll l, ACTII, TSI I.
14 a subunit-hormone subunit common to Acidophils: C ll, PRL.
15
TSH, LH, FSH, and hCC.
• ~ subunit-determines hormone specificity.
16
Posterior pituitary Stores and releases vasopressin (antidiuretic
17
(neurohypophysis) hormone, or ADH) and oxytocin, both
18
made in the h)'pothalamus (supraoptic and
19 paraventricular nuclei) and transported to
0 20 posterior pituitary via neurophysins (carrier
0
21 proteins). Derived from neuroectoderm.

6
lock
s
Suspend
0
End Block
Item: 19 of 41 ~ 1 • M k -<:J 1>- Jil ~· !:';-~
QIO: 2816 ..L ar Pre v ious Next Lab~lues Notes Calcula t o r
& &
1
2 FA17 p 317.3
3 Antidiuretic hormone
4 SOURCE Synthesized in hypothalamus (supraoptic
5 nuclei), stored and secreted by posterior
pituitary.
6
7
FUNCTION Regulates serum osmolaritr (Vz-receptors) AOIJ bel is l in central diabetes insipidus (01 ),
and blood pressure (\1 1-receptors). Primary normal or t in nephrogenic DI.
8
function is serum osmolarit) regulation (J\DII 1'\cphrogcnic 0 1can be caused by mutation in
9 l serum osmolarity, t urine osmolarit)") 'ia v z-receptor.
10 regulation of aquaporin channel insertion in Ocsmopressin acetate (AOH analog) is a
11 principal cells of renal collecting duel. treatment for central 01 and nocturnal
12
enures1s.
13 REGULATION Osmoreceplors in hypotha lamus (1°);
hypovolemia.
14
15
16 FA17 p 466.2

17 Hypothalamus Mainta ins homeostasis by regulat ing T hirst and water balance, controlling Adenohyophysis
18 (anterior pituitary) and Neurohypophysis (posterior pituitary) release of hormones produced in
the hyopthalamus, and regulating I Iunger, \ utonomic nervous system, Temperature, and Sexual
19
urges (TAN I L\TS).
• 20
Inputs (areas not protected by blood-brain barrier}: OVt:r (senses change in osmolarity), area
. 21 postrema (found in medulla, responds to emetics).

a
Lock Suspend
s 8
End Bl ock
Item: 19 of 41 ~ 1 • M k -<:J 1>- Jil ~· !:';-~
QIO: 2816 ..L ar Pre v ious Next Lab~lues Notes Calcula t o r
& &
1 FA17 p 466.2
2 Hypothalamus Maintains homeostasis by regulating T hirst and water balance, controlling Adenohyophysis
3 (anterior pituitary) and 1\euroh) pophysis (posterior pituitary) release of hormones produced in
4 the hyopthalamus, and regulating I Iunger, \ utonomic nervous system, Temperature, and Sexual
5 urges (TAN I lSI'S).
Inputs (areas not protected by blood-brain barrier): 0 I..:f (senses change in osmolarity), area
6
postrema (found in medulla, responds to emetics).
7
Lateral area Hunger. Destruction ..... anore,ia, failure If you zap your lateral area, you shrink laterally.
8
to thri,·e (infants). Stimulated b) ghrelin,
9 inhibited by lcptin.
10 Ventromedial area If you zap your \"entromedial area, you grow
Satiety. Destruction (eg, craniopharyngioma) .....
11 hyperphagia. Stimulated by lcpti n. \Cntrally and medially.
12 Anterior Cooling, parasympathetic. Anterior nucleus = cool off (cooling,
13 hypothalamus pArasympathetic). A/C = anterior cooling.
14 Posterior Heating, sympathetic. Posterior nucleus = get fi red up (heating,
15 hypothalamus sympathetic). If you zap your posterior
16
hypothalamus, you become a poikilotherm
(cold-blooded, like a snake).
17
Su prachiasmatic Circadian rhythm. You need sleep to be charismatic (chiasmatic).
18
nucleus
19
Supraoptic and Synthesize ADH and oxytocin ADH and oxytocin are carried by neurophysins
• 20
paraventricular do" n axons to posterior pituitary, where these
• 21 nuclei hormones are stored and released.

a
Lock
s
Suspend
8
End Bl ock
Item: 20 of 41 ~ 1 • M k -<:J 1>- Jil ~· !:';-~
QIO: 2945 ..L ar Pre v ious Next Lab~lues Not es Calcula t o r

1
2
&

An afebrile 3-week-old infant is noted to have a bulging anterior fontanel on physical examination. His mother indicat es that in the past few days, he
has been feeding poorly and that he appea rs more irritable. His birth history Is remarkable for an episode of Group B streptococcal meningitis that
lA• A] &

resolved after a full course of high-dose intravenous ampicillin.


3
4 Which of the following mechanisms is most likely responsible for this patient's signs and symptoms?
5
:
6 A. Accumulation of blood in the subarachnoid space
7
B. Decreased absorption of cerebrospinal fluid by the arachnoid villi
8
C. I ncreased production of cerebrospinal fluid
9
10 D. Recurrence of incompletely treated meningitis

11 E. Ventricular obstruction
12
13
14
15
16
17
18
19
• 20
. 21

a
Lock
s
Suspend
8
End Bl ock
Item: 20 of 41 ~. I • M k <:] t> al ~· ~
QIO: 2945 .l. ar Previous Next lab 'Vfl1 ues Notes Calculator

1 The co rrect a nswer is B. 55 % cho se this.


2 Group B streptococci is one of the lea ding causes of newborn meningitis. After an episode of meningitis has resolved, meningeal scarring can occur as a
complication . The scarring can be severe enough to decrea se cerebrospinal fluid (CSF) absorption by arachnoid villi, resulting in a communicating
3 (nonobstructive) hydrocephalus. "Communicating" indicat es that the opening between the ventricles and the subarachnoid space is maintained.
Overproduction or underabsorption of CSF causes this condition. CT of the hea d would show dilat ation of the lat eral and third ventricles with reduction of
4 the extra -axial spaces. These findings suggest accumulation of CSF in the brain, lea ding to expansion of the cranial cavity and an increa se in intracranial
5 pressure; this causes the bulging anterior fontanels in newborns. This patient also likely has an increa sed hea d circumference. The common trea tment is a
shunt, but if left untrea t ed, hydrocephalus can potentially lea d to impairment of mental and muscular functions or even dea th .
6 Hydrocephalus Subarachnoid space Cerebrospinal fluid Arachnoid granulation Meningitis Intracranial pressure Cranial cavity Ventricular system Arachnoid mater

7 Fontanelle Streptococcus agalactiae Streptococcus Shunt (medical) Meninges Human brain Ventricle (heart)

8 A is no t co rrect. 5 % cho se this.


Accumulation of blood in the subarachnoid space is usually due to trauma or a ruptured berry aneurysm and is not usually a complication of meningitis.
9
Subarachnoid space Meningitis Aneurysm
10
c is no t co rrect. 8% cho se this.
11 Ra rely, communicating (nonobstructive) hydrocephalus can result from increa sed production of CSF by a choroid plexus tumor. This mechanism is not a
complication of meningitis.
12
Choroid plexus tumor Hydrocephalus Choroid plexus Meningitis Neoplasm Choroid
13
D is no t co rrect. 13% cho se this.
14 Meningitis is associat ed with increa sed intracranial pressure, which does in fact cause bulging of anterior fontanels and nonspecific symptoms in infants.
This answer choice should be on your differential diagnosis. However; the lack of fever and recently resolved meningitis makes communicating
15 (nonobstructive) hydrocephalus more likely in this case.
16 Hydrocephalus Intracranial pressure Meningitis Nonspecific symptoms Differential diagnosis Fontanelle Fever Anatomical terms of location

17 E is no t co rrect. 19% cho se this.


Meningeal scarring may lea d to narrowing of the passageways between the ventricles, but this would rarely be significant enough to cause
18 noncommunicating (obstructive) hydrocephalus. Tuberculosis meningitis may be a possible cause. However; noncommunicating (obstructive) hydrocephalus
19 is much more commonly caused by aqueductal stenosis due to subarachnoid hemorrhage or tumors.
Subarachnoid hemorrhage Hydrocephalus Meningitis Bleeding Tuberculosis Ventricular system Aqueductal stenosis Stenosis Ventricle (heart) Cerebral aqueduct
20
Meninges Neoplasm
• 21

6
lock
s
Suspend
0
End Block
Item: 20 of 41 ~. I • M k <:] t> al ~· ~
QIO: 2945 .l. ar Previous Next lab 'Vfl1 ues Notes Calculator

1
Bottom Line:
2 Postmeningitic scarring is a common cause of communicating (nonobstructive) hydrocephalus.
3 Hydrocephalus

4
5
6
I iii I;fi 1!1 I•J for year:l 2017 ..
FIRST AID FAC T S

8 FA17 p 4 92.1
9 Hydrocephalus t CSF volume ..... ventricular dilation+/- t IC P.
10 Communicating
11 Communicating l CSF absorption by arachnoid granulations (eg, arachnoid scarring post-men ingitis) ..... t ICP,
12 hydrocephalus papilledema, herniation.
13 Normal pressure Affects the elderly; id iopathic; CSF pressure elevated onlr episodically; does not result in increased
14
hydrocephalus subarachnoid space volume. Expansion of ventricles rJ distorts the fibers of the corona rad iata
..... triad of urinary incontinence, ataxia, and cognitive dysfunction (sometimes reversible). "\Vet,
15
wobbly, and wacky." Characteristic magnetic gait (feet appear stuck to Aoor).
16
Noncommunicating (obstructive)
17
Noncommunicating Caused by structural blockage ofCSF circulation within ventricular system (eg, stenosis of
18
hydrocephalus aqueduct of Sylvius; colloid cyst blocking foramen of Monro; tumor [lJ).
19
Hydrocephalus mimics
20
Ex vacuo Appearance of t CSP on imaging B. but is actually due to decreased brain tissue and neuronal
• 21
ventriculomeaalv atroohv (ef!. Alzheimer disease. advanced H!V. Pick disease. Huntington disease). JCP is normal:
6
lock
s
Suspend
0
End Block
Item: 20 of 41 ~. I • M k <:] t> al ~· ~
QIO: 2945 .l. ar Previous Next lab 'Vfl1 ues Notes Calculator

1
Hydrocephalus mimics
2
Ex vacuo Appearance of t CSF' on imaging [!II, but is actually due to decreased brain tissue and neuronal
3 atrophy (eg, Alzheimer disease, advanced HlV, Pick disease, Huntington disease). ICP is normal;
ventriculomegaly
4 triad is not seen.
5
6
7

8
9
10
11

12
13
14
15
FA17 p 176.2
16
CSF findings in meningitis
17 OPENING PRESSURE CEll TYPE PROTEIN GlUCOSE
18 Bacterial t t PMNs t
19 Fungai!TB f t lymphocytes t
20
Viral 'ormal/t t lymphocytes NormalIt Normal
• 21

6
lock
s
Suspend
0
End Block
Item: 20 of 41 ~. I • M k <:] t> al ~· ~
QIO: 2945 .l. ar Previous Next lab 'Vfl1 ues Notes Calculator

1 •

2 FA17 p 176.2
3 CSF findings in meningitis
4 OPENING PRESSURE CEll TYPE PROTEIN GlUCOSE

5 Bacterial t t PMNs t
6 Fungai/TB t t lymphocytes t
7 Viral ormal/t t lymphocytes ormal/t Normal
8
9 FA17 p 176.1
10 Common causes of meningitis
11 NEWBORN (0- 6 MO) CHILDREN (6 M0- 6 YR) 6- 60YR 60YR+

12 Croup B streptococci S pneumoniae S pneumoniae S pneumoniae


13 E coli l meningitidis N meningitidis (#l in teens) Cram 8 rods
Listeria H influenzae type B Enterovi ruses Listeria
14
Enteroviruses HSV
15
Give ceftriaxone and vancomycin empirically (add ampicillin if Listeria is suspected).
16
Viral causes of meningitis: enteroviruses (especially coxsackievirus), HSV-2 (llSV-1 =encephalitis), lllV, West ile virus (also
17 causes encephalitis), VZV.
18 In HIV: Cryptococcus spp.
19 Note: Incidence of H influenzae meningitis has ! greatly due to conjugate H influenzae vaccinations. Today, cases are usually
20
seen in unimmunized children.
• 21

6
lock
s
Suspend
0
End Block
Item: 21 of 41 ~ 1 • M k -<:J 1>- Jil ~· !:';-~
QIO: 3777 ..L ar Pre v ious Next Lab~lues Notes Calcula t o r

1 &

A 26-year-old man presents with left eye pain and intermittent double vision. When at rest, his left eye is deviat ed downward and laterally, as
pictured In the image. Upgaze and adduction are limited in the affected eye; however, abduction appears intact. Also, the left lid droops, and the left
IA•A] &

2
pupil Is dilated and unresponsive to light as shown in the image.
3
4
5
6
7
8
9
10
11
12 Which of the following cranial nerves is most likely to have been injured?
13 :
14 A. Abducens nerve

15 B. Oculomotor nerve
16
c. Optic nerve
17
D. Trigeminal nerve
18
19 E. Trochlear nerve

20
. 21

a
Lock
s
Suspend
8
End Bl ock
Item: 21 of 41 ~ 1 • M k -<:J 1>- Jil ~· !:';-~
QIO: 3777 ..L ar Previous Next Lab~lues Notes Calculat or
& &
1 Th e co rrect a nsw er i s B. 830/o chose this.
2 This patient has oculomotor palsy from disruption of cranial nerve (CN) III. This nerve innervates the medial rectus, the superior rectus, the Inferior
rectus, and the inferior oblique muscles of the eye. When damaged, the extraocular muscles not innervated by CN III dominate (ie, the superior oblique,
3 which depresses and intorts the eye, and the lateral rectus, which abducts the eye). Hence, the eye appears "down and out." CN III also innervates the
levator palpebrae superioris, which causes ptosis. The pupil is dilated because of Involvement of the parasympathetic fibers that run on the outside of the
4
oculomotor nerve and can be compressed by structures such as tumors and aneurysms. If it is a pupil-sparing third nerve palsy, then infarction of the
5 nerve Is commonly the cause.
Oculomoto oerve EJ traocular muscles Lateral rectus muscle Ptosis ,eyehdJ Cranial nerves lnfenor rectus muscle Oculomotor nerve palsy Superior rectus must. e
6
Parasympathetic nervous system Levator palpebrae superioris muscle Medial rectus muscle Superior oblique muscle Inferior oblique muscle Skull Aneurysm
7 Neoplasm
8 A i s not correct. 5 % chose this.
9 The abducens nerve (CN VI) inn ervates the latera l rectus, which abducts the eye. Abduction is intact in this patient's affected eye.
Abducens nerve Lateral rectus muscle Abduction {kinesiology)
10
C is n ot co rrect. 3% ch ose this.
11
The optic nerve does not innervate any of the extraocular muscles and therefore cannot account for the findings above.
12 Extraocular muscles Optic nerve Nerve

13 0 is not co rrect. 20/o chose this.

14 The trigeminal nerve does not innervate any of the extraocular muscles and therefore cannot account for the findings above.
Extraocular muscles Trigeminal nerve Nerve
15
E is no t co rrect. 70fo ch ose this.
16 The trochlear nerve innervates the superior oblique muscle, which depresses and lntorts the eye. In trochlear nerve palsy, the affected eye Is elevated
17 and extorted, and the patient often tilts the hea d away from the affected eye to compensate . These abnormalities were not found here.
Trochlear nerve Superior oblique muscle Femur Muscle
18
19
Bottom Line:
20
Injury to the oculomotor crania l nerve leads to the eye appearing in a "down and out" position.
21 Cranoal nerves Skul Oculomotor nerve

a
Lock
s
Suspend
8
End Block
Item: 21 of 41 ~. I • M k <:] t> al ~· ~
QIO: 3777 .l. ar Previous Next Lab 'Vfll ues Notes Calculator

1 • FA17 p 4 76.2 •

2 Cranial nerves
3
NERVE CN FUNCTION TYPE MNEMONIC

4
Olfactory I Smell (only CN without thalamic relay to cortex) Sensory Some

5 Optic II Sight Sensory Say


6 Oculomotor Ill Eye movement (SR, IR, MR, 10), pupi llary constriction J\ fotor Marry
(sphincter pupillae: Edinger-Westphal nucleus, musca rinic
7
receptors), accommodation, eyelid opening (levator palpebrae)
8
Trochlear IV Eye mo,·cment (SO) 1otor l\ [oney
9
Trigeminal v Mastication, facial sensation (ophthalmic, maxil lary, mandibular Both But
10
divisions), somatosensation from anterior 2/ l of tongue
11
Abducens VI Eye movement (LR) Motor l\ly
12
Facial VII Facial movement, taste from anterior 2h of tongue, lacrimation, Both Brother
13 sal ivation (submandibular and sublingual glands), eyelid closing
14 (orbicularis oculi}, auditory volume modulation (stapedius)
15 Vestibulocochlear VIII Hearing, balance Sensory Says
16 Glossopharyngeal IX Taste and sensation from posterior 113 of tongue, swallowing, Both Big
17 salivation (parotid gland), monitoring carotid body and sinus
18 chemo- and baroreceptors, and elevation of pharynx/larynx
(stylopharyngeus)
19
20
Vagus X Taste from supraglottic region, swallowing, soft palate elevation, Both Brains
midline uvula, talking, cough reflex, parasympathetics to
21 thoracoabdominal viscera. mon itoring aortic arch chemo- and •

6
lock
s
Suspend
0
End Block
Item: 21 of 41 ~ 1 • M k -<:J 1>- Jil ~· !:';-~
QIO: 3777 ..L ar Pre v ious Next Lab~lues Notes Calcula t o r

1
&
Accessory XI Head turning, shoulder sh rugging (SCM , trapezius) J\1otor J\ latter &

2 Hypoglossal Xll Tongue movement J\totor J\lost


3
4 FA17 p 510.1
5
Ocular motility
6
SuperiOr Superior CN \ 1 innervates the Lateral Rectus.
7 rectus oblique rectus Ci'\ 1\' innervates the Superior O blique.
muscle muscle muscle
8 Medial Cl\ Ill innen·ates the Rest.
Trochlea
9 rectus The "chemical fonnula" LR6 S0-1Rl.
muscle
10
lateral Medial The superior obl ique abducts, intorts, and
rectus rectus
muscle muscle
depresses while adducted.
11
SR 10 SR
12
lateral
Inferior Inferior rectus
13
ObtiQ\Je rectus muscle LR
14 muscle muscle
Inferior Inferior
15 rectus obbque
muscle muscle 13 IR so IR c
16
17
To test each muscle, ask patient to move his/ O bliques go O pposite (left SO and 10 tested
18
her eye in the path diagrammed to the right, with patient looking right).
19 from neutral position toward the muscle being IOU: 10 tested looking Up.
20 tested.
21

a
Lock
s
Suspend
8
End Bl ock
Item: 22 of 41 ~ 1 • M k -<:J 1>- Jil ~· !:';-~
QIO: 3261 ..L ar Pre v ious Next Lab~lues Not es Calcula t o r
A A

2 A 73-year-old woman is brought to the emergency department after her husband noted she had a slight right-sided facial droop and was having
difficulty moving her right side. On physical examination the patient has no problem understanding questions and commands and is able to fully
3
express herself using appropriate language. However, when asked to repeat a sentence, the patient has extreme difficulty trying to do so, and
4 ultimately gives up in frustration.

5
This patient's brain les1on includes which of the following areas?
6
7 :
A. Amygdala
8
B. Arcuate fasciculus
9
10 c. Basal ganglia

11 0. Broca a rea
12
E. Wernicke area
13
14
15
16
17
18
19
20
21
. 22 •
a
Lock Suspend
s 8
End Bl ock
Item: 22 of 41 ~ 1 • M k -<:J 1>- Jil ~· !:';-~
QIO: 3261 ..L ar Prev ious Next Lab~lues Not es Calculat o r

A A

2 Th e co rrect an swer i s B. 630/o ch ose this.

3 In addition to her right-sided hemiparesis, t his patient is having problems with repetition, despite good central suicus
comprehension and fluency. Thus this patient 's lesion is located in the left hemisphere and involves the !fissure of Rola<1do)
4 arcuate fasciculus (depicted here), which connects the Wernicke area to the Broca area . This type of
aphasia, In which a patient is unable to repeat but t he rest of her language skills are preserved, is ca lled ~rcuatt

5 conduction aphasia. t..dculus

Arcuate fasciculus Hemiparesis Wernicke's area Conduction aphasia Aphas1a Lesion B• oca's area
6
Lateralization of bram function
7
8
9
(f11su~ of SyMus)
10
Image courtesy of Peter Hagoort
11
12 A is n ot co rrect. 30/o chose this.

13 Bilateral lesions of the amygdala would cause hyperorality, hypersexuality, and dislnhibited behavior, known as Kliiver -Bucy syndrome.
Amygdala Hypersexuality Disinhibition
14
C is not co rrect. soto chose this.
15 Basal ganglia lesions would cause a movement disorder rather than the language deficit that is described here. Upper motor neuron deficits would be
expected.
16
Basal ganglia Neuron Upper motor neuron Motor neuron Ganglion Movement disorder
17
D is not co rrect. 200/o ch ose this.
18 If the lesion involved the Broca area, an expressive aphasia with broken speech but good comprehension would be seen. The patient would understand
19 that she Is being asked to repeat a sentence, but she would not be able to produce the language to do so.
Aphasia Expressive aphasia Lesion Broca's area
20
E i s n ot correct. 90/o chose this.
21 If the lesion involved the Wernicke area, a recept ive aphasia with poor comprehension would be seen. Although t he patient would have fluent speech, she
would not understand that she is being asked t o repeat a sentence.
22 •
a
Lock Suspend
s 8
End Block
Item: 22 of 41 ~. I • M k <:] t> al ~· ~
QIO: 3261 .l. ar Previous Next Lab 'Vfll ues Notes Calculator

2 Bottom Line:
3 Arcuate fasciculus lesions allow appropriate comprehension and fluent speech, but causes poor repetition.
Arcuate fasciculus
4
5
6
I iii I;fi 1!1 I•J for year:l 2017 ..
7 FIRST AID FAC T S

8
FA17 p486.1
9
10 Aphasia ph asia- higher-order language deficit (inabil ity to understand/speak/read/write).
Dysarthria- motor inabil ity to spea k (movement defi cit).
11
TYPE SPEECH FLUENCY COMPREHENSION COMMENTS
12
Repetition impaired
13
Broca {expressive) on Auent Intact Broca = Broken Boca (boca = mouth in Span ish).
14
Broca area in inferior fronta l gyrus of frontal lobe. Patient
15 appears frustrated, insight intact.
16 Wernicke {receptive) Fluent Impaired Wernicke is Wordy but makes no sense. Patients do not
17 have insight.
18 Wern icke area in superior temporal gyrus of temporal
19 lobe.
20 Conduction Fluent Intact Can be caused by damage to arC uate fasciculus.
21 Global on fluent Impaired Arcuate fasciculus; Broca and Wernicke areas affected
22
(all areas).

6
lock
s
Suspend
0
End Block
Item: 22 of 41 ~ 1 • M k -<:J 1>- Jil ~· !:';-~
QIO: 3261 ..L ar Pre v ious Next Lab~lues Not es Calcula t o r
A A

2 FA17 p481 .1

3
Common brain lesions
AREA OFLESION CONSEQUENCE EXAMPLES
4
Frontal lobe Disinhibition and deficits in concentration,
5
orientation, judgment; may ha,·e reemergence
6 of primiti,·e reflexes.
7 Frontal eye fie lds Eres look toward lesion.
'
8
Paramedian pontine Eyes look away from side of lesion.
9 reticular formation
10 Internuclear ophthalmoplegia (impaired ~ lultiple sclerosis.
Medial longitudinal
11 fasciculus adduction of ipsilateral eye; nystagmus of
12 contralateral eye with abduction).
13 Dominant parietal Agraphia, acalculia, finger agnosia, left-right Cerstmann syndrome.
14 cortex disorientation.
15 Nondomlnant parietal Agnosia of the contralatera I side of the "oriel. llemispatial neglect syndrome.
16
cortex
17 Hippocampus Anterograde amnesia-inability to make new
(bilateral) memories.
18
Basal ganglia 1\lav result in tremor at rest, chorea, athetosis. Parkinson disease, Huntington disease.
19
20 Subthalamic nucleus Contra lateral hemiballismus.
21 Mammillary bodies Wernicke-Korsakoff syndrome- Confusion, Wernicke problems come in a CAN 0 ' beer.
(bilateral) \ taxia, :\'ystagmns, Ophthalmoplegia,
22 • ' .

a
Lock
s
Suspend
8
End Bl ock
Item: 22 of 41 ~ 1 • M k -<:J 1>- Jil ~· !:';-~
QIO: 3261 ..L ar Pre v ious Next Lab~lues Not es Calcula t o r
A A

2 Hippocampus Anterograde amnesia-inability to make new


3 (bilateral) memories.
4 Basal ganglia .I\ lay result in tremor at rest , chorea, athetosis. Parkinson disease, Huntington disease.
5 Subthalamic nucleus Contra latera I hemiballismus.
6 Mammillary bodies Wernicke-Korsakoff syndrome- Confu ion, Wernicke problems come in a C Al\ 0 ' beer.
7 (bilateral) Ataxia, '\ystagmus, Ophthalmoplegia,
8 memory loss (anterograde and retrograde
9 amnesia), confabulation, personalit) changes.
10 Amygdala (bilateral) Kluver-Bucy syndrome-disinhibitcd behavior IISV-1 encephalitis.
(cg, hyperphagia, hypersexuality, h) perorality).
11

12
Superior colliculus Parinaud syndrome- paralysis of conjugate Stroke, hydrocephalus, pinealoma.
vertical gaze (rostral interst itial nucleus also
13
involved).
14
Reticular activating Reduced levels of arousal ;md wakefulness
15 system (midbrain) (eg, coma).
16
Cerebellar hemisphere Intention tremor, limb ataxia, loss of balance; Ccrcbclhtr hemispheres are laterally loc;~t cd ­
17 damage to cerebellum - ipsilateral deficits; affect lateral limbs.
18 fall toward side of lesion.
19 Cerebellar vermis Truncal ataxia, dysarthria. crmis is centrally located-affects central body.
20 Degeneration associated with chronic alcohol
21 usc.
22 •
a
Lock
s
Suspend
8
End Bl ock
Item: 23 of 41 ~ 1 • M k -<:J 1>- Jil ~· !:';-~
QIO: 2949 ..L ar Pre v ious Next Lab~lues Notes Calcula t o r

3
A

A 69-year-old man comes to the physician because he has had difficulty with ambulation for the past 9 months. He is also concerned that his hands
lightly shake at rest. His handwriting has become small and cramped In recent years. On physical examination the patient has postural instability, and
lA• A] A

4
when the physician attempts to move an extremity, it resists and then gives way in step-like movements.
5
6 Which of the following pathological findings would be expected in this patient?

7 :
8 A. Brownish-yellow deposits in the cornea

9 B. Extracellular plaques with cores that stain with Congo red


10
c. Intraparenchymal hemorrhage
11
D. Neurons with round, haloed, eosinophilic bodies in the cytoplasm
12
13 E. Spongiform change within the neuropil

14
15
16
17
18
19
20
21
22
. 23 •
a
Lock
s
Suspend
8
End Bl ock
Item: 23 of 41 ~ 1 • M k -<:J 1>- Jil ~· !:';-~
QIO: 2949 ..L ar Prev ious Next Lab~lues Notes Calculat o r

A A

3 Th e co rrect a nsw er i s o. 710/o ch ose this.


Parkinson disease is characterized by depigmentation and degeneration of the substantia nigra . This can •
4 be visua lized as neurons with eosinophilic, cytoplasmic inclusion bodies consisting of a clear halo •
5 surrounding a dense core. Made of o-synuclein, these inclusions are called Lewy bodies (the dark form in ' ••
.
I
the center of the image). Classically, Parkinson disease presents with "pill-rolling" tremor prominent at
6 rest, hypokinesia, micrographia (small, cramped handwriting), cogwheel rigidity (as seen on physical
exam), and postural instability. Lewy bodies are also seen in the Lewy body variant of Alzheimer disease, \. ...
7 known as Lewy body dementia. •
Oementaa <ith Lewy bodies Substantia nigra Hypokinesia Parkinson's disease Lewy body Alzheamer's disease
8
Tremor Balance disorder Neuron Micrographia (handwriting) Dementia Inclusion bodies Cytoplasm Eosinophilic
9

10 •
11
12 Image courtesy of Suraj Rajan

13
A is no t co rrect. 4 0/o chose this.
14 Hepatolentlcular degeneration (Wilson disease) is caused by copper accumulation, especially in the liver, brain, and Descement membrane of the cornea
15 (Kayser-Fleischer rings) . Although these patients can develop parkinsonian symptoms, Wilson disease usually presents at a younger age and Lewy bodies
are not present. Symptoms include those relat ed to liver disease as well as cognitive deterioration.
16 Wilson's disease Cornea Uver Uver disease Human brain Copper Brain Parkinsonism Parkinson's disease Biological membrane Cognition Kayser-Fleischer rings

17 B is not co rrect . 150/o chose this.

18 This Is a description of senile plaques seen in patients with Alzheimer disease. The plaques contain 13-a myloid cores that stain with Congo red and are
surrounded by cell processes that contain t au protein. Memory loss is the primary clinical manifestation in these patients.
19 Thu protein Senile plaques Protein Alzheimer's disease Dementia Congo red

20 C i s n ot correct. 5 0fo chose this.

21 Intraparenchymal hemorrhage is most frequently caused by hypertension. The cerebral vessels most vulnerable to damage from hypertension are those
that supply the basal ganglia. With intracerebral hemorrhage, the presentation of symptoms is more acute, and an irregularly shaped hyperdenslty Is
22 commonly seen on noncontrast CT. Note that although the risk for intracerebral hemorrhage is increased in patients with Alzheimer disease due to
amyloid angiopathy (which weakens cerebral vessels), amyloid angiopathy Is not seen in Parkinson disease.
23 • Basal oano 1a lntraoarenchvmal hemorrhaoe Hvoertension Cerebra hemorrhaoe Amvloid Alzheimer's disease Cerebral cortex Parkinson's disease Bleedino Brain

a
Lock
s
Suspend
8
End Block
Item: 23 of 41 ~. , . M k <:] t> al ~· ~
QIO: 2949 .l. ar Previous Next lab 'Vfl1 ues Notes Calculator
. .. .. . ..
3 This is a description of senile plaques seen in patients with Alzheimer disea se. The plaques contain j3 -amyloid cores that stain with Congo red and are
surrounded by cell processes that contain t au protein. Memory loss is the primary clinical manifestation in these patients.
4
Tau protein Senile plaques Protein Alzheimer' s disease Dementia Congo red
5
c is no t co rrect. 5 % cho se this.
6 Intraparenchymal hemorrhage is most frequently caused by hypertension. The cerebral vessels most vulnerable to damage from hypertension are those
that supply the basal ganglia. With intracerebral hemorrhage, the presentation of symptoms is more acute, and an irregularly shaped hyperdensity is
7
commonly seen on noncontrast CT. Note that although the risk for intracerebral hemorrhage is increa sed in patients with Alzheimer disea se due to
8 amyloid angiopathy (which weakens cerebral vessels), amyloid angiopathy is not seen in Parkinson disea se.
Basal ganglia Intraparenchymal hemorrhage Hypertension Cerebral hemorrhage Amyloid Alzheimer' s disease Cerebral cortex Parkinson' s disease Bleeding Brain
9
Ganglion
10
E is no t co rrect. 5 % cho se this.
11 Spongiform change within the neuropil is seen in the spongiform encephalopathies, such as Creutzfeld-Jakob disea se, kuru, and fat al familial insomnia.
These disorders are thought to be caused by prions. Prions are infectious protein particles devoid of RNA or DNA that can misfold and accumulat e within
12 the central nervous system . Pathologically, the spongiform encephalopathies are characterized by extensive vacuolation and neuronal cell loss in the
13 brain. Prion disea ses are thought to be transmitted by exposure to prion -containing animal products. Clinical fea tures include at axia, rapidly progressive
dementia, and early dea th .
14 Fatal familial insomnia Neuropil Central nervous system Creutzfeld-Jakob disease Prion Insomnia Ataxia Protein Encephalopathy Dementia

15 Transmissible spongiform encephalopathy Kuru {disease) Nervous system Vacuole Protein folding Proteopathy DNA RNA Brain

16
17 Botto m Li ne:

18 Lewy bodies are found in neurons of patients with Parkinson disea se.
lewy body Neuron Parkinson' s disease
19
20
21
I ill ;fi 1!1 I•J f o r yea r :[ 2017 ..
22 FI RST AID FA CTS

23

6
lock
s
Suspend
0
End Block
Item: 24 of 41 ~ 1 • M k -<:J 1>- Jil ~· !:';-~
QIO: 3633 ..L ar Pre v ious Next Lab~lues Notes Calcula t o r
A A

4 Immediately postpartum, a 34-yea r-old woman requires repair of a perineal laceration. The patient's labor and delivery was without epidural
anesthesia, and the obstetrician plans a bilateral local nerve block with lidocaine before repair of the laceration. The needle is introduced
5
transvaglnally, directed inferolaterally, and the local anesthetic is injected In close proximity to the sacrospinous ligament and ischial spine.
6
7 What Is the spinal root composition of the blocked nerve?

8 :
9 A. L4-51

10 B. L4-S2
11
C. L4-S3
12
D. LS-52
13
14 E. 52-54

15
16
17
18
19
20
21
22
23
. 24 •
a
Lock
s
Suspend
8
End Bl ock
Item: 24 of 41 ~ 1 • M k -<:J 1>- Jil ~· !:';-~
QIO: 3633 ..L ar Prev ious Next Lab~lues Notes Calculat o r

A
Th e co rrect a nswer i s E. 56% ch ose this. A

4
The nerve block described is a transvaginal pudendal nerve block, which provides anesthesia for the m ajority of the perineum and the inferior quarter of
5 the vagina. The pudendal nerve exits the pelvis through the greater sciatic foramen, re-enters the pelvis via the lesser sciatic foramen, and courses In the
Ischiorectal fossa in the Alcock canal. Pudendal nerve fibers are derived from 52-54 nerve roots.
6
Pudendal nerve Isch1oanal fossa Greater sciatic foramen Lesser sciatic foramen Perineum vagina Pudendal anesthesia Anesthesia Pelvis Fossa (anatomy)
7
A i s not correct. 100/o chose t his.
8 This clinical Situation describes a pudend al nerve block, a nerve which derives its fibers from 52-54 nerve roots. L4-51 refers to the superior gluteal nerve,
which leaves the pelvis via the greater sciat ic foramen and innervates the gluteus medius, gluteus minim us, and tensor fascia lata.
9
Pudenda •e Gluteus minimus muscle Fascia lata Greater sciatic foramen Gl te IS medius muscle Superior gluteal nerve Pelvis Superior gluteal artery
10
B is not correct. 11% chose this.
11 This clinical situation describes a pudendal ne rve block, a nerve which derives Its fibers from 52-54 nerve roots. The ne rve roots in this answer choice
describe the composition of the common pe ronea l nerve, a branch of the sciatic nerve, which supplies the skin of the lateral part of the posterior leg and
12 the knee joint and then branches into the superficial peroneal (innervating fibularis longus, brevis, and skin on distal third of anterior leg and dorsum of
13 foot) and deep peroneal (innervating anterior muscles of leg and dorsum of foot).
Pudendal nerve Sciatic nerve Peroneus longus Common peroneal nerve Knee Anatomical terms of location
14
C is not co rrect. 1 00/o chose t his.
15
This clinical situation describes a pudendal nerve block, a nerve which derives its fibers from 52 -54 nerve roots. The nerve roots in this answer choice
16 describe the composition of the tibial nerve, a branch of the sciatic nerve that innervates the posterior muscles of the leg and knee joint.
Pudendal nerve Sciatic nerve Tibial nerve Knee
17
D is not co rrect . 1 30/o chose this.
18 This clinical situation describes a pudendal nerve block, a nerve which derives its fibers from 52-54 nerve roots. LS -52 refers to the inferior gluteal nerve,
19 which leaves the greater sciatic foramen and breaks up into several branches to innervate the gluteus m aximus.
Pudendal nerve Inferior gluteal nerve Greater sciatic foramen Gluteus maximus muscle Pudendal anesthesia Nerve
20
21
Bottom Lin e:
22
The pudendal nerve is derived from nerve roots 52-54 and exits the pelvis through the greater sciatic foramen . It innervates the majority of the
23 perineum and the inferior quarter of the vagina.
Pudenda nerve Greater sciatic foramen Perineum Pelvis Vagina Foramen
24 •
a
Lock
s
Suspend
8
End Block
Item: 24 of 41 ~ .I • M k <:] t> al ~· ~
QIO: 3633 .l. ar Previous Next lab 'Vfl1 ues Notes Calculator

4
5 Bottom Line:
The pudendal nerve is derived from nerve roots 52 -5 4 and exits the pelvis through the greater sciatic foram en . It innervates the m ajority of the
6
perineum and the inferior quarter of the vagina.
7 Pudendal nerve Greater sciatic foramen Perineum Pelvis Vagina Foramen

8
9
10 I iii I;fi 1!1 I•J for year:l 2017 ..
FIRST AID FAC T S
11

12 FA17 p480.3
13 Landmark dermatomes CZ-posterior half of the skull. D iaphragm and gallbladder pain referred to the
14 C3-high turtleneck shirt. right shoulder via phrenic nerve (C3-C5).
15 C4-low-collar shirt.
C6-includes thumbs. Thumbs up sign on left hand looks like a six for C 6.
16
T4-at the nipple. T ..f at the teat p ore.
17
T7- at the xiphoid process.
18 TIO-at the umbilicus {important for early T l O at the belly butten .
7
19 appendicitis pain referral).
20 (6 Ll-at the inguinal ligament. Ll is IL (Inguinal Ligament).
L4-includes the kneecaps. Down on ALL of's (14).
21
SZ, S3, S4-erection and sensation of penile and "S2, 3, 4 keep the pen is off the Aoor."
22
anal zones.
lS
23
24

6
lock
s
Suspend
0
End Block
Item: 25 of 41 ~ 1 • M k -<:J 1>- Jil ~· !:';-~
QIO: 3239 ..L ar Pre v ious Next Lab~lues Notes Calcula t o r

5
A

A 22-year-old man presents to his physician complaining that he is unable to raise his left arm. The physician notes that the patient's arm is hanging
at his side, with his forearm pronat ed. Upon further questioning, the patient states that he is an avid horse-rider and had recently been thrown off his
lA• A] A

6
horse and landed on his shoulder. On physical exam, the patient has 1/5 strength in shoulder abduction.
7
8 In addition to the abductors at the shoulder, what other muscles are likely to be affected by this injury?
9 :
10 A. Biceps brachii

11 B. Diaphragm
12
c. Extensor carpii rad ialis
13
D. Interossei
14
15 E. Pronator teres

16
17
18
19
20
21
22
23
24
• 25 •
a
Lock
s
Suspend
8
End Bl ock
Item: 25 of 41 ~ 1 • M k -<:J 1>- Jil ~· !:';-~
QIO: 3239 ..L ar Pre v ious Next Lab~lues Notes Calcula t o r
A A

5 Th e co rrect a nswer i s A. 620/o ch ose this.


This Is a classic case of Erb palsy, with a "waiter's tip" appearance. When the patient fell, his left shoulder was pulled away
6 from his head, which distracted the upper trunk of the brachial plexus. This trunk is formed by the ventral rami of CS- C6,
which were either stretched or torn by this injury, sensory change illustrated by the shaded portion of the image. The
7
Injury resulted in denervation to muscles that are innervated by the CS-C6 nerve roots (the upper trunk). The muscles
8 Involved include the abductors (deltoid, supraspinatus), the latera l rotators (Infraspinatus), and the biceps.
Brachoa pie us Anter;or ramus of spinal nerve Infraspinatus muscle Supraspinatus muscle Biceps Deltoid muscle Oenervation
9
Anatomical terms of location Upper trunk Nerve root
10
11
12
13
14
15
16
17
18
19
20
21
22
23
24
25 •
a
Lock
s
Suspend
8
End Bl ock
Item: 25 of 41 - ,• Mark --<] [::> ""'I ~· 1!';:'1
QIO: 3239 ~ Prev1o u s Next Labf a lu es Notes Calculator

5 The correct answer is A. 62% chose this.


This is a classic case of Erb palsy, with a "waiter's t ip" appearance. When the patient fell, his left shoulder was pulled away from his head, which distracted the upper
6 trunk of the brachial plexus. This trunk is formed by the ventral rami of C5-C6, which were either stretched or torn by this inj ury, sensory change illustrated by the
7 shaded portion of the image. The inj ury resulted in denervation to muscles that are innervated by the C5-C6 nerve roots (the upper trunk). The muscles involved
include the abductors (deltoid, supraspinatus), the lateral rotators (infraspinatus), and the biceps.
8 Brachtal pie us Anter•or ramus of ~pinal nerve Infraspinatus muscle Supraspinatus muscle Bleeps Oeltotd muscle Oenervatlon Anatom•cal terms of location Upper trunk

9 Nerve root

10
11

12
13
14

15
16
17
18
19
20
21
22
23
24
25
• 26
. 27
• 28
0 29
0 30

8
L.odt
s
Su~pl'nd
~
End Block
Item: 25 of 41 ~. , . M k <:] t> al ~· ~
QIO: 3239 .l. ar Previous Next lab 'Vfllues Notes Calculator

B is no t co rrect. 5 % cho se this.


5
Contributions from the C3- CS roots form the phrenic nerve, which innervat es the diaphragm. This patient's pattern of injury would not be seen in an
6 injury to these rami. The phrenic nerve is primarily a motor nerve with only one third of the fibers providing sensory innervation . Injury to C3- CS would
impair diaphragm contractions and sensory sensation to some components of the mediastinum.
7
Phrenic nerve Mediastinum Motor nerve Thoracic diaphragm
8
c is no t co rrect. 1 7% cho se this.
9 The radial nerve comprises fibers from the c s- T1 roots of the brachial plexus and lends motor innervation to wrist extensors in the forearm . Damage to
the radial nerve can occur due to midshaft humeral fractures or compression of the axilla (crutches, " Sa turday night palsy" ). These lesions lea d to
10 wristdrop as a result of unopposed flexors of the wrist .
11 Brachial plexus Axilla Radial nerve Humerus fracture Anatomical terms of motion Extension (kinesiology) Forearm Humerus Nerve

12 D is no t co rrect. 4 % cho se this.


The ulnar nerve is formed from the C8 - T1 roots (lower trunk) and would not be involved in this injury. A fracture of the medial epicondyle of the humerus
13 or fractured hook of the hamat e (distal ) can lea d to ulnar nerve palsies. This may include "ulnar claw" on digit extension, radial deviation upon wrist
14 flexion, loss of flexion of the wrist and medial fingers, loss of abduction and adduction of the fingers, and loss of function in the medial two lumbrical
muscles.
15 Medial epicondyle of the humerus lumbricals of the hand Humerus Ulnar nerve Hamate bone Adduction Flexion Anatomical terms of location Epicondyle

16 lumbricals of the foot Bone fracture Abduction (kinesiology) Distal lumbrical

17 E is no t co rrect. 1 2% cho se this.


The pronator t eres is innervat ed by the median nerve (CS-Tl ) and pronat es the forearm . Injury to this muscle can cause pronator t eres syndrome, which
18 is characterized by wrist pain and numbness in the distribution of the median nerve. An injury to the C6 - C7 rami would not produce this pattern of injury.
19 Median nerve Pronator teres muscle Pronator teres syndrome Forearm Pronation Pronator Muscle

20
21 Bo tto m Line:
22 Erb palsy is due to injury of the nerves arising from CS and C6 (the upper trunk) . Lesions affect the deltoid, supraspinatus, infraspinatus, and biceps
brachii.
23 Biceps Infraspinatus muscle Supraspinatus muscle Deltoid muscle Cervical spinal nerve 6

24

25

6
lock
s
Suspend
0
End Block
Item: 25 of 41 ~ 1 • M k -<:J 1>- Jil ~· !:';-~
QIO: 3239 ..L ar Pre v ious Next Lab~lues Notes Calcula t o r
A A

5 FA17 p428.1

6 Brach ial plexus lesio ns


7 0 Erb palsy ("watter's up·) C5 Lateral Randr
8
a Klumpke palsy (claw hand) 0 Musculocutaneous Tra,·is
€) Wrist drop (6 D rinks
Axil~ry
9 0 Winged scapula Middle Postencr e Cold
10 e DeltOid parai}'SIS a €) (';)lfntn~l e Median (flexors) Beer
(';) "Saturday mght palsy" (wrist drop)
11
0 Difficulty lleXJng elbow. vanable C8 Rac!YI
12 sensory loss lower Medial

13
e Decreased thumb function. a ~ Ulnar
"Pope's blesSing· T1 I '---.--''
I I I I
Trunks DiviSions Cords Branches
14 ~ lntnnSic muscles of hand. 0
claw hand
15 Long thoracic
16
I
17 Roots

18 CONDITION INJURY CAUSES MUSCLE DEFICIT FUNCTIONAL DEFICIT PRESENTATION

19 Erb palsy ("waite r's Traction or Infants-lateral Deltoid, Abduction (arm


t ip") tear of upper traction on neck supraspinatus hangs by side)
20
{"Erb-er") trunk: during de]i,•ery In fraspinalus Lateral rotation (arm
21
C 5-C6 roots Adults-trauma medially rotated)
22
Biceps brachii Flexion, supination
23
(arm extended and
24
pronated)
25 •
a
Lock
s
Suspend
8
End Bl ock
Item: 25 of 41 ~ 1 • M k -<:J 1>- Jil ~· !:';-~
QIO: 3239 ..L ar Pre v ious Next Lab~lues Notes Calcula t o r
A A

5 Klumpke palsy Traction or tear Infants-upward Intrinsic hand Tota l claw hand:
6 of lower trunk: force on arm muscles: lumbricals normallv
C8-TI root during deli,·ery lumbricals, Aex .YICP joints and
7
Adults-trauma interossei, extend DIP and PIP
8
(eg, grabbing a thenar, joints
9 tree branch to hypothenar
10 break a fall )
11 Thoracic outlet Compression Cervical rib Same as Klumpkc Atrophy of intrinsic
12 syndrome of lower trunk (arrows in ), palsr hand muscles;
13
and subcla,·ian Pancoast tumor ischemia, pain,
,·essels and edema
14
due to vascular
15
compression
16
Winged scapula Lesion of long Axillary node Serratus anterior Inability to anchor
17 thoracic nerve dissection after scapula to thoracic
18 mastectomy, cage - cannot
19 stab wounds abduct arm above
20
horizontal position
21
22
FA17 p 4 27.1
23 Upper extremity nerves
24 NERVE CAUSES OF INJURY PRESENTATION
25 • Axillarv ICS-(6) Fractured sun~ica l neck of humems: anterior Flattened deltoid
a
Lock
s
Suspend
8
End Bl ock
Item: 25 of 41 ~. , . M k <:] t> al ~· ~
QIO: 3239 .l. ar Previous Next Lab 'Vfll ues Notes Calculator

• •
5 FA17 p 4 27.1
Upper extremity nerves
6
NERVE CAUSES OF INJURY PRESENTATION
7
Axillary (CS-C6) Fractured surgical neck of humerus; anterior Flattened deltoid
8
dislocation of humerus Loss of arm abduction at shoulder(> 15°)
9 Loss of sensation over deltoid muscle and lateral
10 arm
11 Musculocutaneous Upper trunk compression Loss of forearm Aexion and supination
12 (CS-C7) Loss of sensation over lateral forearm
13 Radial (C5-T1) Midshaft fracture of humerus; compression of Wrist drop: loss of elbow, wrist, and fin ger
14
axilla, eg, due to crutches or sleeping with arm extension
over chair ("Saturday night pa lsy") l grip strength (wrist extension necessary for
15
maxima l action ofAexors)
16 Loss of sensation over posterior ann/forearm and
17 dorsal hand
18 Median (CS-T1) Supracondylar fracture of humerus (proximal "Ape hand" and "Pope's blessing"
19 lesion); carpal tunnel syndrome and wrist Loss of wrist Aexion, Aexion of lateral fingers,
laceration (distal lesion) thumb opposition, lumbricals of 2nd and 3rd
20
digits
21 Loss of sensation over thenar eminence and
22 dorsal and palmar aspects of lateral 3Y2 lingers
23 with proximal lesion
24 Ulnar (C8-T1 ) Fracture of medial epicondyle of humerus "Ulnar claw" on digit extension
"funny bone" (proximal lesion); fractured Radial deviation of wrist upon Aexion (proximal •
25 •

6
lock
s
Suspend
0
End Block
Item: 25 of 41 ~ 1 • M k -<:J 1>- Jil ~· !:';-~
QIO: 3239 ..L ar Pre v ious Next Lab~lues Notes Calcula t o r
A A

5 Ulnar (C8-T1 ) Fracture of medial epicondyle of humerus "U lnar claw" on digit extension
6
"funny bone" (proximal lesion); fractured Radial deviation of wrist upon Aexion (proximal
hook of hamate (distal lesion) from fall on lesion)
7
outstretched hand Loss of wrist Aexion, Aex ion of medial fingers,
8 abduction and adduction of fingers (interossei),
9 actions of medial 2 lumbrical muscles
10 Loss of sensation over medial 11h fingers
including hypothenar eminence
11
Recurrent branch of Superficial laceration of palm " pe hand"
12
media n ne rve (CS-T1) Loss of thenar muscle group: opposition,
13 abduction, and Aexion of thumb
14 'o loss of sensation
15 ll umerus fractures, proximally to distally, follow the ARI\1 ( \\illary - Radial - \1edian)
16
cs
Ax1llarynerve ~ g
17 ...t:: C8
Tl
~ Median nerve
18 Axillary nerve~
Musculocutaneous nerve------.:. / '17 Ulnar nerve
19 _,-Intercostobrachial
20 ~Radial nerve .-" nerve
Rad1al nerve --.._: .....__Radial nerve
..,..-Medial brach1al
21 cutaneousnerve Palmof hand
22
Med1an nerve _ / " " " - - Ulnar nerve ~Medial antebrachial
23 Musculocutaneous nerve"""'-
.-' cutaneous nerve Median nerve --...........
24
Radial nerve/
~ Ulnar nerve
25 •
a
Lock Suspend
s 8
End Bl ock
Item: 25 of 41 ~ 1 • M k -<:J 1>- Jil ~· !:';-~
QIO: 3239 ..L ar Pre v ious Next Lab~lues Notes Calcula t o r
A
actions of medial 2 lumb rical muscles A

5
Loss of sensation over med ial 11h fingers
6 including hypothenar eminence
7 Superficial laceration of palm
Recurrent branch of '' pe hand"
8 median nerve (CS-T1) Loss of thenar muscle group: opposition,
9 abduction, and flexion of thumb
t-:o loss of sensation
10
11 llumerus fract ures, proximallr to distally, follow the \R\1 ( h illarr - Radial - \1edia n)
12 cs

13
Axlllary nerve~
.,t.
g
Tl
~ Medaan nerve
14 'I? Axillary nerve -..._
Musculocutaneous nerve------.:. ~
Ulnar nerve -........
15 ~ Intercostobrachial
"'--- Radial nerve .-' nerve
16 Rad1al nerve --.,__ .._Radial nerve
/ Medial brachaal
17 cutaneous nerve Palm of hand

18
19
Median nerve_/'~ /
Ulnar nerve
Musculocutaneous nerve ~ Medial antebrachial
cutaneousnerve
20 Radial nerve_/, , /.
_.....-Ulnar nerve
Radial nerve--.........
21 Recurrent branch """"'-... Radial nerve ~-
of med1an nerve ,
22
Dorsum of hand
23
24
25 •
a
Lock
s
Suspend
8
End Bl ock
Item: 26 of 41 ~ 1 • M k -<:J 1>- Jil ~· !:';-~
QIO: 320 9 ..L ar Pre v ious Next Lab~lues Notes Calcula t o r
A A

6 A 64-year old man is being investigated for recurrent headaches and unintentional weight loss. He is a heavy smoker but otherwise well with no
comorbldlties. CT brain showed multiple enhancing lesions suggestive of metastases. The largest lesion is found to be in the area of the left
7
subthalamic nucleus.
8
9 Which clinical features would you expect as a result of this lesion?

10 :
11 A. Parkinsonism

12 B. Spastic paralysis of the left arm and leg


13
c. Spastic paralysis of the right arm and leg
14
D. Uncontrollable flailing of the left arm and leg
15
16 E. Uncontrollable flailing of the right arm and leg

17
18
19
20
21
22
23
24
25
. 26 •
a
Lock Suspend
s 8
End Bl ock
Item: 26 of 41 ~ 1 • M k -<:J 1>- Jil ~· !:';-~
QIO: 320 9 ..L ar Prev ious Next Lab~lues Notes Calculat o r

A A

6 Th e co rrect an sw er i s E. 690/o chose this.

7 I n the Indirect pathway, the substant ia nigra pars compacta sends inhibitory neurons to the striatum . From the striat um comes inhibitory output to the
external segment of the globus pallidus, which in t urn inhibits the subthalamic nucleus. The subthalamic nucleus sends excitatory neurons to the Internal
8 segment of the globus pallidus, which increases t he firing of t he inhibitory GABA neurons that run from the GPi to t he t halamus, t hereby inhibiting
movement on the contra lat era l side of t he body. Lesion of t he left subthalamic nucleus would thus lead t o less inhibit ion t hrough t he globus pa llldus,
9 resulting In hemiballismus of t he contra lateral side, or wild, uncontrollable movement of the right arm and leg. This is somewhat similar to the
pathophysiology of Huntingt on disea se (HD), an aut osomal dominant disease caused by trinucleot ide repeat expansion. I n HD, degeneration of the
10
GABAnerglc striatal neurons of t he indirect pathway ca uses a similar decrease In motor inhibition. Pat ient s wit h HD exhibit chorea and dystonia in addition
11 to memory defic1ts and personalit y changes.
•,.;nucleotide epeat disorder Substantia nigra Hemiballismus Huntington's disease Globus paloodus Subthalamic nucleus Thalamus Gamma-Aminobutyric acid
12
Dystonia Striatum Dominance (genetics) Chorea Pars compacta Anatomical terms of location Contralateral Neuron Lesion Pathophysiology
13 Ind' ect p.Jthway of movement Medial globus pallidus Autosome Excitatory postsynaptic potential Cell nucleus
14 A i s no t co rrect. 6% chose this.
15 Parkinsonism Is characterized by features such as rest ing tremor, bradykinesia, rigidity, and masked facies. It ca n result from neurodegeneratlon of the
substantia nigra (idiopathic Parkinson disease ), 1-methyl -4-phenyl -1,2,3,6-tetrahydropyridine (MPTP) exposure, adverse effects of neuroleptic medication,
16 or Ischemic damage to the basal ganglia. Deep brain stimulation of the subthalamic nucleus is a neurosurgical t reatment for medication -refractory
Parkinson disease.
17
Basal ganglia Bradykinesia Substantia nigra Deep brain stimulation Subthalamic nucleus Parkinsonism Parkinson's disease MPTP Neurodegeneration Antipsychotic
18 Tremor Idiopathy Neurosurgery Ganglion Ischemia Pharmaceutical drug Human brain

19 8 is not co rrect. 3 0fo chose this.


20 Spastic paralysis of t he left arm and leg would most likely be ca used by a lesion In the right primary motor co1t ex, right medullary pyramid, or left lateral
white matter tract of t he spinal cord above cs, but would not be seen with a lesion In the left subthalamic nucleus.
21 Subthalamic nucleus Primary motor cortex Medullary pyramids (brainstem) White matter Motor cortex Spinal cord Spasticity Lesion Paralysis Cortex (anatomy)
22 Spastic

23 C i s n ot correct. 130/o ch ose this.

24 Although lesions in the basal ganglia ca n lead to hyperkinetic or hypoklnetlc movement disorders, they do not lead to paralysis. Paralysis indicates a lesion
In the motor neurons themselves, and spastic paralysis is indicative of an upper motor neuron lesion. The corticospinal neurons cont rolling the right side
25 of the body originate in the left primary motor cortex and pass t hrough the posterior limb of the internal capsule, t he midbrain cerebral peduncles,
ventral pons, and medullary pyramids, all on t he left side. The majority of these corticospinal neurons decussate at t he ca udal medulla, run In the lateral
26 • white matter tracts of the riaht side of t he soinal cord. and finally svnaose on the lower motor neurons in t he ventra l horn of t he soinal cord arav matter.

a
Lock
s
Suspend
8
End Block
Item: 26 of 41 ~. I • M k <:] t> al ~· ~
QIO: 3209 .l. ar Previous Next lab 'Vfllues Notes Calculator

6 c is no t co rrect. 13% cho se this.


Although lesions in the basal ganglia can lea d to hyperkinetic or hypokinetic movem ent disorders, they do not lea d to paralysis. Paralysis indicat es a lesion
7 in the motor neurons themselves, and spastic paralysis is indicative of an upper motor neuron lesion. The corticospinal neurons controlling the right side
of the body originat e in the left primary motor cortex and pass through the posterior limb of the internal capsule, the midbrain cerebral peduncles,
8 ventral pons, and m edullary pyramids, all on the left side. The m ajority of these corticospinal neurons decussat e at the caudal m edulla, run in the lat eral
9 white m atter tracts of the right side of the spinal cord, and finally synapse on the lower motor neurons in the ventral horn of the spinal cord gray m atter.
A lesion affecting both the right arm and leg, but not the right face, is likely to be localized to either the arm and leg regions of the primary motor cortex
10 or to the region of the corticospinal tract between the m edullary pyramids and CS.
Upper motor neuron lesion Basal ganglia Internal capsule Midbrain Pons Primary motor cortex Upper motor neuron Pyramidal tracts White matter Synapse
11
Anterior grey column Decussation Neuron Motor cortex Cerebral peduncle Spinal cord Motor neuron Medullary pyramids (brainstem) lesion Grey matter
12
lower motor neuron Paralysis Medulla oblongata Hypokinesia Spasticity Ganglion Anatomical terms of location Movement disorder Cortex (anatomy)
13 Cerebral cortex Ventral Caudal (anatomical term)

14 D is no t co rrect. 9 % cho se this.


15 Lesion of the right, not left, subthalamic nucleus would lea d to hemiballismus of the left arm and leg.
Hemiballismus Subthalamic nucleus lesion
16
17
Botto m Li ne:
18
Lesion of the subthalamic nucleus lea ds to hemiballismus of the opposite -side arms and legs.
19 Hemiballismus Subthalamic nucleus lesion
20
21
22 lijl;fiiJI•l toryear:[2017 • ]
FI RST AID FAC T S
23
24
FA17 p 4 89.1
25 Movement disorders
26 DISORDER PRESENTATION CHARACTERISTICLESION NOTES

6
lock
s
Suspend
0
End Block
Item: 26 of 41 ~ 1 • M k -<:J 1>- Jil ~· !:';-~
QIO: 320 9 ..L ar Pre v ious Next Lab~lues Notes Calcula t o r
A A

6 FA17 p489.1
7 Movement disorders
DISORDER PRESENTATION CHARACTERISTIC LESION NOTES
8
9
Akathisla Restlessness and intense urge Can be seen with neuroleptic
to move use or in Parkinson disease.
10
Asterixis Extension of wrists causes Associated with hepatic
11
'"flapping" motion encephalopathy, \ ilson
12
disease, and other metabolic
13 derangements.
14 Athetosis Slow, snake-like, \\Tithing Basal ganglia
15 movements; especially seen in
16 the fingers
17 Chorea Sudden, jerky, purposeless Basal ganglia Chorea =dancing.
18 movements Sydcnham chorea seen in acute
rheumatic fever.
19
Dystonia Susta ined, involuntary muscle \ Vriter's cramp, blepharospasm,
20
contractions torticoll is.
21
Essential tremor II igh-frcquency I rem or Often familia I. Pal icnls often
22
with sustained posture self-medicate with alcohol,
23 (eg, outstretched arms), which l tremor amplitude.
24 worsened with mo\·cmcnt or Treatment: nonselective
25 when anxious ~-bl ockers (eg, propranolol),

26 primidone.

a
Lock
s
Suspend
8
End Bl ock
Item: 26 of 41 ~. I • M k <:] t> al ~· ~
QIO: 3209 .l. ar Previous Next Lab 'Vfll ues Notes Calculator

• •
6 Hemiballismus Sudden, wild Railing of I arm Contralateral subthalamic Pronounce "Half-of-body
7 +1- ipsilateral leg nucleus (eg, lacunar stroke) ball istic."
Contralateral lesion.
8
9
Intention tremor Slow, zigzag motion when Cerebellar dysfunction
pointing/extending toward a
10
target
11
Myoclonus Sudden, brief, uncontrolled Jerks; hiccups; common in
12
muscle contraction metabolic abnormalities such
13 as renal and liver failure.
14 Resting tremor Uncontrolled movement of distal Substantia nigra (Parkinson Occurs at rest; "pill-rolling
15 appendages (most noticeable disease) tremor" of Parkinson disease.
16 in hands); tremor alleviated by When you park your car, it is
17
intentional movement at rest.

18
19 FA 17 p481 .1

20 Common brain lesions


AREA Of LESION CONSEQUEN CE EXAMPLES
21
Frontal lobe Disinhibition and defi cits in concentration,
22
orientation, judgment; may have reemergence
23 of primitive reflexes.
24
Frontal eye fields Eyes look toward lesion.
25
Paramedian pontine Eyes look away from side of lesion.
26 • rotir1 d;:ar fnrm;:at inn •

6
lock Suspend
s 0
End Block
Item: 26 of 41 ~ 1 • M k -<:J 1>- Jil ~· !:';-~
QIO: 320 9 ..L ar Pre v ious Next Lab~lues Notes Calcula t o r
A A

6 FA17 p481 .1
7 Common brain lesions
AREAOFLESION CONSEQUENCE EXAMPLES
8
Frontal lobe Disinhibition and deficits in concentration,
9
orientation, judgment; may have reemergence
10
of primitive reflexes.
11
Frontal eye fie lds Eves

look toward lesion.
12
Paramedian pontine Eyes look away from side of lesion.
13
reticular formation
14
Medial longitudinal Internuclear ophthalmoplegia (impaired ~ lultiple sclerosis.
15 fa sciculus adduction of ipsilateral eye; nystagmus of
16 contralateral eye with abduction).
17 Dominant parietal Agraph ia, acalculia, finger agnosia, left-right Cerstmann srndrome.
18 cortex disorientation.
19 Nondominant parietal Agnosia of the contra latera I side of the world. 1-lemispatial neglect syndrome.
20 cortex
21 Hippocampus Anterograde amnesia-inability to make new
22
(bilateral) memoncs.
23 Basal ganglia l\ lay result in tremor at rest, chorea, athetosis. Parkinson disease, Huntington disease.
24 Subthalamic nucleus Contralateral hemiballismus.
25 Mammillary bodies Wernicke-Korsakoff syndrome- C onfmion, Wernicke problems come in a C. \ N 0 ' beer.
26
(bilateral) Ataxia, ~ystagmus, O phthalmoplegia,

a
Lock
s
Suspend
8
End Bl ock
Item: 26 of 41 ~ 1 • M k -<:J 1>- Jil ~· !:';-~
QIO: 320 9 ..L ar Pre v ious Next Lab~lues Notes Calcula t o r

6
A
cortex A

7 Hippocampus Anterograde amnesia-inability to make new


8
(bilateral) memones.
9 Basal ganglia t.. lay result in tremor at rest, chorea, athetosis. Parkinson disease, Huntington disease.
10 Subthalamic nucleus Contralateral hemiba II ismus.
11 Mammillary bodies Wernicke-Korsakoff syndrome- Confusion, Wernicke problems come in a C.\ :\ 0 ' beer.
12 (bilateral) Ataxia, :"\ystagmus, Ophthalmoplegia,
memory loss (anterograde and retrograde
13
amnesia), confabulation, personaI it} changes.
14
Amygdala (bilateral) Kluver-Bucy syndrome-disinhibited behavior IISV-1 encephalitis.
15
(eg, hyperphagia, hypersexuality, hypcrorality).
16
Superior colliculus Parinaud syndrome-paralysis of conjugate Stroke, hydrocephalus, pinealoma.
17 vertical gaze (rostral interstitial nucleus also
18 involved).
19 Reticular activating Reduced levels of arousal and W<lkcful ness
20 system (midbrain) (cg, coma).
21 Cerebellar hemisphere Intention tremor, limb ataxia, loss of balance; Cerebellar hemispheres are laterally located-
22 damage to cerebellum - ipsilateral defi cits; affect lateral limbs.
23
fall toward side of lesion.
24 Cerebellar vermis Truncal ataxia, dysarthria. crmis is centrally located-affects central body.
Degeneration associated with chronic alcohol
25
usc.
26 •
a
Lock
s
Suspend
8
End Bl ock
Item: 26 of 41 ~ 1 • M k -<:J 1>- Jil ~· !:';-~
QIO: 320 9 ..L ar Pre v ious Next Lab~lues Notes Calcula t o r
A A

6 FA17 p 4 70.1
7 Basal ganglia Important in voluntary movements and making postura l
8 adjustments.
9 Recei,·es cortical input, pro,·ides negati' e feedback to cortex to
modulate mo,·emenl.
10
Striatum= putamen (motor) + caudate (cogniti,e). 0 1-Receptor = DIRect
11
Lentiform= putamen+ globus pallidus. pathway.
12 Indirect= Inhibitory.
13 Input from SNc
14
15
16
Dopallllne •• Stimulatory

Inhibitory

SNc Substantia nigra pars compacta


17
GPe Globus pallid us externus
18 GPi Globus pallidus 1nternus
19
.
Direct lnd1rect'
STN Subthalamic nucleus

20
Motor cortex pathway pathway o, Dopamine D1 receptor
facilitates inhibits Dz Dopamine D2 receptor
21 movement

22
23
24
25 Thalamus

26 • t
a
Lock
s
Suspend
8
End Bl ock
6
SNc Substantia nigra pa~ compacta
7
GPe Globus pallidus extemus
8 GPi Globus pallidus 1ntemus
9 .
Direct lnduect'
STN Subthalamic nucleus

10 Motor cortex pathway pathway o. Dopamine 01 receptor


faditates •nh•bits Dz Dopamine 0 2 receptor
11 movement
12
13
14
15 Thalamus

16
17
18
19
20
21
Pedunculo-
22
23
24
Spinal
25 cord
26 •
a
Lock
s
Suspend
8
End Block
6
7
8
Thalamus
9
10
11
12
13
14
15 Pedunculo-
pontine
16 nucleus
17
18 Spinal
19 cord

20 Excitatory pathway-cortical inputs stimulate the striatum, stimulating the release of GABA, which
21 inh ibits GABA release from the GPi, disinhibiting the thalamus via the GPi (f motion).
22 Inhibitory pathway-cortical inputs stimulate I he strial um, releasing GABA that disinhibits ST I
23
via GPe inhibition, and ST l stimulates C Pi to inhibit the thalarnus (l motion).
Dopamine binds to D1, stimulating the excitatory pathway, and to D2• inhibiting the inhibitory
24
pathway .... f motion.
25
26 •
a
Lock
s
Suspend
8
End Block
Item: 27 of 41 ~ 1 • M k -<:J 1>- Jil ~· !:';-~
QIO: 4593 ..L ar Pre v ious Next Lab~lues Notes Calcula t o r
A A

17 A 70-year-old woman with a history of atrial fibrillation visits her doctor because she is having trouble walking. This began a few days ago, following
what she described as a "senior moment" in which she was a little dizzy and confused . Physical examination reveals unilat eral leg weakness and
18 decreased sensation, but no visual deficits.
19
20 A
21
B
22
23
24
25
26 c
. 27
. 28
. 29
o30
D
• 31
• 32
• 33
• 34
o35
E
. 36
. 37 •
a
Lock
s
Suspend
8
End Bl ock
17
18
19
20
21
22
23
24
25
26
. 27
. 28
. 29
Infarction of which artery in the image would lead to contralateral deficits of the leg and foot?
o30
:
• 31 A
• 32
B
• 33
• 34
c
o35 0

. 36 E
. 37 •
a
Lock
s
Suspend
8
End Block
Item: 27 of 41 ~. I • M k <:] t> al ~· ~
QIO: 4593 .l. ar Previous Next lab 'Vfl1 ues Notes Calculator

17
The co rrect a nswer is A. 57% cho se this.
18
The image depicts the circle of Willis. The artery labeled A is the anterior cerebral artery (ACA ), which supplies the medial surface of the brain, the area
19 responsible for the contralat eral leg and foot area s of the motor and sensory cortices. Thus, a lesion in the artery would lea d to deficits in contralat eral
motor function of the leg and foot .
20 Circle of Willis Anterior cerebral artery Anatomical terms of location Contralateral Human brain Cerebral arteries lesion Brain
21 B is no t co rrect. 1 8% cho se this.
22 Label B points to the middle cerebral artery (MCA). A deficit in the MCA would cause contralat eral face and arm paralysis and sensory loss.
Middle cerebral artery Paralysis Cerebral arteries Anatomical terms of location Contralateral
23
c is no t co rrect. 11% cho se this.
24
Label C points to the posterior cerebral artery (PCA). A deficit in the PCA would cause contralat eral hemianopia with macular sparing .
25 Posterior cerebral artery Macula of retina Contralateral Hemianopsia

26 D is no t co rrect. 8 % cho se this.


27 Label D points to the anterior inferior cerebellar artery (AICA). A deficit in the AICA would lea d to lat eral inferior pontine syndrome.
Anterior inferior cerebellar artery Cerebellum Pons Inferior lateral genicular artery Anatomical terms of location
• 28
E is no t co rrect. 6 % cho se this •
• 29 Label E points to the posterior inferior cerebellar artery (PICA). A deficit in the PICA would lea d to lat eral medullary syndrome.
o30 Posterior inferior cerebellar artery lateral medullary syndrome

• 31
• 32 Botto m Line:
• 33 The ACA supplies the medial surface of the brain, the area responsible for the contralat eral leg and foot area s of the motor and sensory cortices.
Anatomical terms of location Contralateral Human brain Brain
o34
o35
• 36
lijj ;fi IJ l•l f o r yea r :l 20 1 7 ..
• 37 FI RST AID FAC T S

6
lock
s
Suspend
0
End Block
Item: 27 of 41 ~ 1 • M k -<:J 1>- Jil ~· !:';-~
QIO: 4593 ..L ar Pre v ious Next Lab~lues Notes Calcula t o r
A A

17 FA17 p473.1

18
Circle of Willis ystem of anastomoses between anterior and posterior blood supplies to brain.
19 I
ACorn Anterior .
commurucating - - -- - - --, , - - - Optoc chiasm
20
21 I ..

X: '"-: 'I"
ACA Anterior
cerebral ---<:::_
22
ACA
23
24
0
Antenor
ClfCUUtlon
ACA
I
MCA Middle _.
ce.rebral - - - -
- ·
~
CIRCLE

#~ striate
./ ~MCA

OICA M1j : OF ...-- / /PCA


25
26
O MCA
Posterior
I
PC 001 Posterior \
communicating - - - - - - f .
WILLIS 10f
Jidal ~:.-----BA
0 Clrculat•on External
27
• 28
I
PCA Posterior_.----::;
cerebral
ECA carotid
artery
'---.,---PCom
·~-.....;.--ICA
Common '---~
• 29 CCA carotid
artery - - -.,.....-iEJ
o30
• 31
I
SCA Superior
cerebellar - - - '
Pont1ne
Brachio·
cephalic .....:._....::.~\/
• 32

• 33
I
AICA Anterior inferior
cerebellar - - - - - '!..rj. Basilar I BA
Aota
• 34
o35 I
PICA Posterior inferior
__,.~ / J
'f \
J
~ tl - Vertebral I VA
OBLIOUE·LATERAL VIEW
cerebellar _ _ _
• 36
• 37
INFERIOR VIEW
/ ~ AnteriOf ~ nal 1ASA

a
Lock
s
Suspend
8
End Bl ock
Item: 27 of 41 ~ 1 • M k -<:J 1>- Jil ~· !:';-~
QIO: 4593 ..L ar Pre v ious Next Lab~lues Notes Calcula t o r
A A

17
FA17 p 4 72.2
18
Cerebral arteries- cortical distribution
19
20 Antenor cerebral artery (supplies anteromedial surface)

21 Middle cerebral artery (supplies lateral surface)

22 • Postenor cerebral artery (supplies posterior and inferior surfaces)

23
24
25
26

27
• 28
• 29
o30
• 31
• 32
Watershed zones Between anterior cerebral/m idd le cerebral, posterior cerebral/middle cerebral arteries. Damage by
• 33 se\·ere hypotension --+ upper leg/upper arm weakness, defects in higher-order visual processing.
• 34
o35 FA17p 484.1
• 36 Effects of strokes
• 37 ARTERY AREA OF LESION SYMPTOMS NOTES

a
Lock
s
Suspend
8
End Bl ock
Item: 27 of 41 ~. I • M k <:] t> al ~· ~
QIO: 4593 .l. ar Previous Next lab 'Vfl1 ues Notes Calculator

• FA17 p484.1 •
17
18
Effects of strokes
ARTERY AREA OF LESION SYMPTOMS NOTES
19
Anterior circulation
20
Middle Motor and sensory cortices fJ - upper Contralateral paralysis and sensory Wernicke aphasia is associated
21
cerebral limb and face. loss-face and upper limb. with right superior quadrant
22 artery Temporal lobe (Wernicke area); Aphasia if in dominant (usually visua 1 field defect due to
23 frontal lobe (Broca area). left) hemisphere. I lemineglect temporal lobe involvement.
24 if lesion affects nondominant
(usually right) side.
25
26
Anterior Motor and sensory cortices-lower Contralateral paralysis and sensory
cerebral limb. loss- lower limb.
27
artery
• 28
Lenticulo- Striatum, internal capsule. Contralateral paralysis and/or Common location of lacunar
• 29 striate sensory loss-face and body. infarctsrn, due to hyaline
o30 artery Absence of cortical signs arteriosclerosis zoto
• 31 (eg, neglect, aphasia, visual field unmanaged hypertension.
loss).
• 32
Posterior circulation
• 33
o34
Anterior Lateral corticospinal tract. Contralateral paralysis-upper and Medial medullary syndrome-
spinal lower limbs. caused by infarct of
o35
artery Medial lemniscus. l contralateral proprioception. paramedian branches of ASA
• 36 Caudal medulla-hypoglossal nerve. Ipsilateral hypoglossal dysfunction and/or vertebral arteries.
• 37 •
(tongue deviates ipsilaterally). •

6
lock
s
Suspend
0
End Block
Item: 27 of 41 ~ 1 • Ma rk -<:J 1>- Jil ~· !:';-~
QIO: 4593 ..L Prev ious Next Lab~lues Not es Cal culat o r

A A

17 Posterior Lateral medulla: Lateral medullary (Wallenberg)


inferior 'ucleus ambiguus (C1 IX, X, XI ) Dy~ ph agia , hoarseness, l gag syndrome.
18
cerebellar Vestibular nuclei rcflc\ Nucleus ambiguus effects arc
19
artery Lateral spinothalamic tract, spinal Vomiti ng, vertigo, nystagmus specific to PICA lesions
20 trigeminal nucleus l pain and temperature sensation "Don't pick a (PICA) horse
21 from contra latera I body, (hoarseness) that can' t cat
22 ipsilateral face (dysphagia)."
ympathetic fibers Ipsilateral Horner S) ndrome Also supplies inferior cerebellar
23
Inferior cerebellar peduncle Ataxia, d)Smetria peduncle (part of cerebellum).
24
Anterior Lateral pons Lateral pontine syndrome.
25
inferior Facial nucleus Paraly'i' of face, l lacrimation, Facial nucleus effects are
26 cerebellar specific to AICA lesions.
l salin 1tion, l taste from anterior
27 artery ~ of tongue "Facial droop means AICA's
. 28 Vestibular nuclei Vomiting, vertigo, nystagmus pooped."
. 29 Spinothalamic tract, spinal l pain and temperature sensation Also supplies middle and
trigem ina! nucleus from conira lateral body, inferior cerebellar peduncles
o30
ipsilatera I face (part of cerebellum).
• 31
Sympathetic fibers Ipsilatera I llorner sync! rome
• 32 Middle and inferior cerebellar taxia, dysmetria
• 33 peduncles
• 34 Basilar artery Pons, medulla, lower midbrain RAS spared, therefore prcscr"cd "Locked-in syndrome."
o35 COnSCIOIISI1CSS
. 36 Corticospinal and corticobulbar Quadriplegia; loss of ' olnntary
tracts facial, mouth, and longue
. 37 •
a
Lock
s
Suspend
8
End Bl ock
Item: 27 of 41 ~ 1 • M k -<:J 1>- Jil ~· !:';-~
QIO: 4593 ..L ar Pre v ious Next Lab~lues Notes Calcula t o r
A
.. .. .. - ~ ~ -
A

17 peduncles
18 Basilar artery Pons, medulla, lower midbrain RAS spared, therefore preserved "Locked-in syndrome."
19 eonSCIOIISness
20 Corticospinal and corticobulbar Quadriplegia; loss of ' oluntary
tracts facial, mouth, and longue
21
1110\·ements
22
Ocular cranial nerve nuclei, Loss of horizontal, but not vertical,
23 paramedian pontine reticular e\e 1110\ ements
'
24 formation
25 Posterior Occipitallobc [!]. Contralateral hemianopia with
26 cerebral macular sparing.
artery
27
. 28
. 29
o30
• 31
• 32
• 33
• 34
o35
. 36
. 37 •
a
Lock
s
Suspend
8
End Bl ock
Item: 28 of 41 ~. , . M k <:] t> al ~· ~
QIO: 5197 .l. ar Previous Next Lab 'Vfll ues Notes Calculator


17 An 89 -year -old man is brought by his family to his primary care physician. They are concerned that he has started making inappropriat e sexual
comments and gestures. His family members are also aware that he has increa singly been putting nonedible items into his mouth. His past medical
18 history is notable for several strokes over the last year. Physical exam is remarkable for residual weakness in his right arm, dysarthria, and gait
19 unstea diness, which he has as a consequence of his strokes. Testing of primitive reflexes is negative.

20
Damage to which of the following regions would most likely produce the new symptoms of hypersexuality and hyperora lity observed in this patient?
21
22
A. Amygdala
23
B. Arcuat e fasciculus
24
25 c. Frontal lobe

26 D. Mammillary bodies
27
E. Right pariet al lobe
• 28
• 29
o30
• 31
• 32
• 33
o34
o35
• 36
• 37 •

Q
lock
s
Suspend
0
End Block
Item: 28 of 41 ~. , . M k <:] t> al ~· ~
QIO: 5197 .l. ar Previous Next Lab 'Vfll ues Notes Calculator

17 The co rrect a nswer is A. 65% cho se this.


18 Hypersexuality, hyperorality (fixation on oral exploration ), and disinhibited behavior are suggestive of KIOver -Bucy syndrome, a constellation of symptoms
that can also include visual agnosia and anterograde amnesia. It results from insult to the amygdala bilat erally, often secondary to ischemic stroke. A
19 history of previous strokes, as evidenced in this case by the patient's clumsiness, at axia, and dysarthria (characteristic of lacunar infarcts), is a strong risk
factor for the development of KIOver -Bucy syndrome. Other causes of amygdala injury include infection (encephalitis) and trauma.
20
B is no t co rrect. 4 % cho se this.
21 The arcuat e fasciculus forms the neural connections between Wernicke and Broca area s in the left t emporal lobe. A lesion here results in conduction
aphasia; patients have comprehensible and fluent speech (unlike Wernicke and Broca aphasia), but are unable to repea t words or phrases.
22
c is no t co rrect. 23 % cho se this.
23
Although patients with frontal lobe lesions may exhibit findings that mimic those resulting from amygdala lesions (eg, hypersexuality and inappropriat e
24 speech), a frontal lobe lesion can be differentiat ed from an amygdala lesion by t esting the primitive reflexes, such as the snouting, Moro, grasp, and
palmomental reflexes. In the case of a frontal lobe lesion, these primitive reflexes, which are absent in healthy adult patients, may return . As this patient
25 did not exhibit primitive reflexes, a frontal lobe lesion is less likely.
26 D is no t co rrect. 5 % cho se this.
A lesion in the mammillary bodies bilat erally results in Wernicke -Korsakoff syndrome, a constellation of symptoms often seen in long-standing alcoholics
27 with thiamine deficiency. Patients with Wernicke -Korsakoff present with confusion and gait at axia, and are at risk for permanent anterograde and
28 retrograde amnesia, as well as confabulation .
E is no t co rrect. 3 % cho se this •
• 29
Insult to the right pariet al lobe results in a profound spatial neglect of the contralat eral visual field. Despite intact ocular pathways, patients with this
o30 neglect are unaware of the left side of the world and are unaware of their deficit (anosognosia ) .
• 31
• 32 Botto m Li ne:

• 33 KIOver -Bucy syndrome results from bilat eral lesions of the amygdala. It is marked by hypersexuality, hyperorality, disinhibited behavior; and occasionally
anterograde amnesia. Unlike frontal lobe lesions, there is less often a change in personality, impaired judgment, or the return of primitive reflexes.
o34
o35
• 36
lijl;fiiJI•l foryear:[2017 • ]
• 37 FIRST AID FACTS

6
lock
s
Suspend
0
End Block
Item: 28 of 41 ~ 1 • M k -<:J 1>- Jil ~· !:';-~
QIO: 5197 ..L ar Pre v ious Next Lab~lues Notes Calcula t o r
A A

17 FA17 p 481 .1

18
Common brain lesions
AREA OFLESION CONSEQUENCE EXAMPLES
19
Frontal lobe Disinhibition and deficits in concentration,
20
orientation, judgment; may ha"e reemergence
21 of primiti,·e reflexes.
22
Frontal eye fie lds Eyes look toward lesion.
23
Paramedian pontine Eyes look away from side of lesion.
24 reticular formation
25 Medial longitudinal Internuclear ophthalmoplegia (impaired ~ lultiple sclerosis.
26 fasciculus adduction of ipsilateral eye; nystagmus of
27 contralateral ere with abduction).
28 Dominant parietal Agraph ia, acalculia, finger agnosia, left-right Cerstmann syndrome.
• 29 cortex disorientation.
o30 Nondomlnant parietal Agnosia of the contra latera I side of the "oriel. ll emi~ patial neglect syndrome.
• 31 cortex
• 32 Hippocampus Anterograde amnesia-inability to make new
(bilateral) memories .
• 33
• 34
Basal ganglia 1\ lav result in tremor at rest, chorea, athetosis. Parkinson disease, Huntington disease.
o35 Subthalamic nucleus Contra lateral hemiballismus.
• 36 Mammillary bodies Wernicke-Korsakoff syndrome- Confusion, Wernicke problems come in a CAN 0 ' beer.
(bilateral) .\taxia, :\'ystagmus, Ophthalmoplegia,
• 37 •
a
Lock
s
Suspend
8
End Bl ock
Item: 28 of 41 ~ 1 • M k -<:J 1>- Jil ~· !:';-~
QIO: 5197 ..L ar Pre v ious Next Lab~lues Notes Calcula t o r
A A

17 Hippocampus Anterograde amnesia-inability to make new


18 (bilateral) memories.
19 Basal ganglia l\ lay result in tremor at rest , chorea, athetosis. Parkinson disease, Huntington disease.
20 Subthalamic nucleus Contra latera I hemiballismus.
21 Mammillary bodies Wernicke-Korsakoff syndrome- Confu ion, Wernicke problems come in a CA:\ 0 ' beer.
22 (bilateral) Ataxia, '\ystagmus, Ophthalmoplegia,
23 memory loss (anterograde and retrograde
24 amnesia), confabulation, personalit) changes.
25 Amygdala (bilateral) Kluver-Bucy syndrome-disinhibitcd behavior IISV-1 encephalitis.
26
(cg, hyperphagia, hypersexuality, h) perorality).

27
Superior colliculus Parinaud syndrome-paralysis of conjugate Stroke, hydrocephalus, pinealoma.
vertical gaze (rostral interst itial nucleus also
28
involved).
• 29
Reticular activating Reduced levels of arousal ;md wakefulness
o30
system (midbrain) (eg, coma).
• 31
Cerebellar hemisphere Intention tremor, limb ataxia, loss of balance; Cerebellar hemispheres are laterally located-
• 32 damage to cerebellum - ipsilatera l clc~ci t s; affect lateral limbs.
• 33 fall toward side of lesion.
• 34 Cerebellar vermis Truncal ataxia, dysarthria. ennis is centrally located-affects central body.
o35 Degeneration associated with ch ronic alcohol
• 36 use.
• 37 •
a
Lock
s
Suspend
8
End Bl ock
Item: 28 of 41 ~ 1 • M k -<:J 1>- Jil ~· !:';-~
QIO: 5197 ..L ar Pre v ious Next Lab~lues Notes Calcula t o r
A A

17 FA17 p 484.1
18 Effects of strokes
ARTERY AREA OFLESION SYMPTOMS NOTES
19
Anterior circulation
20
21 Middle lotor and sensor) cortices []-upper Contralateral para I) sis and sensor~ Wernicke aphasia is associated
cerebral limb and face. loss-face and upper limb. with right superior quadrant
22
artery Temporal lobe (Wernicke area); ph asia if in dominant (usually ,·isual field defect due to
23 frontal lobe (Broca area). left) hemisphere. llemineglect temporal lobe involvement.
24 if bion affects nondominant
25 (usuallr right) side.
26 Anterior lotor and sensory cortices-lower Contralateral paralysis and sensory
cerebral limb. loss-lower limb.
27
artery
28
Lenticulo- Striatum, internal capsule. Contralateral paralysis and/or Common location of lacunar
infarcts rn,due to hyaline
• 29
striate sensorr loss-face and body.
o30 artery Absence of cortical signs arteriosclerosis zoto
• 31 (eg, neglect, aphasia, visual field unmanaged hypertension.
• 32 loss).
• 33 Posterior circulation
• 34 Anterior Lateral corticospinal tract. Contralateral paralrsis-upper and Medial medullary syndrome-
spinal lo11er limbs. caused by infarct of
o35
artery Medial lemniscus. l contralateral proprioception. paramedian branches of /\SA
• 36
Caudal medulla-hypoglossal nerve. lp ilateral h) poglossal dysfunction and/or vertebral arteries.
• 37 • (tomwe de,·iates iosilaterall\').

a
Lock
s
Suspend
8
End Bl ock
Item: 29 of 41 ~ 1 • M k -<:J 1>- Jil ~· !:';-~
QIO: 3222 ..L ar Pre v ious Next Lab~lues Not es Calcula t o r
A A

17 A 72-year-old man is noted to have slowed voluntary movements and Increased resistance to passive movement of his elbow joints. He is also noted
to fall backward easily when gently pushed from the front . His symptoms have been worsening over the past several years.
18
19
Receptors for the neurotransmitter that is deficient in t his illness are located In which of the following brain regions?
20
:
21 A. Globus pallidus
22
B. Putamen
23
24 c. Substantia nigra

25 D. Subthalamic nucleus
26
E. Thalamus
27
28
. 29
o30
• 31
• 32
• 33
• 34
o35
. 36
. 37 •
a
Lock
s
Suspend
8
End Bl ock
Item: 29 of 41 ~. I • M k <:] t> al ~· ~
QIO: 3222 .l. ar Previous Next Lab 'Vfll ues Notes Calculator

17

18 The co rrect a nswer is B. 35% cho se this.


Parkinson disea se is a bradykinetic movement disorder that results from degeneration of dopaminergic neurons of the substantia nigra. The main
19
dopaminergic pathway in the basal ganglia is the nigrostriat al pathway, projecting from the substantia nigra to the striatum (putamen and caudat e),
20 where there are two main classes of dopamine receptors. D1 receptor stimulation results in excitatory modulation of the direct pathway, which facilitat es
movement, wherea s 0 2 receptor stimulation provides inhibitory modulation to the indirect pathway, which inhibits movement. Thus the end result of
21 dopaminergic stimulation to both pathways is the facilitation of movement. The cardinal symptoms of Parkinson disea se are cogwheel rigidity, resting
tremor; bradykinesia, and postural instability. Additional symptoms include akinesia, a characteristic shuffling gait, decrea ses in eye blink frequency, and
22 decrea sed facial expression.
23 A is no t co rrect. 13 % cho se this.

24 Although the globus pallidus is part of the basal ganglia, it does not contain the dopamine receptors that play a key role in Parkinson disea se. The
putamen projects to the globus pallidus.
25
c is no t co rrect. 37% cho se this.
26 The nigrostriat al dopaminergic pathway originat es in the substantia nigra pars compacta. Neurons that relea se dopamine originat e in the substantia nigra
and extend their axons into the striatum . The dopamine receptors are locat ed in the striatum, which consists of the caudat e and putamen.
27
D is no t co rrect. 9 % cho se this.
28 Although the subthalamic nucleus is part of the basal ganglia, it does not contain the dopamine receptors that play a key role in Parkinson disea se. The
29 globus pallidus projects to the subthalamic nucleus, which projects to the thalamus and back to the globus pallidus. Lesions of the subthalamic nuclei
result in contralat eral hemiballismus.
o30 E is no t co rrect. 6 % cho se this.
• 31 The thalamus receives projections from the basal ganglia via the subthalamic nucleus. Dopamine receptors involved in Parkinson disea se are locat ed in
the striatum, not the thalamus.
• 32

• 33
Botto m Line:
o34
Parkinson disea se is characterized by degeneration of dopaminergic neurons of the substantia nigra; these neurons project to the striatum (putamen
o35 and caudat e) to facilitat e movement .
• 36
• 37

6
lock
s
Suspend
0
End Block
Item: 29 of 41 ~. I • M k <:] t> al ~· ~
QIO: 3222 .l. ar Previous Next Lab 'Vfll ues Notes Calculator

18 Bottom Line:
19 Parkinson disease is chara cterized by degeneration of dopaminergic neurons of the substantia nigra ; these neurons project to the striatum (putam en
20 and caudate) to facilitate movem ent.
Substantia nigra Putamen Striatum Dopaminergic Parkinson' s disease Caudate nucleus Neuron
21
22

23 lijl;fiiJI•l foryear:[2017 • ]
FI RST AID FAC T S
24
25
FA17 p468.3
26
Dopaminergic Commonly a lte red by drugs (eg, antipsychotics) a nd movement disorders {eg, Parkinson disease).
27
pathways
• 28
PATHWAY SYMPTOMS OF AlTEREO ACTIVITY NOTES
29
Mesocortical l activity - "negative" symptoms (eg, anergia, Antipsychotic drugs have l imited effect.
o30 apathy, lack of spontaneity).
• 31 Mesolimbic t ~• ct i ,· ity - "positive" symptoms (eg, delusions, ]
0
then1peutic ta rget of antipsychotic drugs
• 32 hallucinations). - l positive symptoms (eg, in schizophre nia).
• 33 Nigrostriatal l activitr - extrapyra midal symptoms Major clopamine rgic pathway in bra in.
o34 (cg, dystonia, a kathisia, parkinsonism, tardive Significantly affected by movement disorde rs
o35 dyskinesia). and antipsycho tic drugs.

• 36 Tuberoinfundibular l activity - t prolactin - l li bido, sexua l


• 37 dysfunction, galactorrhea, gynecomastia (in
.• men).

6
lock
s
Suspend
0
End Block
Item: 29 of 41 ~ 1 • M k -<:J 1>- Jil ~· !:';-~
QIO: 3222 ..L ar Pre v ious Next Lab~lues Not es Calcula t o r
A A

18 FA17 p 4 70.1

19 Basal ganglia Important in voluntary 1110\'ements and making postura l


20 adjustments.
21
Recei,·es cortical input, pro,·ides negali\ e feedback to cortex to
modulate mo,·emenl.
22
Striatum = putamen (motor) + caudate (cogniti' e). D 1-Receptor = D IRect
23 Lentiform =putamen+ globus pallidus. pathway.
24 Indirect= Inhibitory.
25 Input from SNc
26 • Stimulatory
Dopall'llne
27 • Inhibitory
. 28
SNc Substantia nigra pars compacta
29 GPe Globus pallid us externus
o30 GPi Globus pallidus 1nternus

• 31
.
Diroct lnd1rect'
STN Subthalamic nucleus
Motor cortex pathway pathway 01 Dopamine D1 roceptor
• 32 facilitates inhibits 02 Dopamine D2roceptor
• 33 movement

• 34
• 35
. 36
Thalamus
. 37
.• t
a
Lock
s
Suspend
8
End Bl ock
Item: 29 of 41
QIO: 3222
~
..L
1 • Ma r k -<:J
Pre v ious
I>
Next Lab
fJ
lues
£!1}>'

Not es
!!":-~
Calcula t o r
A A
Input from SNc
18
• Stimulatory
19 Dopam1ne
• Inhibitory
20
21 SNc Substlntia nigra pars compacta
GPe Globus paltidus extemus
22 GPi Globus pallidus mternus
23 .
Direct lnd~rect
• STN Subthalamic nucleus
24 MotOf cortex pathway pathway 01 Dopamine D1 receptor
facilitates 1nh1bits D2 Dopamine D2 receptor
25 movement
26
27
. 28
29 Thalamus

o30
• 31
• 32
• 33
o3 4
• 35
Pedunculo·
. 36 pontine
nucleus
. 37
.•
a
Lock
s
Suspend
8
End Bl ock
Item: 29 of 41 ~ 1 • M k -<:J 1>- Jil ~· !:';-~
QIO: 3222 ..L ar Pre v ious Next Lab~lues Not es Calcula t o r
A facilitates InhibitS 02 Dopam1ne 02 receptor A

18 movement

19
20
21
Thalamus
22
23
24
25
26
27
. 28 Pedunculo-
pontine
29
nucleus
o30
• 31
Spinal
• 32 cord

• 33
Excitatory pathway-cortical inputs stimulate the striatum, stimulating the release of GABA, which
• 34 inh ibits GABA release from the GPi, disinhibiting the thalamus via the GPi (t motion).
• 35 Inhibitory pathwar-cortical inputs stimulate I he slriatum, releasing GABA that disinhibits ST I
. 36 via GPe inhibition, and ST 1 stimulates C Pi to inhibit the thalamus U motion).
. 37
Dopamine binds to D 1, stimulating the excitatory pathway, and to D2• inhibiting the inhibitory
.• path,,·a,· - t motion .

a
Lock
s
Suspend
8
End Bl ock
Item: 30 of 41 ~ 1 • M k -<:J 1>- Jil ~· !:';-~
QIO: 3781 ..L ar Pre v ious Next Lab~lues Notes Calcula t o r

18
A

A 17-year-old man presents to his physician with what he describes as "shooting pain" in his right hand. He says the problem started 2 months ago,
after he had a snowboarding accident in which he directly struck a tree with his arms raised . On examination, he is noted to have decreased sensation

lA A] A

19 over the medial aspect of his right palm, as well as atrophy of the hypothenar eminence. He is also unable to maintain finger abduction against
resistance.
20
21 Which part of the brachial plexus is most likely injured in this patient?
22
:
23 A. Lateral cord
24
B. Lower trunk
25
c. Middle trunk
26
27 o. Posterior cord

. 28 E. Upper trunk
29
o30
• 31
• 32
• 33
• 34
• 35
. 36
. 37
.•
a
Lock
s
Suspend
8
End Bl ock
Item: 30 of 41 ~ 1 • M k -<:J 1>- Jil ~· !:';-~
QIO: 3781 ..L ar Pre v ious Next Lab~lues Notes Calcula t o r
A A

18
19 Th e correct an sw er i s B. 5 40/o ch ose this.
The lower trunk of the brachial plexus is made up of the CB and T1 nerve roots. It contributes to the medial and posterior cords, and eventually to the
20
axillary, radial, median, and ulnar nerves. The unifying factor in the presentation above is that the ulnar nerve innervates the hypothenar muscles, two of
21 the lumbrical muscles, and all of the interosseous muscles; also, it gives off two sensory branches: ( 1) the palmar cutaneous branch and (2) the dorsal
cutaneous branch, which supplies the skin over the hypothenar eminence, and the dorsal ulnar aspect of the hand, respectively. The only answer choice
22 that contributes to the formation of the ulnar nerve is the lower trunk of the brachial plexus.
Hypothen. emo ence Brachoal plexus Lumbricals of the hand Ulnar nerve Lumb cal oft oe foot Palmar interossei muscles Interosseous muscles of the hand
23
lntero~sei Cervical spinal nerve 8 Lumbrical Axilla Anatomical terms of location Palmar branch of the median nerve Nerve root
24
A i s not co rrect. 11% chose this.
25
The lateral cord of the brachial plexus contributes to the musculocutaneous nerve, which primarily innervates the biceps muscle, as well as the median
26 nerve, which supplies most flexors of the arm and thenar muscles. It does not contribute any branches to the ulnar nerve.
Musculocutaneous nerve Brachial plexus Thenar eminence Ulnar nerve Median nerve Lateral cord Biceps Anatomical terms of motion Muscle
27
C is not co rrect. 1 5% ch ose t h is•
. 28
The middle trunk of the brachial plexus, arising from the C7 nerve root, supplies branches to the lateral and posterior cords. The lateral cord contributes
29 to the musculocutaneous nerve, which primarily innervates the biceps muscle, as well as the median nerve, which supplies most flexors of the arm. The
posterior cord gives rise to the axillary and radial nerves. None of these nerves Innervate the structures of the hand listed in the stem.
30 Musculocutaneous nerve Brachial plexus Median nerve Lateral cord Biceps Posterior cord Nerve root Axilla Anatomical terms of motion Muscle

• 31 Anatomical terminology Cervical spinal nerve 7 Nerve

• 32 D is not co rrect . 130/o ch ose t h is .


The posterior cord of the brachial plexus contributes to the axillary and radial nerves. Injury to these nerves would not produce the signs and symptoms
• 33
described above.
• 34 Brachial plexus Posterior cord Axilla

• 35 E i s n ot correct. JO/o ch ose this .


The upper trunk of the brachial plexus originates from the CS-C6 nerve roots, and contributes to the lateral and posterior cords. It does not contribute to
. 36
the formation of the ulnar nerve.
. 37 Brachial plexus Ulnar nerve Nerve root Upper trunk

.•
a
Lock
s
Suspend
8
End Bl ock
Item: 30 of 41 ~. I • M k <:] t> al ~· ~
QIO: 3781 .l. ar Previous Next lab 'Vfllues Notes Calculator

18 Bottom Line:
19 The ulnar nerve originates from the C8 -Tl nerve roots, which form the lower trunk of the brachial plexus. The ulnar nerve innervates the hypothenar
muscles, two of the lumbrical muscles, and all of the interosseous muscles.
20 Brachial plexus lumbricals of the hand Ulnar nerve Hypothenar eminence lumbricals of the foot Interossei Interosseous muscles of the hand lumbrical
Palmar interossei muscles
21
22

23
I iii I;fi 1!1 I•J for year:l 2017 ..
24 FIRST AID FAC T S

25
FA17 p428.1
26
27 Brachial plexus lesions
• 28
0 Erb palsy ("waiter's tip") cs
E) Klumpke palsy {claw hand)
29 E) Wrist drop C6
30 0 Winged scapula Middle
• 31 0 Deltoid paralysis C7
(;) ·saturday night palsy· {wrist drop)
• 32 0 Difficulty flexing elbow. variable CB
sensory loss lower
• 33

o34
(l) Decreased thumb function. a
"Pope's blessing· T1 I
Trunks
o35 0 Intrinsic muscles of hand. 0
claw hand
• 36 long thoracic
• 37
.• I
Roots

6
lock
s
Suspend
0
End Block
Item: 30 of 41 ~ 1 • M k -<:J 1>- Jil ~· !:';-~
QIO: 3781 ..L ar Pre v ious Next Lab~lues Notes Calcula t o r
A A

18 CONDITION INJURY CAUSES MUSCLE DEFICIT FUNCTIONAL DEFICIT PRESENTATION

19 Erb palsy ("waiter's Traction or Infants-lateral Deltoid, Abduction (arm


tip" ) tear of upper traction on neck ~upmspinatus hangs by side)
20
("Erb-er") trunk: during delhcry Infraspinatus Lateral rotation (arm
21
C5-C6 roots Adults-trauma medially rotated)
22
Biceps brachii Flexion, supination
23
(arm extended and
24 pronated)
25
26 Intrinsic hand Total claw hand:
Klumpke palsy Traction or tear Infants- upward
27 of lower trunk: force on arm muscles: lumbricals normally
• 28 C8-TI root during del j, err lumbricals, fl ex ~!C P joints and
29 Adults-trauma interossei, extend Dl P and PIP
(eg, grabbing a thenar, joints
30
tree bra nch lo hypothenar
• 31
break a fall)
• 32
Thoracic outlet Compression Cervical rib Same as Klumpkc Atrophy of intrinsic
• 33 syndrome of lower trunk (arrows in r.J). palsy hand muscles;
• 34 and subclavian Pancoast tumor ischemia, pain,
• 35 ,·essels and edema
• 36
due to vascular
compressiOn
• 37
.• Winged scapula Lesion of long Axillary node

a
Lock
Serratus anterior lnabilitr to anchor
- s
Suspend
8
End Bl ock
Item: 31 of 41 ~ 1 • M k -<:J 1>- Jil ~· !:';-~
QIO: 3852 ..L ar Pre v ious Next Lab~lues Notes Calcula t o r

18
A

An 86-year-old right-handed woman presents to the emergency department with right-sided paralysis of the arm and face with forehead sparing. She
Is able to move the left side of her body, can follow commands, and appears to understand the questions she is being asked. However, her speech Is
lA• A] A

19 very slow and lacks a normal grammatical structure. She appears frustrated with her communication difficulty.

20
What region of the brain is most likely ischemic?
21
22 :
A. Left inferi or frontal gyrus
23
24 B. Left posterior corpus ca llosum

25 c. Left superior tempora l gyrus

26
D. Right angular gyrus
27
E. Right precentral gyrus
. 28
29
30
• 31
• 32
• 33
o34
• 35
. 36
. 37
.•
a
Lock
s
Suspend
8
End Bl ock
Item:31of41 ~. , . M k <:] t> al ~· ~
QIO: 3852 .l. ar Previous Next lab 'Vfl1 ues Notes Calculator

The co rrect a nswer is A. 63% cho se this.


18
This patient presents with a right -sided hemiparesis and an expressive aphasia, in which speech is nonfluent, but comprehension is intact. Since this
19 patient is right -handed, it is assumed that her language center is locat ed in the left hemisphere (for left-handed individuals, there is also a left -
hemisphere language predominance). Expressive aphasia, also known as Broca aphasia, occurs with a lesion in Broca area, most likely locat ed on the left
20 inferior frontal gyrus for this patient. Due to its proximity to the primary motor cortex, lesions involving Broca area also often cause paralysis of the lower
face and arm, which explains the right -sided hemiparesis.
21 Hemiparesis Inferior frontal gyrus Expressive aphasia Primary motor cortex Aphasia Broca's area Motor cortex lateralization of brain function lesion Paralysis
22 language center Handedness Cortex (anatomy)
23 B is no t co rrect. 5 % cho se this.
24 The left posterior corpus callosum, or splenium, is locat ed on the medial surface of the cerebral cortex. Lesions result in severe rea ding impairment.
Corpus callosum Splenium Cerebral cortex Cortex (anatomy)
25
c is no t co rrect. 23% cho se this.
26
Wernicke area is locat ed on the posterior part of the left superior t emporal gyrus. Lesions result in a receptive aphasia known as Wernicke aphasia.
27 Comprehension of written and spoken language is severely impaired. Speech is fluent but meaningless. Associat ed paralysis is uncommon.
Receptive aphasia Superior temporal gyrus Aphasia Wernicke's area Paralysis
• 28
D is no t co rrect. 4 % cho se this.
29
The angular gyrus is locat ed in the pariet al lobe above the superior t emporal sulcus. A lesion to the right angular gyrus (in most patients, the
30 nondominant hemisphere) would cause hemineglect, lack of attention to a whole visual field, lack of recognition of a given body part (known as
asomat agnosia ), abnormal localization of sensations, and dressing apraxia.
31
Lesions to the angular gyrus in the dominant hemisphere (the left hemisphere, in the majority of patients) would result in a pattern of deficits known as
• 32 Gerstmann syndrome. Patients suffer deficits in calculation (acalculia), writing (agraphia), naming objects, directional orientation, and planned
movements. Rea ding is classicaly not impaired with this syndrome (agraphia, but not alexia, are present) .
• 33
Gerstmann syndrome Parietal lobe Acalculia Hemispatial neglect Angular gyrus Superior temporal sulcus Apraxia Agraphia lesion Dyslexia
o34
lateralization of brain function Visual field Parietal bone
o35
E is no t co rrect. 5 % cho se this •
• 36 The right primary motor cortex is locat ed on the precentral gyrus and the anterior wall of the central sulcus ( Brodmann area 4 ). Lesions result in left -
• 37 sided hemiparesis.
.• Hemiparesis Precentral gyrus Primary motor cortex Central sulcus Motor cortex Brodmann area Sulcus (neuroanatomy) Cortex (anatomy) Cerebral cortex

6
lock
s
Suspend
0
End Block
Item:31of41 ~. , . M k <:] t> al ~· ~
QIO: 3852 .l. ar Previous Next lab 'Vfl1 ues Notes Calculator

18 Bottom Line:
Lesions of the inferior frontal gyrus in the dominant hemisphere produce an expressive aphasia, or Broca aphasia, in which speech is nonfluent, but
19
comprehension is intact.
20 Inferior frontal gyrus Expressive aphasia Aphasia lateralization of brain function

21
22

23 lijl;fiiJI•l toryear:[2017 • ]
FI RST AID FAC T S
24
25 FA17 p 4 86.1
26 Aphasia ph asia- higher-order language deficit (inabil ity to understand/spea k/read/wrile).
27 Dysarthria- motor inabil ity to speak (movement defi cit).
• 28 TYPE SPEECH FLUENCY COMPREHENSION COMMENTS

29 Repetition impaired
30 Broca (expressive) onAuent Intact Broca = Broken Boca (boca = mouth in Spanish).
31
Broca area in inferior fronta l gyrus of frontal lobe. Patient
appears frustra ted, insight intact.
• 32
Wernicke (receptive) Fluent Impa ired Wernicke is Wordy but makes no sense. Patients do not
• 33
have insight.
o34
Wernicke area in superior temporal gyrus of temporal
o35 lobe .
• 36 Conduction Fluent Intact Can be caused by damage to arCuate fasc iculus.
• 37
Global on Auent Impa ired Arcuate fasciculus; Broca and Wernicke areas affected
.•
6
lock
s
Suspend
0
End Block
Item: 31 of 41 ~ 1 • M k -<:J 1>- Jil ~· !:';-~
QIO: 3852 ..L ar Pre v ious Next Lab~lues Notes Calcula t o r
A A

18 Re petition intact
19 Transcortical motor NonAuent Intact ffects frontal lobe around Broca area, but Broca area is
20
spared.
21 Transcortical sensory Fluent Impaired ffec ts tcmporal lobc around Wernicke area, but
Wernicke area is spared.
22
Transcortical, mixed 'onAuent Impaired Broca and Wernicke areas and arcuate fasciculus remain
23
intact; surrounding watershed areas affected.
24
25
26 FA17p481 .1
Common brain lesions
27
AREA OFLESION CONSEQUENCE EXAMPLES
• 28
Frontal lobe Disinhibition and defi cits in concentration,
29
orientation, judgment; may ha\'e reemergence
30 of primitive reflexes.
31 Frontal eye fields Eyes look toward lesion.
• 32
Paramedian pontine Eyes look away from side of lesion.
• 33 reticular formation
o34 Medial longitudinal Internuclear ophthalmoplegia (impaired Multiple sclerosis.
• 35 fasciculus adduction of ipsilateral eye; nystagmus of
• 36 contralateral eye with abduction).
• 37 Dominant parietal Agraphia, acalculia, fin ger agnosia, left-right Ccrstmann syndrome.
.• cortex disorientation
a
Lock
s
Suspend
8
End Bl ock
Item: 31 of 41 ~ 1 • M k -<:J 1>- Jil ~· !:';-~
QIO: 3852 ..L ar Pre v ious Next Lab~lues Notes Calcula t o r
A A
Nondominant parietal Agnosia of the contralatera I side of the world. Hemispatial neglect syndrome.
18
cortex
19
Hippocampus Anterograde amnesia-inability to make new
20
(bilateral) memones.
21
Basal ganglia lay result in tremor at rest, chorea, atheto is. Parkinson disease, Huntington disease.
22
Subthalamic nucleus Contralateral hemiba II ism us.
23
Mammillary bodies Wernicke-Korsakoff syndrome- Confttsion, Wernicke problems come in a C \:\ 0 ' beer.
24
(bilateral) Ataxia, ~ystagmus, Ophthalmoplegia,
25 memory loss (anterograde and retrograde
26 amnesia), confabulation, personalit} changes.
27 Amygdala (bilateral) KIOver-Bucy syndrome-disinhibited behavior IISV-1 encephalitis.
. 28 (eg, hyperphagia, hypersexuality, hypcrorality).
29 Superior colliculus Parinaud syndrome-paralysis of conjugate Stroke, hydrocephalus, pinea loma.
30 vertical gaze (rostral interstitial nucleus also
involved).
31
• 32
Reticular activating Reduced levels of arousal and wakeful ness
system (midbrain) (eg, coma).
• 33
Cerebellar hemisphere Intention tremor, limb ataxia, loss of balance; Cerebellar hemispheres are laterally located-
o34
damage to cerebellum - ipsilatera l deficits; affect lateral limbs.
• 35
fall toward side of lesion .
. 36
Cerebellar vermis Truncal ataxia, dysarthria. crmis is centrally located-affects central body.
. 37 Degeneration associated with chronic alcohol
.•
a
Lock
s
Suspend
8
End Bl ock
Item: 31 of 41 ~ 1 • M k -<:J 1>- Jil ~· !:';-~
QIO: 3852 ..L ar Pre v ious Next Lab~lues Notes Calcula t o r
A • • • • • A

18 Cerebellar vermis Truncal ataxia, dysarthria. ennis is centrally located-affects central body.
19 Degeneration associated with chronic alcohol
20 use.
21
22 FA17p 471 .1

23 Cerebral cortex regions


24 Premotor c ~lsut

25
26 Frontal - ~ \
~F~ It/
27
f
lobe
! t 1 - _" .....
• 28 -.......... ~·~¢!'»~
29

30 Brocaarea ~ Vern1cke area


•Occipital
31
Temporal \ lobe
• 32 / , lobe '- Primary
Sylv1an fissure / visual cortex
• 33
Lm b Primary
o34
association a•ea auditory cortex
• 35
• 36
• 37
.•
a
Lock
s
Suspend
8
End Bl ock
Item: 33 of 41 ~ 1 • M k -<:J Jil ~· !:';-~
QIO: 3774 ..L ar Pre v ious Next Lab~lues Notes Calcula t o r

18
A

A 45-year-old woman with a long history of m enstrual irregularities and Infertility presents with complaints of worsening vision . A thorough review of
systems reveals the presence of constipation and cold intolerance. The physician performs basic visual field testing and m aps the visual disturbance In
IA•A] A

19 the patient's chart.

20 Defect in visual field of


21 Left eye Right eye

22
23
A
0
24 B () ()
25
26 c () ()

~ ~
27
D
. 28

~ ~
29
E
30
31
Which of the following visual field defects is most likely in this patient?
• 32
:
• 33 A
o34
B
• 35
. 36 c
. 37 D
.•
a
Lock
s
Suspend
8
End Bl ock
Item: 33 of 41 ~ 1 • M k -<:J 1>- Jil ~· !:';-~
QIO: 3774 ..L ar Pre v ious Next Lab~lues Notes Calcula t o r

Defect in visual field of


A A

18
Left eye Right eye
19
20
21
A
0
22
8 () ()
23
24
c () ()
25
26
D
~ ~
27
. 28
E
~ ~
29 Which of the following visual field defects is most likely in this patient?
30
A
Correctl :
31
The patient has bitemporal hemianopia secondary to a lesion compressing the optic chiasm. The
• 32 B history of m enstrual irregularities, cold Intolerance, and constipation suggests multiple endocrine disorders that can be
attributed to dysfunction of the anterior pituitary. Due to the close anatomic relationship of the pituitary and the optic
33 chiasm, expanding lesions of the pituitary can compress the optic chiasm, lea ding to visual loss in the temporal
c portions of the visual fields bilaterally. This is because only the nasal (or m edial) retinal fibers for each eye cross In the
o3 4
optic chiasm, and the medial retinal fibers are responsible for the temporal (or lateral) hemifields as depleted In the
• 35 0 stem image. Common lesions leading to bitemporal hemianopia include pituitary adenoma, meningioma,
craniopharyngioma, and hypothalamic glioma.
. 36 E
. 37
.•
a
Lock
s
Suspend
8
End Bl ock
Item: 33 of 41 ~. I • M k <:] t> al ~· ~
QIO: 3774 .l. ar Previous Next lab 'Vfllues Notes Calculator

18 The co rrect a nswer is B. 8 4 % cho se this.


19 The patient has bitemporal hemianopia secondary to a lesion compressing the optic chiasm. The history of menstrual irregularities, cold intolerance, and
constipation suggests multiple endocrine disorders that can be attributed to dysfunction of the anterior pituitary. Due to the close anatomic relationship of
20 the pituitary and the optic chiasm, expanding lesions of the pituitary can compress the optic chiasm, lea ding to visual loss in the t emporal portions of the
visual fields bilat erally. This is because only the nasal (or medial ) retinal fibers for ea ch eye cross in the optic chiasm, and the medial retinal fibers are
21 responsible for the t emporal (or lat eral ) hemifields as depicted in the stem image. Common lesions lea ding to bitemporal hemianopia include pituitary
22 adenoma, meningioma, craniopharyngioma, and hypothalamic glioma.
Bitemporal hemianopsia Craniopharyngioma Meningioma Pituitary adenoma Optic chiasm Glioma Anterior pituitary Constipation Pituitary gland Hypothalamus
23 lesion Hemianopsia Adenoma Endocrine system Endocrinology Visual field
24
A is no t co rrect. 4 % cho se this.
25 This defect is monocular visual loss caused by complet e destruction of the ipsilat eral retina or ipsilat eral optic nerve. A lesion in this area would not
account for this patient's endocrine abnormalities.
26
Retina Optic nerve lesion Visual impairment Ipsilateral Endocrine system Anatomical terms of location Monocular
27
c is no t co rrect. 6 % cho se this •
• 28 This defect is contralat eral homonymous hemianopia, which can be caused by lesions of the contralat eral optic tract (in this case, the right optic tract), a
lesion of the contralat eral optic radiation, or lesions diffusely damaging the contralat eral primary visual cortex. In general, retrochiasmal lesions (those
29 distal to the optic chiasm including the optic tracts, lat eral geniculat e nucleus, optic radiations, or visual cortex) cause homonymous visual field defects
30 (meaning the same regions of the fields for both eyes are involved). Lesions in these area s would not account for this patient's endocrine abnormalities.
Homonymous hemianopsia Optic radiation Optic chiasm lateral geniculate nucleus Optic tract Visual cortex Contralateral Visual field lesion
31
Anatomical terms of location Hemianopsia Cortex (anatomy) Neoplasm Endocrine system Cerebral cortex
• 32
D is no t co rrect. 4 % cho se this.
33 This defect is contralat eral superior quadrantanopia due to lesions in the contralat eral Meyer loop (or inferior optic radiations) through the t emporal lobe.
The inferior optic radiations carry information from the inferior retina or the superior visual field. Inferior optic radiations t erminat e in the inferior aspect
o34 of the primary visual cortex in the occipital lobe. Therefore, lesions of the contralat eral inferior primary visual cortex could lea d to contralat eral superior
o35 quadrantanopia. However; lesions in these area s would not account for this patient's endocrine abnormalities.
Occipital lobe Temporal lobe Retina Visual cortex Contralateral Visual field Quadrantanopia Endocrine system Anatomical terms of location Cortex (anatomy)
• 36
Cerebral cortex Occipital bone
• 37
E is no t co rrect. 2% cho se this •
.•
6
lock
s
Suspend
0
End Block
Item: 33 of 41 ~. I • M k <:] t> al ~· ~
QIO: 3774 .l. ar Previous Next Lab 'Vfll ues Notes Calculator

• E is not correct. 2 % chose this •


18 This defect is contralateral inferior quadrantanopia due to lesions in the contralateral superior optic radiations that pass under the parietal lobe. The
superior optic radiations carry information from the superior retina or the inferior visual field . Superior optic radiations terminate in the superior aspect of
19
the primary visual cortex in the occipital lobe. Therefore, lesions of the contralateral superior primary visual cortex could lead to contralateral inferior
20 quadrantanopia. However; lesions in these areas would not account for this patient's endocrine abnormalities.
Parietal lobe Occipital lobe Retina Visual cortex Contralateral Visual field Endocrine system Anatomical terms of location Parietal bone Cortex (anatomy)
21
Cerebral cortex Occipital bone Quadrantanopia
22
23
Bottom Line:
24
Bitemporal hemianopia, which may accompany anterior pituitary dysfunction that causes compression of the optic chiasm, presents with visual loss in
25 the temporal portions of the visual fields bilaterally.
Bitemporal hemianopsia Optic chiasm Anterior pituitary Pituitary gland Hemianopsia Visual field Anatomical terms of location
26
27
• 28
I iii I;fi 1!1 I•J for year:l 2017 ..
29 FI RST AID FA CTS

30
FA17 p 511 .2
31
• 32 Visual field defects I. Right anopia Defect in VIsual field of
2. Bitemporal hemianopia Leye Reye
33
(pituitary lesion, chiasm)
o34
3. Left homonymous hemianopia Optic
o35 4. Left upper quadrantanopia I
netVf
- r 3 Optic uaa
• 36 (right temporal lesion, MCA) 2
Lateral
V"' - /
4
Meytr
• 37 5. Left lower quadrantanopia genocutate /
1~
.• (right parietal lesion lCA) body ttefl110ral

6
lock
s
Suspend
0
End Block
Item: 33 of 41 ~. I • M k <:] t> al ~· ~
QIO: 3774 .l. ar Previous Next Lab 'Vfll ues Notes Calculator
; - II - II 'f :.: I I :.: - I I 'J :.: 1:.: 'J I 'J - I I - I I - II I - - 'f :.; 'J I

the temporal portions of the visual fields bilaterally.


18 Bitemporal hemianopsia Optic chiasm Anterior pituitary Pituitary gland Hemianopsia Visual field Anatomical terms of location
19
20

21 I iii I;fi 1!1 I•J for year:l 2017 ..


FI RST AID FA CTS
22

23
FA17 p 511 .2
24
Visual field defects I. Right anopia Defect in vtsual field of
25
2. Bitemporal hemianopia Leye Reye
26 (pituitary lesion, chiasm)
27 3. Left homonymous hem ianopia
• 28 4. Left upper quadrantanopia Oil< it I
nerve
- r 3 Optic uaa
(right temporal lesion, MCA) d11asm /
29 _,...-
2
lateral
v - /
4
Meytr
5. Left lower quadrantanopia
30
genotutate /
1~
(right parietal lesion, lCA) body ttefl110ral
Donat optic 5 - lobo)
31 6. Left hemianopia with macular sparing ra<iatloo
(parietal
• 32 (PCA infarct) lobol
"- 3 16 ~
33 7. Central scotoma (eg, macular degeneration) I Visual PCA onfarctl
Caltanne cortex
fiSSU'e
o34
Meyer Loop-Lower retina; Loops around
o35
inferior horn of Late ral ,·entricle.
• 36 Note: When an image hits 1• visual cortex, it is upside
Dorsal optic radiation- superior retina; takes down and left-right reversed.
• 37 shortest path via internal capsule.
.•
6
lock
s
Suspend
0
End Block
Item: 33 of 41 ~ 1 • M k -<:J 1>- Jil ~· !:';-~
QIO: 3774 ..L ar Pre v ious Next Lab~lues Notes Calcula t o r
A A

FA17 p 332.3
18
19 Pituitary adenoma Benign tumor, most commonly prolactinoma (arises from lactotrophs). Adenoma · may be
20
functional (hormone producing) or nonfunctional (silent). 1onfunctional tumors present with
mass effect (bitemporal hemianopia, h} popituitarism, headache). F'unctional tumor presentation
21
is based on the hormone produced.
22 Prolactinoma in women classically presents as galactorrhea, amenorrhea, and l bone density due to
23 suppression of estrogen. Prolaclinoma in men classically presents as low libido and infertility.
24 Treatment: dopamine agonists (eg, bromocriptinc. cabcrgoline), transsphenoidal resection.
25
26

27
FA17 p496.1
• 28 Adult primary brain tumors
29 TUMOR DESCRIPTION HISTOLOGY
30 Glioblastoma Common, highly malignant 1° brain tumor with Astrocyte origin, CFAP ®. "Pseuclopalisading"
31 multiforme (grade IV - 1-year median survival. Found in cerebral rn
pleomorphic tumor cells border central
• 32
astrocytoma) hemispheres r.J. Can cross corpus callosum areas of necrosis and hemorrhage.
("butterAy glioma").
33
o34
Oligodendroglioma Rclati,·ely rare, slow growi ng. 11.osl often in Oligodendrocyte origin. "Fried egg" cells-
frontal lobes ~- "Chicken-wire" capi llarr round nuclei with clear cytoplasm [!]. Often
• 35
pattern . calcified.
• 36
Meningioma Common, typically benign 1° brain tumor. Arachnoid cell origin. Spindle cells
• 37
!lost often occurs near surfaces of brain and concentrically arranged in a whorled
.•
a
Lock
s
Suspend
8
End Bl ock
Item: 33 of 41 ~ 1 • M k -<:J 1>- Jil ~· !:';-~
QIO: 3774 ..L ar Pre v ious Next Lab~lues Notes Calcula t o r
A A

Meningioma Common, typically benign 1° brain tumor. Arachnoid cell origin. Spind le cells
18
Most often occurs near surfaces of brain and concentrically arranged in a whorled
19
in parasagittal region. Extra-axial (e\ternal pattern; psammoma bodies 0 (laminated
20 to brain parenchyma) and may have a dural calcifications).
21 attachment ("tail" ). Oflen asymptomatic;
22 may present with seizures or focal neurologic
23
signs. Resection and/or radiosurgery.
24 Hemangioblastoma ~lost oftencerebellar (!1. Associated "ith 'on Blood vessel origin. Close I~ arranged, thin-
Hippei-Lindau syndrome when found with walled capillaries with minimal intenening
25
retinal angiomas. Can produce erythropoietin parenchyma C).
26
- zopolyc} themia.
27
Pituitary adenoma Adenoma may be non functioning or Hyperplasia of only one type of endocrine cells
• 28
hyperfunctioning. .VIost commonly found in pituitary (ie, lactotroph, gonadotroph,
29 from lactotrophs (prolactinoma) 0 somatotroph, corticotroph).
30 - hyperprolactinemia; less commonly
31 adenoma of somatotrophs (G il) .... acromega ly/
gigantism; corticotrophs (ACTH) .... Cushing's
• 32
disease. Rarely, adenoma of thyrotrophs (TSH)
33
and gonadotroph (FSII , Ll l). Bitemporal
o34 hemianopia due to pressure on opt ic chiasm
• 35 (0 shows normal visual field abo,·e, patient's
• 36 perspective below). Sequelae include hyper-
• 37
or hypopituitarism, which may be caused by
.• pituitarv apople>:v.

a
Lock
s
Suspend
8
End Bl ock
Item: 33 of 41
QIO: 3774
~
..L
1 • Ma r k -<:J
Prev ious
I>
Next Lab
fJ
lues
£!1}>'

Notes
!!":-~
Cal culat o r

A A

18
Schwan noma Classically at the cerebellopont i ne angle 13, Schwann cell origin 0 , S-100 $ .
but can be along any peripheral nerve. O ften
19
localized to C N VII I in internal acoustic
20 meatus --7 \·estibular sehwannoma. Bilateral
21 \·estibular schwannomas found in NF-2.
22 Resection or stereotactic rad iosurgcry.
23
24
25
26

27
• 28

29

30
31
• 32
33
o34
• 35
• 36
• 37
.•
a
Lock
s
Suspend
8
End Bl ock
Item: 33 of 41
QIO: 3774
~
..L
1 • Ma r k -<:J
Prev ious
I>
Next Lab
fJ
lues
£!1}>'

Notes
!!":-~
Cal culat o r

A A

18
19
20
21
22
23
24
25
26
27
. 28
29
30
31
• 32
33
o34
• 35
. 36
. 37
.
· a
Lock
s
Suspend
8
End Bl ock
Item: 34 of 41 ~ 1 • M k -<:J 1>- Jil ~· !:';-~
QIO: 3543 ..L ar Pre v ious Next Lab~lues Notes Calcula t o r
A A

Below Is an artist's rendition of the anatomy of the base of the brain.


18
19
20
21
22
23
24
25
26
27
. 28
29
30
31
• 32
What Is a major function of the cranial nerve indicated by the arrows?
33
o3 4
A . Eye movement
• 35
B. Facial movement
. 36
. 37 c. Hearing
.•
a
Lock
s
Suspend
8
End Bl ock
26

27
• 28

29 What Is a major function of the cranial nerve indicated by the arrows?

30 :
A . Eye movement
31
• 32 B. Facial movement

33 c. Hearing
• 34
D. Mastication
• 35
E. Tongue movements
• 36
• 37
.•
a
Lock
s
Suspend
8
End Block
Item: 34 of 41 ~. I • M k <:] t> al ~· ~
QIO: 3543 .l. ar Previous Next lab 'Vfl1 ues Notes Calculator

18
The co rrect a nswer is D. 65% cho se this.
19 The arrows point to the trigeminal nerve (cranial nerve V) in the image. The major functions of the trigeminal nerve include mastication and facial
sensation .
20
Cranial nerves Trigeminal nerve Mastication Skull
21
A is no t co rrect. 1 0 % cho se this.
22 Eye movements are controlled by the oculomotor; trochlea r; and abducens nerves (cranial nerves III, IV, and VI, respectively), rather than the trigeminal
nerve.
23 Trigeminal nerve Cranial nerves Abducens nerve Trochlear nerve Femur Eye movement Oculomotor nerve
24
B is no t co rrect. 1 4 % cho se this.
25 Fa cial movement is a function of the facial nerve (cranial nerve VII ) rather than the trigeminal nerve.
Trigeminal nerve Facial nerve Cranial nerves
26
27 c is no t co rrect. 8 % cho se this.
Hearing is a function of the vestibulocochlear nerve (cranial nerve VIII ) rather than the trigeminal nerve .
• 28 Vestibulocochlear nerve Trigeminal nerve Cranial nerves Skull

29 E is no t co rrect. 3 % cho se this.


30 Tongue movements are controlled by the hypoglossal nerve (cranial nerve XII ) rather than the trigeminal nerve.
Hypoglossal nerve Trigeminal nerve Cranial nerves Skull
31
• 32
Botto m Li ne:
33
Cranial nerve V, the trigeminal nerve at the base of the brain, controls muscles of mastication and facial sensation .
34 Trigeminal nerve Mastication Cranial nerves Muscles of mastication Skull Brain Human brain
o35
• 36
• 37 lijj ;fi IJ l•l f o r yea r :l 2017 ..
.• FIRST AID FAC T S

6
lock
s
Suspend
0
End Block
Item: 34 of 41 ~ 1 • M k -<:J 1>- Jil ~· !:';-~
QIO: 3543 ..L ar Pre v ious Next Lab~lues Notes Calcula t o r
A A
FA17 p474.2
18
Brain stem- ventral
19 Optic chiasm Olfactory bulb (CN I)
view
20 Olfactory tract
21 CN II
Infundibulum
Optic tract
22
23 ~CNIII
24
25
T '"w"""'"""'
and immediately
decussates)

~
- - - CNV
26 CNVl
Middle cerebellar ~ ~ • CNVll
27 peduncle
. 28 Pyram1d
29 Pyramidal decussation
CN XI
30
Cl CN XII
31
• 32 4 C arc above pons (1, II, Ill, IV).
33
4 CN are in pons (V, Vi, V II, V III).
4 C are in medulla (IX, X, XI, XII).
34
4 C1 nuclei are medial (Il l, IV, V I, XI I). "Factors of 12, except I and 2."
• 35
. 36
FA17 p 476.2
. 37
Cranial nerves
.•
a
Lock
s
Suspend
8
End Bl ock
Item: 34 of 41 ~ 1 • M k -<:J 1>- Jil ~· !:';-~
QIO: 3543 ..L ar Pre v ious Next Lab~lues Notes Calcula t o r

FA17 p 476.2 •
18 Cranial nerves
19 NERVE CH FUNCTION TYPE MNEMONIC
20 Olfactory 1 Smell (only C t without I halamic relay to cortex) Sensory Some
21 Optic II Sight Sensory Say
22 Oculomotor Ill Eye 111o,·ement (SR, I R, \I R, 10), pupillar) constriction \lotor \ larr)
23 (sphincter pupillae: Edinger-Westphal nucleus, muscarinic
24 receptors), accommodation, C) clicl opening (levator palpebrae)
25 Trochlear IV Eye 111o,·cment (SO) \lotor \lone\'•
26 Trigeminal v r.. lastication, facial sen ation (ophthalmic, maxillary, mandibular Both But
27 di\'isions}, somatosensation from anterior 2/ ; of tongue
• 28 Abducens Eye mo,·ement (LR) \lotor 1\ ly
29 Facial VII Facial 1110\'Cmcnt, taste from anterior 2h of tongue, lacrimation, Both Brother
30
salivation (submandibular and sublingual glands), eyelid closing
(orbicularis oculi), auditory ,·olumc modulation (stapedius)
31
Vestibulocochlear Vlll l learing, balance Sensory Says
• 32
Glossopharyngeal IX Taste and sensation from posterior 1h of tongue, swallowing, Both Big
33
salivation (parotid gland), monitoring carotid body and sinus
34 chemo- and barorcceptors, and elevation of pharynx/larynx
• 35 (stylopharyngeus)
• 36 Vagus X Taste from supraglottic region, swallowing, soft palate elevation, Both Brains
• 37 midline uvula, talking, cough reAex, parasympathetics to
.• thoracoabdominal ,·iscera monitoring aortic arch chemo- and
a
Lock
s
Suspend
8
End Bl ock
Item: 34 of 41 ~ 1 • M k -<:J 1>- Jil ~· !:';-~
QIO: 3543 ..L ar Pre v ious Next Lab~lues Notes Calcula t o r
A A

Abducens l Eye movement (LR) ~lotor l\ly


18
19 Facial II Facial moYement, taste from anterior ~h of tongue, lacrimation, Both Brother
sa)i,·ation (submandibular and sublingual glands), eyelid closing
20
(orbicularis oculi), auditory volume modulation (stapedius)
21
Vestibulocochlear VIII Hearing, balance Sensory
22
Glossopharyngeal IX Taste and sensation from posterior 1h of tongue, swallowing, Both
23 saJi,·ation (parotid gland), monitoring carotid body and sinus
24 chemo- and baroreceptors, and elevation of pharynx/larynx
25 (stylopharyngeus)
26 Vagus X Taste from supraglottic region, swallowing. soft palate elevation, Both Brains
27 midline u\'llla, talking, cough reAex, parasympathetics to
thoracoalxlominal ,·iscera, monitoring aortic arch chemo- and
• 28
baroreccptors
29
Accessory XI Head turning, shoulder shrugging (SCM, trapezius) l\ lotor l\ latter
30
Hypoglossal Xll Tongue movement Motor l\ lost
31
• 32
FA17 p 477.3
33
34 Mastication muscles 3 muscles close jaw: vrasscter, te\l[poralis, l\ t's !\lunch.
.Medial pterygoid. l opens: Lateral pterygoid. Lateral Lowers (when speaking of pterygoids
• 35
All are innervated by trigeminal nerve ( ~). with respect to jaw motion).
• 36
"It takes more muscle to keep your mouth shut."
• 37
.•
a
Lock
s
Suspend
8
End Bl ock
Item: 35 of 41 ~ 1 • M k -<:J 1>- Jil ~· !:';-~
QIO: 320 6 ..L ar Pre v ious Next Lab~lues Notes Calcula t o r

18
A

A 33-year-old woman presents to the emergency department complaining of pain behind her right ear since that morning. Neurologic examination Is
notable for paralysis of the right side of her face, decreased taste sensation on the right side of her tongue, and increased sensitivity to loud sounds
lA• A] A

19 In her right ear. The rest of her neurologic examination is normal. The physician suspects a short course of steroids will improve her facial weakness
over the next few weeks.
20
21 In addition to the symptoms described above, t his patient may also develop:
22
:
23 A. Decreased sensation over her left upper cheek
24
B. Decreased sensation over her right upper cheek
25
c. Deviation of the uvula and soft palate to the left when the patient Is asked to say "Ahh"
26
27 D. Deviation of the uvula and soft palate to the right when the patient Is asked to say "Ahh "

. 28 E. Dryness in her left eye


29
F. Dryness in her right eye
30
31
• 32
33
34
o35
. 36
. 37
.•
a
Lock
s
Suspend
8
End Bl ock
Item: 35 of 41 ~. I • M k <:] t> al ~· ~
QIO: 3206 .l. ar Previous Next Lab 'Vfll ues Notes Calculator

18
19
20 The co rrect a nswer is F. 53 % cho se this.
This patient's history and physical exam findings are consistent with Bell pa lsy, an acute peripheral facial nerve palsy of unknown cause. The symptoms
21 seen in this patient can be understood if one remembers the different nerve fibers carried by the facial nerve (cranial nerve VII) : Afferent t aste fibers
22 from the anterior two-thirds of the ipsilat eral tongue (decrea sed t aste sensation), general touch and pain sensory fibers from a small area around the
ipsilat eral ear (retroauricular pain), and motor fibers to the muscles of facial expression (ipsilat eral facial paralysis) and stapedius muscle (increa sed
23 sensitivity to noise in the ipsilat eral ear due to weakness of the stapedius muscle, which normally prevents excessive movement of the stapes). This
patient may also experience dryness in her ipsilat eral (right) eye and mouth because the facial nerve carries parasympathetic fibers to the ipsilat eral
24 lacrimal gland and submandibular/sublingual glands, which provides lubrication to the eye and mouth, respectively. In addition, weakness of the facial
muscles prevents complet e eye closure, exacerbating the eye dryness. Patients with facial nerve paralysis should be given lubricating eye drops and
25
instructed to t ape their eye closed at night. Although facial nerve paralysis can be caused by hea d trauma, AIDS, Lyme disea se, sarcoidosis, or brain stem
26 lesions, in most cases no cause is discovered and the diagnosis of Bell palsy is made.
Lyme disease lacrimal gland Sarcoidosis Stapedius muscle Bell' s palsy Stapes Cranial nerves Facial nerve paralysis Parasympathetic nervous system Facial nerve
27
Brainstem Axon Facial muscles Motor neuron Paralysis Head injury Skull HIV/AIDS Physical examination Human brain Anatomical terms of location Brain
• 28 lacrimal bone Facial expression Ipsilateral Muscle
29 A is no t co rrect. 4 % cho se this.
30 Decrea sed sensation of the left upper cheek could be caused by lesion of the left maxillary division of the trigemina l nerve (cranial nerve V2) ; however;
this patient shows signs of f acia l nerve but no signs of trigeminal nerve involvement.
31 Trigeminal nerve Facial nerve Cranial nerves Maxillary nerve lesion Skull
• 32
B is no t co rrect. 16 % cho se this.
33 Decrea sed sensation on the right upper cheek could be caused by lesion of the right maxillary division of the trigemina l nerve (cranial nerve V2 ) ;
however; this patient shows signs of f acia l nerve palsy but no signs of trigeminal nerve involvement.
34 Trigeminal nerve Facial nerve Cranial nerves Maxillary nerve lesion Skull
35 c is no t co rrect. 14 % cho se this •
• 36 Lesion of the right vag us nerve would lea d to deviation of the uvula and soft palat e to the left ; however; all signs and symptoms in this case suggest
damage to the f acia l nerve (CN VII) and not the vagus nerve (CN X) .
• 37
Palatine uvula Vagus nerve Soft palate Facial nerve Palate lesion
.•
6
lock
s
Suspend
0
End Block
Item: 35 of 41 ~. I • M k <:] t> al ~· ~
QIO: 3206 .l. ar Previous Next lab 'Vfllues Notes Calculator

c is not correct. 14% chose this.


18
Lesion of the right vagus nerve would lea d to deviation of the uvula and soft palat e to the left; however; all signs and symptoms in this case suggest
19 damage to the facial nerve (CN VII) and not the vagus nerve (CN X) .
Palatine uvula Vagus nerve Soft palate Facial nerve Palate lesion
20
D is not correct. 9 % chose this.
21
Unilat eral lesions of the vagus nerve or b nucleus ambiguous prevent elevation of the uvula on that side and thus cause the uvula and soft palat e to
22 deviat e away from the side with the lesion due to unopposed action from the normal side. A lesion of the left vagus nerve would cause the uvula and soft
palat e to deviat e to the right; however; the patient in this case has symptoms indicating palsy of the facial nerve (cranial nerve VII), not the vagus nerve
23 (cranial nerve X) .
24 Nucleus ambiguus Facial nerve Palatine uvula Soft palate Vagus nerve Cranial nerves Palate lesion Skull

25 E is not correct. 4 % chose this.


The facial nerve (cranial nerve VII) carries sensory and motor fibers that innervat e the ipsilat eral face; thus a lower motor nerve palsy of the facial nerve
26 would cause dryness of the ipsilateral (right ), not contralat eral (left), eye.
27 Cranial nerves Facial nerve Motor nerve Motor neuron Anatomical terms of location Axon Contralateral Skull Ipsilateral Nerve

• 28

29 Bottom Line:

30 Bell palsy results from a facial nerve lesion that causes ipsilat eral dry eye, dry mouth, decrea sed t aste sensation, retroauricular pain, sensitivity to noise,
and facial paralysis.
31 Bell's palsy Facial nerve Hypogeusia Xerostomia Facial nerve paralysis Paralysis Keratoconjunctivitis sicca Anatomical terms of location lesion Ipsilateral
• 32
33
34 lijl;fiiJI•l foryear:[2017 • ]
FI RST AID FAC T S
35
• 36 FA17 p 502.2
• 37 Facial nerve lesions
.•
6
lock
s
Suspend
0
End Block
Item: 35 of 41 ~ 1 • M k -<:J 1>- Jil ~· !:';-~
QIO: 320 6 ..L ar Pre v ious Next Lab~lues Notes Calcula t o r
A A

18
19 14J;fii!11•J for y ear:
FIRST 1110 FIICTS
2017 ...

20
21 FA17 p 502.2

22 Facial nerve lesions

23 Upper motor neuron Destruction of motor corte:. or connect ion


24
lesion between motor cortex and facial nucleus in .,,_
roc•-

25
26
pons ..... contralateral paralysis of lower muscles
of facial expression. Forehead is spared due to
its bilateral U.\ IN innen·ation.
Clltlfa

~ c
....
•OICI

-
27 l ower motor neuron Destruction offacia I nucleus or C II
~
aJ.. ___-;:;::
t..qoor
• 28 lesion any\\ here along its course ..... ipsilateral lnooll•
c.ntnl .._,
""-
29 paralysis of upper and lower 111uscles of locilll
facial expression · , hyperacusis, loss of taste
_,
U4H
30
sensation to anterior tongue.
31
When idiopathic (most common), fe~cial nerve
• 32
palsy is called Bell palsy. May also be caused
33 by Lyme disease, herpes simplex, herpes
34 zoster (Ramsay Hunt syndrome), sarcoidosis,
35 tumors (eg, parotid gland), diabetes rnellitm.
• 36
Treatment is corticosteroids, ac)ciO\ ir. Most
patients gradually recover function .
• 37
.•
a
Lock
s
Suspend
8
End Bl ock
Item: 35 of 41 ~ 1 • M k -<:J 1>- Jil ~· !:';-~
QIO: 320 6 ..L ar Pre v ious Next Lab~lues Notes Calcula t o r


FA17 p476o2
18
Cranial nerves
19
NERVE CH FUNCTION TYPE MNEMONIC
20 1
Olfactory Smell (only C 1 without I halamic relay to cortex) Sensory Some
21
Optic II Sight Sensory Say
22
Oculomotor Ill Ere mo,oement (SR, IR, \I R, 10), pupillar) consI riction \lotor \ larr)
23 (sphincter pupillae: Edinger-Westphal nucleus, muscarinic
24 receptors), accommodation, C) clid opening (levator palpebrae)
25 Trochlear IV Eye mo,ocment (SO) \lotor \lone\'•
26 Trigeminal V l\ lastication, facial sen ation (ophthalmic, maxillary, mandibular Both But
27 di\'isions}, somatosensation from anterior 2/ ; of tongue
• 28 Abducens Eye mo"ement (LR) \lotor \ly
29 Facial II Facial movement, taste from ;mtcrior z;; of tongue, lacrimation, Both Brot her
30 salivation (submandibula r and sublingual glands), eyelid closing
31
(orbicularis oculi), auditory \oo lumc modulation (stapedius)
• 32
Vestibulocochlear Ill l learing, balance Sensory Says
33 Glossopharyngeal IX Taste and sensation from posterior 1/; of tongue, swallowing, Both Big
salivation (parotid gland), monitoring carotid body and sinus
34
chemo- and barorcccptors, and elevation of pharynx/larynx
35 (stylopharyngeus)
• 36
Vagus X Taste from supraglottic region, swallowing, soft palate ele\'ation, Both Brains
• 37 midline uvula, talking, cough reAex, parnsympathetics to
.• •h lvl
0

I
0

ot
0


0

I I rl

a
Lock
s
Suspend
8
End Bl ock
Item: 35 of 41 ~ 1 • M k -<:J 1>- Jil ~· !:';-~
QIO: 320 6 ..L ar Pre v ious Next Lab~lues Notes Calcula t o r
A A

Oculomotor Ill Eye mo,·ement (SR, IR, ,\ IR, 10), pupi llary constriction ~1 otor ~ l arry
18
(sphincter pupillae: Ed inger-\Veslph a I nucleus, muscarinic
19
receptors), accommodation, eyelid opening (levator palpebrae)
20
Trochlear TV Eye mo,·ement ($0) ~ lotor J\loner
21
Trigeminal v lastication, facial sensation (ophthalmic, maxillary, mandibular Both But
22 di\'isions), somatosensalion from anterior ~h of longue
23 VI Ere mo,·ement ( LR) \ lotor \ ly
Abducens
24
Facial VII Facialmo, emenl, taste from anterior ~/l of tongue, lacrimation, Both Brother
25 sali\'ation (submandibular and sublingual glands), eyelid closing
26 (orbicularis oculi), auditory \'Oinme modulation (stapedius)
27 Vestibulocochlear III Hearing, balance Sensory Says
. 28 Glossopharyngeal IX Taste and sensation from posterior 1/ l of tongue, swallowing, Both Big
29 salivation (parotid gl<md), monitoring carotid body and sinus
30
chemo- and baroreceptors, and elevation of pharynx/larynx
(stylopharyngeus)
31
Vagus X Taste from supraglottic region, swallowing, soft palate elevation, Both Brains
• 32
midline uvula, talking, cough reflex, parasympathetics to
33 thoracoabdominal viscera, monitoring aortic arch chemo- and
34 baroreceptors
35 Accessory XI Head turning, shoulder sh rugging (SC 1, I rapezius) J\1otor ~ l atter
. 36 Hypoglossal Xll Tongue movement Motor ~ l ost
. 37
.•
a
Lock
s
Suspend
8
End Bl ock
Item: 36 of 41 ~ 1 • M k -<:J 1>- Jil ~· !:';-~
QIO: 10 71 ..L ar Pre v ious Next Lab~lues Notes Calcula t o r
A A

18 A 29-year-old woman delivers a male infant at 31 weeks' gestation via cesarean section. The infant dies after 12 hours, secondary to respiratory
difficulty. The infant is noted to have microcephaly, colobomas, and a cleft palate. A karyotype reveals a trisomy of chromosome 13.
19
20 What brain abnormality would most likely be seen on autopsy?
21
:
22 A. Anencephaly

23 B. Holoprosencephaly
24
c. Lissencephaly
25
D. Porencephaly
26
27 E. Schizencephaly

. 28
29
30
31
• 32
33
34
35
. 36
. 37
.•
a
Lock
s
Suspend
8
End Bl ock
Item: 36 of 41 ~. I • M k <:] t> al ~· ~
QIO: 1071 .l. ar Previous Next lab 'Vfl1ues Notes Calculator

18
19 The co rrect a nswer is B. 69% cho se this.
Holoprosencephaly ranges in severity from complet e fusion of the cerebral hemispheres (alobar holoprosencephaly) to partial fusion (lobar
20 holoprosencephaly). Often other midline forebrain structures, including the optic tracts and bulbs, olfactory tracts and bulbs, corpus callosum, pituitary,
and hypothalamus, are also affected. These midline defects are characteristic of trisomy 13 (Pat au syndrome).
21
Holoprosencephaly Corpus callosum Hypothalamus Patau syndrome Olfactory tract Cerebral hemisphere Forebrain Trisomy Olfaction Pituitary gland
22
A is no t co rrect. 1 7% cho se this.
23 Anencephaly is the absence of the brain and calva ria. It is not characteristic of trisomy 13 (Pat au syndrome).
24 Anencephaly Patau syndrome Trisomy Human brain Brain

25 c is no t co rrect. 8 % cho se this.


Lissencephaly, also known as agyria, refers to the absence of normal cerebral convolutions and a poorly formed sylvian fissure. These infants frequently
26 present with associat ed microcephaly, failure to thrive, and significant developmental delay with seizure disorder. This is not characteristic of trisomy 13
(Pat au syndrome).
27
lissencephaly Microcephaly lateral sulcus Patau syndrome Failure to thrive Trisomy Epilepsy Specific developmental disorder
• 28
D is no t co rrect. 3 % cho se this.
29 Porencephaly refers to the presence of cavities or cysts within the brain that represent developmental abnormalities of cell migration . They are frequently
associat ed with other congenital malformations, such as encephalocele and microcephaly. Affected infants t end to have many problems, including
30
intellectual disability, optic atrophy, and seizures. This condition is not characteristic of trisomy 13 (Pat au syndrome).
31 Encephalocele Porencephaly Microcephaly Patau syndrome Intellectual disability Congenital disorder Trisomy Atrophy Epileptic seizure Optic neuropathy Brain

• 32 Cyst Cell migration Human brain

33 E is no t co rrect. 3 % cho se this.


Schizencephaly refers to the presence of clefts within the cerebral hemispheres due to abnormalities in morphogenesis. They may be unilat eral or
34 bilat eral and fused or unfused and are frequently bordered by abnormal brain. These patients have severe intellectual disability and have seizures that
are typically difficult to control. This condition is not characteristic of trisomy 13 (Pat au syndrome).
35
Schizencephaly Patau syndrome Morphogenesis Intellectual disability Trisomy Cerebral hemisphere Epileptic seizure Human brain Brain
36
• 37
.• Botto m Line:

6
lock
s
Suspend
0
End Block
Item: 36 of 41 ~. I • M k <:] t> al ~· ~
QIO: 1071 .l. ar Previous Next lab 'Vfl1ues Notes Calculator

18 Bottom Line:
19 Holoprosencephaly is a severe developmental brain defect that occurs when the prosencephalon (precursor of the cerebral hemispheres and thalami)
fails to divide in the midline to form two separate hemispheres. It is often associated with midline facial defects such as cleft lip/palate and
20 synophthalmia (a single midline orbit with or without a globe). It is most commonly seen with Patau syndrome (trisomy 13).
Holoprosencephaly Forebrain Patau syndrome Trisomy Cleft lip and palate Cerebral hemisphere Brain Thalamus Human brain Orbit
21
22

23
lijl;fiiJI•l foryear:[2017 • ]
24 FIRST AID FACTS

25
FA17 p 59.1
26
Autosomal trisomies
27
Down syndrome Findings: intellectual disability, flat facies, Incidence 1:700.
• 28
(trisomy 21) prominent epicanthal folds, single palmar Drinking age (21).
29
crease, gap between 1st Z toes, duodenal Most common viable ch romosomal disorder and
30 atresia, Hirschsprung disease, congenital heart most common cause of genetic intellectual
31 disease (eg, atrioventricular septal defect), disability.
• 32 Brushfield spots. Associated with early-onset First-trimester ultrasound commonly sho,,·s
Alzheimer disease (chromosome Zl codes for t nuchal translucency and hypoplastic nasal
33
amyloid precursor protein) and t risk of ALL bone; l serum PAPP-A, t free ~-hCG.
34
andAML. Second-trimester quad screen shows
35 95% of cases due to meiotic nondisjunction l a -fetoprotein, t ~-hCG, l estriol,
36 (t with advanced maternal age; from 1:1500 in t inhibin A.
• 37 women< ZO to 1:25 in women> 45 rears old).
.• 4% of cases clue to unbalanced Robertson ian

6
lock
s
Suspend
0
End Block
Item: 36 of 41 ~ 1 • M k -<:J 1>- Jil ~· !:';-~
QIO: 10 71 ..L ar Pre v ious Next Lab~lues Notes Calcula t o r
A A

Edwards syndrome Find ings: PRIJ\C E Edwa rd- P rominent Incidence 1:8000.
18
(trisomy 18) occiput, Rocker-bottom feet, Intellectua I Election age (18).
19
disability, ;\ondisjunction, C lenched fists 2nd most common autosomal trisomy resulting
20 (with o,·erlapping fin gers}, low-set Ears, in Ji,·e birth (most common is Down syndrome}.
21 micrognathia (small jaw), congenital heart PAPP-A and free ~-hCG are l in first trimester.
22 disease. Death usually occurs by age I. Quad screen shows l a-fetoprotein, l P-hCG,
l estriol, l or normal inhibin A.
23
24 Patau syndrome Findings: se\·ere intellectual disabilit). rocker- Incidence 1:15,000.
(trisomy 13) bottom feet, microphthalmia, microcephaly, Puberty ( 13).
25
cleft liP/Palate, holoP rosencephaly, First-trimester pregnancr screen shows l free
26
Polydact) ly, cutis aPlasia, congenital heart ~-hCG , 1 PAPP-.\.
27 disease. Death usuall) occurs by age I.
• 28 Nondisjunction in meiosis I Nondisjunction in meiosis II
29

30
Meiosis I
31
• 32
33
34
35
Meiosis II
c' c'
36
• 37
c' '
.• I Nondi.sjunction

a
Lock
s
Suspend
8
End Bl ock
Item: 36 of 41 ~ 1 • M k -<:J 1>- Jil ~· !:';-~
QIO: 10 71 ..L ar Pre v ious Next Lab~lues Notes Calcula t o r
A
.. .. . ...
A

18 Patau syndrome Findings: severe intellectual clisabilit), rocker- Incidence 1:15,000.


19 (trisomy 13) bottom feet, microphthalmia, microcephaly, Puberty (13).
cleft liP/Palate, holoProsencephaly, First-trimester pregnancy screen shows l free
20
Pol}clact}ly, cutis a Plasia, congenital hearl P-hCG, ' PAPP-A.
21
disease. Death usually occurs by age I.
22
Nondisjunction in meiosis I Nondisjunction in meiosis II
23
24
25 Meiosis I

26
27 Nondisjunction
. 28
29
30 ''
'' '' Meiosis II
'' ''
31
'' '' '
) ~''"'"i"""''"
• 32
33 A A A
34
Ill Ill I u II I Ill
'
Gametes
35
n+1 n+ 1 n-1 n- 1 n n n- 1 n+l
36
. 37 Trisomy Monosomy Noonal Monosomy Trisomy lil
.•
a
Lock
s
Suspend
8
End Bl ock
Item: 36 of 41
QIO: 10 71
~
..L
1 • Ma r k -<:J
Pre v ious
I>
Next Lab
fJlues
£!1}>'

Notes
!!":-~
Calcula t o r
A A

18 FA17 p 460.2

19 Regional specification of developing brain


20 Three primary Five secondary Adult derivatives of:
21 vesicles vesicles Walls Cavities

----=(~-/
22
23 Telencephalon -+- Cerebral lateral
Wall Cavity hemispheres ventriCles
24
25
26
I- - - Forebrain Thalamus. Third
(prosencephalon) ventocle
Hypothalamus
27
. 28
Midbrain - ----+ Mesencephalon -----+-- Midbrain Aqueduct
29 (mesencephalon)
30
Pons Upper part of
31 fourth ventricle

• 32 Hindbrain ~ Metencephalon Cerebellum


(rhombencephalon) ~
33
Myelencephalon ----___;~
34 Medulla Lower part of
fourth ventricle
35
36
Sp1nalcord
. 37
.
· a
Lock
s
Suspend
8
End Bl ock
Item: 37 of 41 ~ 1 • M k -<:J 1>- Jil ~· !:';-~
QIO: 3776 ..L ar Pre v ious Next Lab~lues Notes Calcula t o r

18
A

During a neurosurgical intervention, a region of the brain is accidentally damaged. As a result, the patient has involuntary, violent, jerky flinging,
flapping, and circular movements involving only one side of the body. Mechanistically, this limb flailing is thought to result from loss of inhibition of the
lA• A] A

19 thalamus by the globus pallidus internus.

20
In an experimental model, isolated destruction of which anatomic location would lead to this syndrome?
21
22 :
A. Bilateral lesions of the amygdalae
23
24 B. Either globus pallidus externus

25 c. The contralateral substantia nigra pars compacta

26
D. The contralateral subthalamic nucleus
27
E. The ipsilateral intermediate zone of the cerebellum
. 28
29
30
31
• 32
33
34
35
36
. 37
.•
a
Lock
s
Suspend
8
End Bl ock
Item: 37 of 41 ~ 1 • M k -<:J 1>- Jil ~· !:';-~
QIO: 3776 ..L ar Prev ious Next Lab~lues Notes Calculat o r

A A

18 Th e correct a nsw er i s D. 720/o ch ose this.


19 To understand why lesions to the subthalamic nucleus cause hemiballismus, one must understand how the basa l ganglia modulate the activity of cortical
motor neurons. In brief, the subthalamic nucleus is part of the indirect pathway of the basal ganglia whose end result (under normal conditions) Is
20 Inhibition of the tha lamus, lea ding to decreased movement. Normally, the subthalamic nucleus potentiates (excites) the globus pallidus internus (GPI),
which then sends inhibitory input to the thalamus via the action of y-amlnobutyrlc acid. When the subthalamic nucleus is knocked out, there Is less
21 excitatory input to the GPi, leading to less inhibition to the thalamus, Increased stimulation of upper motor neurons, and increased movement. lYPically,
hemiballismus Is due either to an infa rction of the contralateral subthalamic nudeus or atrophy of the caudate nucleus as is seen in Huntington's disease.
22
Huntington s d sease Hemobaliismus Basal ganglia caudate nucleus Subthalamoc no cle s Globus pal odus Medial globus pallidus Thalamus Contralatera Ganglion
23 Motor neuron Upper motor neuron Neuron Bradykinesia Indirect pathway of movement Atrophy Anatomical terms of location Excitatory postsynaptic potential
24 Cerebo ao corte Infarction

25 A i s not co rrect. 2% chose this.

26 Normally the amygdalae are thought to mediate emotions and drives such as fear, anxiety, and aggression. Bilateral lesions of the amygdalae lead to
KIOver-Bucy syndrome (reported in monkeys more t han in humans). KIOver-Bucy syndrome is characterized by hyperreactivit y in response to visual stimuli,
27 Increased oral and sexual activity, and depressed drive and emotional reactions, not hemiballismus.
Amygdala Hemiballismus
. 28
B is not co rrect. 90/o chose t h is.
29
The globus pallidus externus (GPe) normally functions to inhibit the subthalamic nucleus when we want to initiate movement. In other words, the
30 subthalamic nucleus is like a "brake pedal" and the GPe allows us to release the brake. If we knock out this release, our brake or inhibition will remain,
resulting In decreased stimulation of upper motor neurons and decreased movement (opposite of hemiballismus).
31 Hemiballismus Globus pallidus Subthalamic nucleus Neuron Motor neuron Upper motor neuron Bradykinesia
• 32 C is not co rrect. 130/o ch ose this.
33 Destruction of the substantia nigra pars compacta would result in Parkinson's disease. Loss of these dopamine-secreting neurons results in net Inhibition
of the thalamus, leading to the paucity of movement seen in Parkinson's disease.
34 Substantia nigra Parkinson's disease Thalamus Pars compacta Neuron
35 E i s n ot co rrect. 40/o ch ose this.
36 Damage to the intermediate zone of the cerebellar hemisphere results In ataxia of the limbs with poor performance on finger-to-nose and heel-to-shin
tests, and impaired rapid alternating movements. Damage to this area does not result in hemiballismus.
37 Hemiba ismus Ata 1a Cerebellum
.•
a
Lock
s
Suspend
8
End Bl ock
Item: 37 of 41 ~. I • M k <:] t> al ~· ~
QIO: 3776 .l. ar Previous Next Lab 'Vfll ues Notes Calculator

18
Bottom Line:
19
The best way to understand the basal ganglia circuitry, and any neural circuit in general, is to m ake sure you can localize clinical syndromes that result
20 from lesions.
Basal ganglia Biological neural network Ganglion Basal (phylogenetics)
21
22

23
I iii I;fi 1!1 I•J for year:l 2017 ..
24 FIRST AID FAC T S

25
26 FA17 p 4 81 .1
Common brain lesions
27
AREA Of LESION CONSEQUEN CE EXAMPLES
• 28
Frontal lobe Disinhibition and deficits in concentration,
29
orientation, judgment; may have reemergence
30 of primitive reflexes.
31 Frontal eye fields Eyes look toward lesion.
• 32 Paramedian pontine Eyes look away from side of lesion.
33 reticular formation
34 Medial longitudinal Internuclear ophthalmoplegia (impaired Iultiple sclerosis.
35 fasciculus adduction of ipsilateral eye; nyst<1gmus of
36 contralateral eye with abduction).
37 Dominant parietal Agraphia, acalculia, finger agnosia, left-right Ccrstmann srndrome.
.• cortex disorientation
6
lock
s
Suspend
0
End Block
Item: 37 of 41 ~ 1 • M k -<:J 1>- Jil ~· !:';-~
QIO: 3776 ..L ar Pre v ious Next Lab~lues Notes Calcula t o r
A A

Nondominant parietal Agnosia of the contralatera I side of the world. Hemispatial neglect syndrome.
18
cortex
19
Hippocampus Anterograde amnesia-inability to make new
20
(bilateral} memones.
21
Basal ganglia lay result in tremor at rest, chorea, atheto is. Parkinson disease, Huntington disease.
22
Subthalamic nucleus Contralateral hemiba II ismus.
23
Mammillary bodies Wernicke-Korsakoff syndrome- Confttsion, Wernicke problems come in a C \:\ 0 ' beer.
24
(bilateral) Ataxia, :\ystagmus, Ophthalmoplegia,
25 memory loss (anterograde and retrograde
26 amnesia), confabulation, persom1lit} changes.
27 Amygdala (bilateral) KIOver-Bucy syndrome-disinhibited behavior IISV-1 encephalitis.
. 28 (eg, hyperphagia, hypersexuality, hypcrorality).
29 Superior colliculus Parinaud syndrome-paralysis of conjugate Stroke, hydrocephalus, pinea loma.
30 vertical gaze (rostral interstitial nucleus also
31
involved).
• 32 Reticular activating Reduced levels of arousal and W<lkcful ness
system (midbrain) (cg, coma).
33
34
Cerebellar hemisphere Intention tremor, limb at·axia, loss of balance; Cerebellar hemispheres are laterally located-
damage to cerebellum - ipsilatera l deficits; affect lateral limbs.
35
fall toward side of lesion.
36
Cerebellar vermis Truncal ataxia, dysarthria. crmis is centrally located-affects central body.
37 Degeneration associated with chronic alcohol
.•
a
Lock
s
Suspend
8
End Bl ock
Item: 37 of 41 ~ 1 • M k -<:J 1>- Jil ~· !:';-~
QIO: 3776 ..L ar Pre v ious Next Lab~lues Notes Calcula t o r
A A
FA17 p 4 70.1
18
19
Basal ganglia Important in voluntary movements and making postura l
adjustments.
20
Recei,·es cortical input, pro,·ides negati' e feedback to cortex to
21 modulate mo,·ement.
22 Striatum = putamen (motor) + caudate (cognili, e). 0 1-Receplor = D IRect
23 Lentiform = putamen +globus pallidus. pathway.
24 Indirect= Inhibitory.

Input from SNc
25
• Stimulatory
26
Dopamine
27 • Inhibit()()'

. 28 SNc Substantia mgra pars compacta


29 GPe Globus pallidus externus
GPi Globus pallidus internus
30
Direct
• STN Subthalamic nucleus
lnd1rect
31 Motor cortex D, Dopamine D1 receptor
pathway pathway
• 32 facilitates mhibits 02 Dopamine 02receptor
movement
33 I
I

I
I
34 I
I
I
35 I

36
Thalamus
37
.• t
a
Lock
s
Suspend
8
End Bl ock
Item: 37 of 41
QIO: 3776
~
..L
1 • Ma r k -<:J
Pre v ious
I>
Next Lab
fJ
lues
£!1}>'

Notes
!!":-~
Calcula t o r
A A

••
18
19 Stimulatory
Dopamine
20 lnhibitOI)'

21 SNc Substlnba mgra pa~ compacta


22 GPe Globus pallidus extemus
GPi Globus pallidus intemus
23
Direct
• STN Subthalamic nucleus
24 Indirect
MotOf cortex pathway pathway o, Dopamine 01receptor
25 lac11rtates 1nhlbits 02 Dopamine 02 receptor
movement
26
27
. 28
29
Thalamus
30
31
• 32
33
34
35
36 Pedunculo-
ponttne
37 nucleus
.•
a
Lock
s
Suspend
8
End Bl ock
Item: 37 of 41 ~ 1 • M k -<:J 1>- Jil ~· !:';-~
QIO: 3776 ..L ar Pre v ious Next Lab~lues Notes Calcula t o r
A
.. . .. . A
fa<:il1tates Inhibits 02 Dopamine 02 receptor
18 movement
19
20
21
22 Thalamus

23
24
25
26

27
• 28
Pedunculo-
29 ponbne
nucleus
30
31
Spinal
• 32 cord
33
Excitatory pathway-cortical inputs stimulate the striatum, stimulating the release of GABA, which
34
inh ibits GABA release from the GPi, disinhibiting the thalamus via the GPi (t motion).
35 Inhibitory pathway-cortical inputs stimulate the striatum, releasing GA B that disinhibits ST l
36 via CPe inhibition, and ST l stimulates CPi to inhibit the thalamus(' motion).
37 Dopamine binds to D 1, stimulating the excitatory pathway, and to D2• inhibiting the inhibitory
.• oathwa\· - t motion
a
Lock
s
Suspend
8
End Bl ock
Item: 38 of 41 ~ 1 • M k -<:J 1>- Jil ~· !:';-~
QIO: 320 7 ..L ar Pre v ious Next Lab~lues Notes Calcula t o r
A A

18 A 40-year-old man was admitted to the neurology service for evaluation of persistent numbness over his left j aw and lower face . MRI reveals a
schwannoma, which is compressing a cranial nerve as the nerve exits the skull.
19
20
21
22
23
24
25
26
27
. 28
29
30
31
• 32
33
34
35
36
37 Image copyright © 2009 Jaiswal eta/; licensee BioMed Central Ltd.

. 38

a
Lock
s
Suspend
8
End Bl ock
18
19
20
21
22
23
24
25
26
27
. 28 The cranial nerve involved in this case exits the skull through which of the following foramina?

29 :
A. Foramen ovale
30
31 B. Foramen rotundum
• 32
c. Foramen spinosum
33
D. Jugular foramen
34
35 E. Stylomastoid foramen

36 F. Superior orbital fissure


37
. 38

a
Lock
s
Suspend
8
End Block
Item: 38 of 41 ~ 1 • M k -<:J 1>- Jil ~· !:';-~
QIO: 320 7 ..L ar Prev ious Next Lab~lues Notes Calculat o r

A A

18
19 Th e correct an sw er i s A. 40 0/o ch ose this.
20 The foramina of the trigeminal nerve divisions can be remembered with the mnemonic Standing Room Only (SRO) for the Superior orbital fissure,
foramen Rotundum, and foramen Ovale, which t ransmit cranial nerves (CNs) v 1 , v 2, and v 3, respectively. This patient has a schwannoma of the
21 mandibular division of the trigeminal nerve (CN V 3 ) as the nerve exits the skull through the foramen oval e. Compression of CN v 3 causes numbness over
the Ipsilateral Jaw and lower face.
22
-rigemina er,,e Cranial nerves Ust of foramina of the human body Foramen ovale \heart, Mandibular nerve Foramen ovale {skull) Anatomical terms of location
23 Foramen Huma1 sl ul Ipsolateral Mandible Skull Jaw

24 B i s n ot correct. 1 30/o chose thi s.


25 The maxillary division of the trigeminal nerve (CN V 2 ) exits the skull through the foramen rotundum, and compression would cause decreased sensation
over the cheek and middle face.
26 Foramen rotundum Trigeminal nerve Maxillary nerve Human skul. Ust of foramina of the human body Skull Foramen

27 C is n ot co rrect. 120/o ch ose this.


• 28 The meningeal (recurrent) branch of the mandibular nerve (CN V3) enters the skull through the foramen spinosum, along with the middle meningeal
artery. This nerve innervates the dura mater and is responsible for pain sensation.
29 Foramen spinosum Mandibular nerve Dura mater Middle meningeal artery Trigeminal nerve Human skull Mandible Skull Ust of foramina of the human body
30 Meninges Foramen

31 0 is no t co rrect . 11 Ofo chose this.

• 32 The jugular foramen transmits the glossopharyngeal (CN IX), vagus (CN X), and spinal accessory (CN XI) nerves. The glossopharyngea l nerve Is
responsible for motor innervation of the stylopharyngeus muscle, parasympathetic Innervation of the parotid gland, and sensory innervation of the
33 pharynx, middle ear, and posterior third of the tongue . It also innervates the chemoreceptors and baroreceptors of the carotid body. The vagus nerve Is
responsible for motor innervation of the pharyngeal and laryngea l muscles, parasympathetic innervation to visceral organs, and sensory innervation to the
34 pharynx and meninges. It also innervates the chemoreceptors and baroreceptors of the aortic arch . The spinal accessory nerve innervates the
sternomastoid and upper part of the trapezius muscles.
35
Parotid gland Meninges Glossopharyngeal nerve Accessory nerve Carotid body Pharynx Stylopharyngeus muscle Aortic arch Jugular foramen
36 Parasympathetic nervous system Vagus nerve Middle ear Sternocleidomastoid muscle Larynx Baroreceptor Chemoreceptor Trapezius muscle Laryngeal muscles

37 Gland Muscle Common carotid artery Foramen Organ (anatomy) carotid artery

38 E i s n o t correct. 1 90/o chose t his•



a
Lock
s
Suspend
8
End Block
Item: 38 of 41 ~. I • M k <:] t> al ~· ~
QIO: 3207 .l. ar Previous Next Lab 'Vfllues Notes Calculator

18 E is not correct. 19% chose this.


Although the facial nerve exits the skull base through the internal auditory canal, the stylomastoid foramen serves as the exit for the facial nerve and
19
stylomastoid artery. A lesion here would result in unilat eral paralysis of the muscles of facial expression and facial drooping.
20 Facial nerve Stylomastoid foramen Facial muscles Internal auditory meatus Stylomastoid artery Ear canal Skull Paralysis Human skull

21 F is not correct. 5 % chose this.


CN III (oculomotor), CN IV (trochlear ), CN V1 (ophthalmic), and CN VI (abducens) exit the skull through the superior orbital fissure, together with the
22
superior ophthalmic vein. Lesions of these nerves would lea d to ipsilat eral extraocular muscle paralysis (CNs III, IV, and VI) and numbness of the
23 ipsilat eral forehea d and upper face (CN Vi) .
Superior ophthalmic vein Superior orbital fissure Extraocular muscles Abducens nerve Anatomical terms of location Trigeminal nerve Trochlear nerve Femur
24
Paralysis Human skull Ophthalmic veins Ipsilateral Oculomotor nerve Ophthalmology Skull Muscle
25
26
Bottom Line:
27
The mandibular division of the trigeminal nerve exits the skull via the foramen ovale .
• 28 Trigeminal nerve Foramen ovate (heart) Mandibular nerve Mandible Foramen ovate (skull) Human skull Skull list of foramina of the human body Foramen

29
30
31 I ill ;fi 1!1 I•J for year:[ 2017 ..
FI RST A I D FA CT S
• 32
33 FA17 p476.1
34 Cranial nerve and vessel pathways
35
Ao.re oor Cribriform p~te
36 cranoal fossa

37 Optic canal
38

6
lock
s
Suspend
0
End Block
Item: 38 of 41 ~ 1 • M k -<:J 1>- Jil ~· !:';-~
QIO: 320 7 ..L ar Pre v ious Next Lab~lues Notes Calcula t o r
A A

18
FA17 p 4 76.1
19
Cranial nerve and vessel pathways
20
Cnbnform plate
21
22
Opttccanal
23
24
25 Supenor orbital fissure
26 (through
sphenoid bone)
27
-~---CNV1
• 28 Foramen Rotundum CNV1
Foramen Ovate CNV1
29
Foramen Sptnosom M1ddle memngeal artery
30 CNVII
Internal aud1tory meatus
CN VIII
31
P t roo (NIX
• 32 0 In fl CNX
Jugular foramen CNXI
(through
33 Jugular vein
temporal or
34 occlpttal bone) Hypoglossal canal CN XII
Brainstem
35 Foramen magnum Spinal root of CN XI
Vertebral artenes
36
Divisions of CNV exit owing to Standing Room Only
37
38 •
a
Lock
s
Suspend
8
End Bl ock
Item: 38 of 41 ~ 1 • M k -<:J 1>- Jil ~· !:';-~
QIO: 320 7 ..L ar Pre v ious Next Lab~lues Notes Calcula t o r
A A

18 FA17 p476.2

19
Cranial nerves
NERVE CN FUNCTION TYPE MNEMONIC
20
Olfactory 1 Smell (only CN without thalamic relay to cortex) Sensory Some
21
Optic II Sight Sensory Say
22
Oculomotor Ill Eye mo,·ement ( R, IR, \IR, IO), pupillar) constriction \ lotor \ larr)
23
(sphincter pupillae: Edinger-Westphal nucleus, muscarinic
24
receptors), accommodation, C)clid opening (levator palpebrae)
25
Trochlear IV Eye mo,·ement (SO) ~ Iotor ~ lone\'

26
Trigeminal v i\ lastication, facial sen sat ion (ophthalmic, maxi liar}, mandibular Both But
27 divisions), somatosensation from anterior ~/; of tongue
. 28 I \ lotor
Abducens Eve mm·ement (LR) i\ ly
29
Facial VII Facial movement, taste from anterior 2h of tongue, lacrimation, Both Brother
30 sal ivation (submandibular and sublingual glands), eyelid closing
31 (orbicularis oculi), auditory volume •nodulation (stapedius)
• 32 Vestibulocochlear VIII Hearing, balance Sensory Says
33 Glossopharyngeal IX Taste and sensation from posterior 1/3 of tongue, swallowing, Both Big
34 sal ivation (parotid gland), monitoring carotid body and sinus
35
chemo- and baroreceptors, and elevaI ion of pharynx/larynx
(stylopharyngeus)
36
Vagus X Taste from supraglottic region, swallowing, soft palate eb'3tion, Both Brains
37
midline uvula, talking, cough rcAcx, parasympathetics to
38 • thoracoabdominal 'iscem. monilorinP aortic arch chemo- and
a
Lock
s
Suspend
8
End Bl ock
Item: 39 of 41 ~ 1 • M k -<:J 1>- Jil ~· !:';-~
QIO: 2944 ..L ar Pre v ious Next Lab~lues Not es Calcula t o r

19
A

A 34-year-old man comes to the physician because of the gradual onset of Involuntary limb and facial movements, mood swings, and trouble with his
memory. He says that his father displayed similar symptoms when he was In his 40s. MRI reveals an abnormality in the basal ganglia .
lA• A] A

20
21
Which of the following changes would most likely be seen in this patient?
22
:
23 A. Accumulation of neuritic plaques in t he globus pallid us
24
B. Copper accumulation in the basal ganglia
25
26 C. Loss of pigmentation in the substantia nigra

27 D. Protein aggregates and neuronal death in the caudate nucleus


. 28
E. Scattered plaques of demyelination in the putamen
29
30
31
• 32
33
34
35
36
37
38
o39 •
a
Lock
s
Suspend
8
End Bl ock
Item: 39 of 41 ~. I • M k <:] t> al ~· ~
QIO: 2944 .l. ar Previous Next lab 'Vfl1 ues Notes Calculator

19
20 The co rrect a nswer is D. 6 1% cho se this.
21 Huntington disea se is characterized by chorea, dystonia, altered behavior; and dementia. It is an autosomal dominant disea se caused by CAG triplet
repea ts on chromosome 4p. It is the classic example of genetic anticipation, in which disea se severity increa ses and age of onset becomes earlier with
22 ea ch generation . The caudat e and putamen are mainly affected, altering the indirect pathway of the basal ganglia, which results in loss of motor
inhibition. On imaging, the lat eral ventricles may appear dilat ed because of the caudat e atrophy. On histology, Huntington patients show generalized
23 brain atrophy and loss of Purkinje cells. On the cellular level, protein aggregat es can often be seen in the cytoplasm and nucleus of affected cells.
24 Huntington's disease Basal ganglia Putamen Dystonia Histology Cytoplasm Purkinje cell Dominance (genetics) Chorea Anticipation (genetics) Dementia Protein
lateral ventricles Ventricular system Caudate nucleus Autosome Atrophy Chromosome Ganglion Protein aggregation Cell nucleus Cerebral atrophy
25
Ventricle (heart) Brain Human brain
26
A is no t co rrect. 10 % cho se this.
27
Alzheimer disea se is the most common cause of dementia in the elderly. It is marked by progressive memory loss and cognitive impairment.
• 28 Pathophysiologically, this disea se is associat ed with deposition of neuritic plaques (abnormally clea ved amyloid protein) and neurofibrillary t angles
(phosphorylat ed t au protein) in the cerebral cortex. Cholinergic neurons in the globus pallidus have been shown to be lost in Alzheimer disea se.
29 Tau protein Globus pallidus Cerebral cortex Cholinergic Alzheimer's disease Neurofibrillary tangle Amyloid Dementia Protein Senile plaques Phosphorylation

30 Neuron Cognitive deficit Amnesia

31 B is no t co rrect. 7% cho se this.


• 32 Wilson disea se is caused by failure of copper to enter circulation bound to ceruloplasmin. This disorder results in copper accumulation in the liver; cornea s,
and basal ganglia. Symptoms include asterixis, parkinsonian symptoms, cirrhosis, and Kayser -Fleischer rings (corneal deposits of copper ). Although
33 Wilson& disea se can cause chorea and dementia, it is less likely in this scenario as it is inherited in an autosomal recessive fashion and other expected
manifestations are not present.
34 Cornea Ceruloplasmin Asterixis Basal ganglia Wilson's disease Cirrhosis Chorea Autosomal recessive Dementia liver Autosome Copper Dominance (genetics)

35 Kayser-Fleischer rings Ganglion Parkinsonism Parkinson's disease

36 c is no t co rrect. 1 2% cho se this.


37 Parkinson disea se results from loss of dopaminergic neurons and therefore loss of pigmentation in the substantia nigra. These changes alter the direct
pathway of the basal ganglia, resulting in loss of excitation . Patients with Parkinson disea se present with difficulty initiating movement, cogwheel rigidity,
38 shuffling gait, and pill-rolling tremor; not chorea .
Basal ganglia Substantia nigra Parkinson's disease Chorea Tremor Dopaminergic Neuron Gait Ganglion Gait (human)
39

6
lock
s
Suspend
0
End Block
Item: 39 of 41 ~. I • M k <:] t> al ~· ~
QIO: 2944 .l. ar Previous Next lab 'Vfl1 ues Notes Calculator

19 B is no t co rrect. 7 % cho se this.


Wilson disea se is caused by failure of copper to enter circulation bound to ceruloplasmin. This disorder results in copper accumulation in the liver; cornea s,
20 and basal ganglia. Symptoms include asterixis, parkinsonian symptoms, cirrhosis, and Kayser -Fleischer rings (corneal deposits of copper) . Although
Wilson& disea se can cause chorea and dementia, it is less likely in this scenario as it is inherited in an autosomal recessive fashion and other expected
21
manifestations are not present.
22 Cornea Ceruloplasmin Asterixis Basal ganglia Wilson' s disease Cirrhosis Chorea Autosomal recessive Dementia liver Autosome Copper Dominance (genetics)
Kayser-Fleischer rings Ganglion Parkinsonism Parkinson' s disease
23
24 c is no t co rrect. 1 2 % cho se this.
Parkinson disea se results from loss of dopaminergic neurons and therefore loss of pigmentation in the substantia nigra . These changes alter the direct
25 pathway of the basal ganglia, resulting in loss of excitation . Patients with Parkinson disea se present with difficulty initiating movement, cogwheel rigidity,
shuffling gait, and pill-rolling tremor; not chorea .
26
Basal ganglia Substantia nigra Parkinson' s disease Chorea Tremor Dopaminergic Neuron Gait Ganglion Gait (human)
27
E is no t co rrect. 1 0 % cho se this •
• 28 Multiple sclerosis (MS) is characterized by scattered plaques of demyelination that can occur anywhere in the CNS. Periventricular area s and the optic
nerve are commonly affected because of their high degree of myelination . Oligodendrocytes, which are responsible for CNS myelination, are the specific
29 t argets of this autoimmune disea se. The classic patient with MS is a white woman who presents in her 20s or 30s. Patients typically present with recurring
30 multifocal lesions that are separat ed in time (intervening periods of recovery) and space and diagnosed by MRI.
Multiple sclerosis Autoimmune disease Demyelinating disease Optic nerve Oligodendrocyte Myelin Autoimmunity Central nervous system
31
• 32
Botto m Li ne:
33
Huntington disea se is an autosomal dominant disea se caused by CAG triple repea ts that results in chorea and dementia. Genetic anticipation is a key
34 characteristic.
Huntington' s disease Dominance (genetics) Chorea Anticipation (genetics) Autosome Dementia
35
36
37
38
I iii I;fi 1!1 I•J f o r yea r:l 20 1 7 ..
FI RST AID FAC TS

39

6
lock
s
Suspend
0
End Block
Item: 40 of 41 ~ 1 • M k -<:J 1>- Jil ~· !:';-~
QIO: 3259 ..L ar Pre v ious Next Lab~lues Notes Calcula t o r

20
A

A 63-year-old homeless woman is brought to the emergency department (ED) by the police because she is disoriented and confused . On questioning,
the patient frequently forgets what she has been asked. She provides seemingly plausible details of the events prior to her coming to the hospital,
IA•A] A

21
but her accounts are entirely inconsistent with the police report . On physical examination she is emaciated and has nystagmus and an unsteady gait.
22 ED records indicate that she has presented multiple times in the past for alcohol withdrawal and alcohol-related injuries.

23
The lesion accounting for the patient's signs and symptoms is located in which part of the brain?
24
25 :
A. Amygdala
26
B. Basal ganglia
27
. 28 c. Broca area

29 o. Mammillary bodies
30
E. Wernicke area
31
• 32
33
34
35
36
37
38
39
• 40 •
a
Lock
s
Suspend
8
End Bl ock
Item:40of41 ~. , . M k <:] t> al ~· ~
QIO: 3259 .l. ar Previous Next lab 'Vfllues Notes Calculator

20
The co rrect a nswer is D. 72% cho se this.
21 The patient presents with signs of Wernicke- Korsakoff syndrome, which is caused by thiamine (vitamin Btl deficiency. Malnourished chronic alcohol abusers
22 are particularly prone to this disea se. Thiamine pyrophosphat e serves as a cofactor for several enzymes that are needed in key glucose met abolic
pathways. Wernicke encephalopathy manifests first and may progress to Korsakoff psychosis if left untrea t ed. The classic triad of Wernicke encephalopathy
23 is confusion, at axia, and ophthalmoplegia (weakness of eye muscles that may result in diplopia and/or nystagmus). Wherea s Wernicke encephalopathy
may be reversible if trea t ed early, Korsakoff psychosis is an irreversible condition characterized by anterograde amnesia, confabulation, and personality
24 changes. Anterograde amnesia is the inability to crea t e new memory, and confabulation is the act of filling in gaps in one's memory with fabrications that
25 are believed to be true. The lesion in Wernicke- Korsakoff syndrome is locat ed in the mammillary bodies.
Wernicke-Korsakoff syndrome Anterograde amnesia Diplopia Thiamine pyrophosphate Nystagmus Mammillary body Thiamine Confabulation
26
Cofactor (biochemistry) Ophthalmoparesis Ataxia Wernicke's encephalopathy Metabolism Encephalopathy Malnutrition lesion Glucose Amnesia Psychosis Alcohol
27 Enzyme

• 28 A is no t co rrect. 7% cho se this.


29 Bilat eral lesions of the amygdala causes KIOver -Bucy syndrome, which is characterized by hyperorality, hypersexuality, and disinhibition. Hyperorality
means placing inappropriat e objects into one's mouth.
30 Hypersexuality Amygdala Disinhibition

31 B is no t co rrect. 7% cho se this.


• 32 Lesions in the basal ganglia are associat ed with movement disorders such as Parkinson disea se, a hypokinetic disorder; and Huntington disea se, a
hyperkinetic disorder.
33 Hyperkinetic disorder Basal ganglia Huntington's disease Hyperkinesia Parkinson's disease Hypokinesia Movement disorder Ganglion

34 c is no t co rrect. 3 % cho se this.


35 Broca area is locat ed in the inferior frontal gyrus. Patients with lesions here have motor/nonfluent/expressive aphasia, meaning that although they can
understand what others are saying, they have difficulty producing coherent speech .
36 Inferior frontal gyrus Aphasia Broca's area

37 E is no t co rrect. 11% cho se this.


38 Wernicke area is locat ed in the superior t emporal gyrus. Patients with lesions here have sensory/fluent/receptive aphasia, meaning that they can speak
fluently but cannot understand what others or they themselves are saying .
39 Superior temporal gyrus Aphasia Wernicke's area

40

6
lock
s
Suspend
0
End Block
Item:40of41 ~. , . M k <:] t> al ~· ~
QIO: 3259 .l. ar Previous Next Lab 'Vfll ues Notes Calculator

20 Bottom Line:
21 Thiamine deficiency causes Wernicke- Korsakoff syndrome, which is associat ed with m ammillary body degeneration .
Wernicke -Korsakoff syndrome Mammillary body Thiamine
22

23
24
I ill ;fi 1!1 I•J for year:[ 2017 ..
25 FIRST AID FAC T S

26

27 FA11 p 541.2

• 28 Alcoholism Physiologic tolerance and dependence on alcohol with symptoms of withdrawal when intake is
29 interrupted.
Compl ications: alcoholic cirrhosis, hepatitis, pancreatitis, peri pheral neuropathy, testicular atrophy.
30
Treatment: disulfiram (to condition the patient to abstain from alcohol use}, acamprosate,
31
naltrexone, supportive care. Support groups such as Alcoholics Anonymous are helpfu l in
• 32 sustaining abstinence and supporting patient and fami ly.
33 Wernicke-Korsakoff Caused by vitamin B1 deficiency. Triad of confusion, ophthalmoplegia, ataxia (Wernicke
34 syndrome encephalopathy). May progress to irreversible memory loss, confabulation, personality change
35 (Korsakoff syndrome). Symptoms may be precipitated by giving dextrose before administering
36
vitamin B1 to a patient with thiamine deficiency. Associated with periventricular hemorrhage/
necrosis of mammillary bodies. Treatment: IV vitamin B1.
37
38
FA11 p 540.1
39 Psychoactive drug intoxication and withdrawal
40 DRUG INTOXlCATION WITHDRAWAL

6
lock
s
Suspend
0
End Block
Item:40of41 ~. , . M k <:] t> al ~· ~
QIO: 3259 .l. ar Previous Next Lab 'Vfll ues Notes Calculator

• •
20 FA17 p 540.1
21 Psychoactive drug intoxication and withdrawal
22 DRUG ltiTOXICATION WITHDRAWAL

23 Depressants
24 onspecific: mood elevation, l anxiety, onspecific: anxiety, tremor, seizures,
. .
25 sedation, behavioral disinhibition, respiratory lllSOmnaa.
depression.
26
27
Alcohol Emotional labil ity, slurred speech, ataxia, Time from last drink:
coma, blackouts. Serum y-glutamyltransferase 3-36 hr: minor symptoms similar to other
• 28
(GGT)-sensitive indicator of alcohol usc. depressants
29 AST value is twice ALT value. 6- 48 hr: withdrawal seizures
30 12-48 hr: alcoholic hallucinosis (usually visual)
31 48-96 hr: delirium tremens (DTs) in 5% of
• 32 cases
Treatment: benzodiazepines.
33
Opioids Euphoria, respiratory and CNS depression, Sweating, dilated pupils, piloerection ("cold
34
l gag reAex, pupillary constriction (pinpoint turkey"), fever, rhinorrhea, yawning, nausea,
35 pupils), seizures (overdose). Most common stomach cramps, diarrhea {"Au-like" symptoms).
36 cause of drug overdose death. Treatment: Treatment: long-term support, methadone,
37 naloxone. buprenorphine.
38 Barbiturates Low safety margin, marked res piratory Delirium, life-threatening cardiovascular
39 depression. Treatment: symptom management collapse.
40
(eg, assist respiration, t BP). •

6
lock
s
Suspend
0
End Block
Item: 40 of 41 ~ 1 • M k -<:J 1>- Jil ~· !:';-~
QIO: 3259 ..L ar Pre v ious Next Lab~lues Notes Calcula t o r
A A

20 Benzodiazepines Greater safety margin. laxia, minor Jeep disturbance, depression, rebound anxiety,
21 respiratory depression. Treatment: Aumazcn il Seizure.
22 (benzodiazepine receptor antagonist, but
rarely used as it can precipitate seizures).
23
Stimulants
24

25
1\'onspecific: mood eJe,·ation, ps)chomotor i\onspecific: post-use "crash," including
agitation, insomnia, cardiac arrh} thmias, depression, letharg~·, t appetite, sleep
26
taclwcardja,

a nxieh·

. disturbance. ,-ivid nightmares.
27
Amphetamines Euphoria, grandiosity, pupillary dilation,
• 28
prolonged wakefulness and attention,
29 hypertension, tachycardia, anorexia, paranoia,
30 fever. se,·ere: cardiac arrest, seizures.
31 Treatment: benzodiazepines for agitation and
SeJZUTeS .
• 32
Cocaine Impaired judgment, pupillary dilation,
33
hallucinations (including tactile), paranoid
34
ideations, angina, sudden cardiac death.
35 Treatment: a -blockers, bcnzodiazcpincs.
36 P-blockers not recommended.
37 Caffeine Restlessness, t diuresis, muscle twitching. Headache, difficulty concentrating, Au-like
38 symptoms.
39 Nicotine Restlessness. Irritability, anxiety, restlessness, difficulty
40 concentrating. Treatment: nicotine patch,

a
Lock
s
Suspend
8
End Bl ock
Item: 40 of 41 ~ 1 • M k -<:J 1>- Jil ~· !:';-~
QIO: 3259 ..L ar Pre v ious Next Lab~lues Notes Calcula t o r
A A

20 Nicotine Restlessness. lrrita bility, anxiety, restlessness, difficulty


21 concentrating. Treatment: nicotine patch,
22 gum, or lozenges; bupropion/vareniclinc.
23 Phencyclidine iolence, impulsi,ity, psychomotor agitation,
24 nystagmus, tach~ cardia, h) pertension,
analgesia, psychosis, delirium, sei.wres.
25
Trauma is most common complication.
26
Treatment: benzodiazcpincs, rapid-acting
27 antipsychotic.
• 28 Lysergic acid Perceptual distortion (visual, auditory},
29 diethylamide (LSD) depersonalization, anxiel), paranoia,
30 psychosis, possible Aashbacks.
31 Marijuana Euphoria, anxiety, paranoid delusions, Irritability, anxiety, depression, insomnia,
• 32 (cannabinoid) perception of slowed time, impaired judgment, restlessness, l appetite.
social withdrawal, t appetite, dry mouth,
33
conjunctival injection, hallucinations.
34 Pharmaceutical fo rm is dronabinol: used
35 as antiemetic (chemotherapy) and appetite
36 stimulant {in AIDS).
37 MDMA (ecstasy) Hallucinogenic stimu lant: euphoria, Depression, fa tigue, change in appetite, difficulty
38 disinhibition, hyperactivil), distorted sensory concentrating, anxiety.
39
and time perception, teeth clenching. Life-
threatening effects include hypertension,
40 • ~.. ......... L . .......... .. .J: .... 1.. . . - .....J..L ...... ..-.. : .... l .. . . - -. ........ .. ... . ... : .,

a
Lock
s
Suspend
8
End Bl ock
Item: 41 of 41 ~ 1 • M k -<:J 1>- Jil ~· !:';-~
QIO: 5 0 11 ..L ar Pre v ious Next Lab~lues Notes Calcula t o r
A A

21 A 65-year-old man presents to the emergency department complaining of acute-onset left-sided face and arm weakness with forehead sparing. The
patient's wife states that his symptoms began while watching television. On physical examination the resident physician detects a gaze deviation
22
toward the right . The patient reports an allergy to penicillin and states that he takes one "water pill" daily.
23
24 What Is the most likely cause of the patient's symptoms?

25 :
26 A. Infarction of the left anterior cerebral artery

27 B. Infarction of the left middle cerebra l artery


• 28
c. I nfarction of the left posterior cerebra l artery
29
D. Infarction of the right anterior cerebral artery
30
31 E. Infarction of the right middle cerebral artery

. 32 F. Infarction of the right posterior cerebral artery


33
34
35
36
37
38
39
40
. 41 •

a
Lock
s
Suspend
8
End Bl ock
Item: 41 of 41 ~ 1 • M k -<:J 1>- Jil ~· !:';-~
QIO: 5 0 11 ..L ar Prev ious Next Lab~lues Notes Calculat o r

A A

21
22
Th e correct an swer i s E. 6 0 0/o chose this.
23 Infarcts and Ischemia-related neurologic events occur most commonly In the middle cerebral artery (MCA). The patient in the vignette most likely has an
24 Infarction of the roght MCA superior division. Deficits associated with Infarction in this location include left face and arm weakness of the upper motor
neuron type, gaze preference toward the side of the lesion (especially In the acute period, shortly after onset), and variable hemineglect. Given the
25 distribution of weakness on the left side and ga ze preference to the right, the patient likely has an infarction of t he right MCA. At this patient's age, males
do have a higher risk of experiendng a stroke; however, epidemiologic studies have shown that above the age of 8 5, females are more at risk of having a
26 stroke than their male counterparts. Regard less of gender, hypertension remains one of the most common risk factors for stroke. The patient's "water
pill " likely represents single-agent antihypertensive therapy with a thiazide diuretic.
27
l"oddle cerebral artery Diuretic Neuron Hemispatial neglect Antihypertensove dr 1g Hyo.:rtensoon Stroke Neurology Thiazide Motor neuron Lesion Infarction
• 28 Uppe motoo neuron
29 A i s not correct. 5 % chose this.
30 Left anterior cerebral artery (ACA) infarction manifests with right leg weakness and right leg cortical-type sensory loss. Similarly to right ACA Infarctions,
patients can present with grasp reflex and behavioral abnormalities, as well as transcortical aphasia.
31 Anterior cerebral artery Aphasia Anatomical terms of location Palmar grasp reflex Refle) Cerebral arteries
. 32 B is not co rrect. 100/o chose this.
33 Left MCA Infarction may manifest with one or more of the following signs and symptoms: right face and arm weakness, nonfluent ( Broca) or fluent
(Wernicke) aphasia, and homonymous hemianopia. The patient's weakness Is on the left side, therefore the infarction is necessarily in the right (not left)
34 MCA.
35 Aphasia Homonymous hemianopsia Hemianopsia Infarction

36 C is no t co rrect . 60/o ch ose this.


Left posterior cerebral artery (PCA) infarction m anifests most commonly with right homonymous hemianopia. La rger PCA infarctions may cause alexia
37 without agraphia, aphasia, right hemisensory loss, and right hemiparesis.
38 Posterior cerebral artery Hemiparesis Homonymous hemianopsia Aphasia Agraphia Dyslexia Pure alexia Hemianopsia Infarction Cerebral arteries

39 D i s n ot correct. 120/o ch ose this.


Infarction of the right ACA manifests with left leg wea kness and left leg cortical-type sensory loss. In addition, patient s m ay present with frontal release
40 signs, such as grasp reflex, frontal lobe behavioral abnormalities, and left hemlneglect. None of these signs or symptoms is present in this patient.
41 Hemispatia neglect Fronta obe Frontal release sign Infarction
~

a
Lock
s
Suspend
8
End Block
Item: 41 of 41 ~. I • M k <:] t> al ~· ~
QIO: 5011 .l. ar Previous Next lab 'Vfl1 ues Notes Calculator

D is not correct. 12% chose this.


21
Infarction of the right ACA manifests with left leg weakness and left leg cortical-type sensory loss. In addition, patients may present with frontal release
22 signs, such as grasp reflex, frontal lobe behavioral abnormalities, and left hemineglect. None of these signs or symptoms is present in this patient.
Hemispatial neglect Frontal lobe Frontal release sign Infarction
23
F is not correct. 7 % chose this.
24
Right PCA infarction manifests with left homonymous hemianopia. If the infarct is rather large, left hemisensory loss and left hemiparesis may be present.
25 Hemiparesis Homonymous hemianopsia Infarction Hemianopsia

26
27 Bottom Line:
• 28 Infarction in the MCA commonly causes contralateral face and arm weakness of the upper motor neuron type, with ga ze preference toward the side of
the lesion and variable unilateral neglect.
29
Neuron Upper motor neuron Hemispatial neglect Contralateral Motor neuron lesion Anatomical terms of location Infarction
30
31
• 32 I ill ;fi 1!1 I•J for year:[ 2017 ..
FI RST AID FAC T S
33
34
FA17 p472.2
35
Cerebral arteries-cortical distribution
36
37 Anterior cerebral artery (supplies anteromedial surface)

38 Middle cerebral artery (supplies lateral surface)

39 • Posterior cerebral artery (supplies posterior and inferior surfaces)

40
41

6
lock
s
Suspend
0
End Block
Item: 41 of 41 ~ 1 • M k -<:J 1>- Jil ~· !:';-~
QIO: 5 0 11 ..L ar Pre v ious Next Lab~lues Notes Calcula t o r
A A

21
FA17 p 4 72.2
22
Cerebral arteries- cortical distribution
23
24 Anterior cerebral artery {supplies anteromedial surface)

25 Middle cerebral artery {supplies lateral surface)

26 • PosteriOr cerebral artery {suppl~es posteriOr and lnlerlor surfaces) I


27
• 28

29

30
31
. 32
33
34
35
36 Watershed zones Between anterior cerebral/m idd le cerebral, posterior cerebral/middle cerebral arteries. Damage by
37 severe hypotension .... upper leg/upper arm weakness, defects in higher-order visual processing.
38
39 FA17 p 484.1
40 Effects of strokes
41 ARTERY AREA OF LESION SYMPTOMS NOTES

a
Lock
s
Suspend
8
End Bl ock
Item: 41 of 41 ~. I • M k <:] t> al ~· ~
QIO: 5011 .l. ar Previous Next lab 'Vfl1 ues Notes Calculator

• •
21 FA17 p484.1

22 Effects of strokes
ARTERY AREA OF LESION SYMPTOMS NOTES
23
Anterior circulation
24
25
Middle Motor and sensory cortices fJ- upper Contralateral paralysis and sensory Wernicke aphasia is associated
cerebral limb and face. loss-face and upper limb. with right superior quadrant
26 artery Temporal lobe (Wernicke area); Aphasia if in dominant (usually visua 1 field defect due to
27 frontal lobe (Broca area). left) hemisphere. I lemineglect temporal lobe involvement.
• 28 if lesion affects nondominant
29
(usually right) side.
30 Anterior Motor and sensory cortices-lower Contralateral paralysis and sensory
cerebral limb. loss- lower limb.
31
artery
• 32
Lenticulo- Striatum, internal capsule. Contralateral paralysis and/or Common location of lacunar
33 striate sensory loss-face and body. infarctsrn, due to hyaline
34 artery Absence of cortical signs arteriosclerosis zoto
35 (eg, neglect, aphasia, visual field unmanaged hypertension.
loss).
36
37 Posterior circulation
38 Anterior Lateral corticospinal tract. Contralateral paralysis-upper and Medial medullary syndrome-
spinal lower limbs. caused by infarct of
39
artery Medial lemniscus. l contralateral proprioception. paramedian branches of ASA
40 Caudal medulla-hypoglossal nerve. Ipsilateral hypoglossal dysfunction and/or vertebral arteries.
41 • (tongue deviates ipsilaterally). •

6
lock
s
Suspend
0
End Block
Item: 41 of 41 ~ 1 • Ma rk -<:J 1>- Jil ~· !:';-~
QIO: 5 0 11 ..L Pre v ious Next Lab~lues Notes Calcula t o r
A A

21 Posterior Lateral medulla: Lateral medullary (Wallenberg)


22 inferior 'ucleus ambiguus (C l IX, X, XI ) Dy~ ph agi a , hoarseness, l gag syndrome.
cerebellar Vestibular nuclei rcflc\ Nucleus ambiguus effects arc
23
artery Lateral spinothalamic tract, spinal Vomiting, vert igo, nystagmus specific to PICA lesions
24 "Don't pick a (PICA) horse
trigeminal nucleus l pain and temperature sensation
25 from contralateral body, (hoarseness) that can' t eat
26 ipsilateral face (dysphagia)."
ympathetic fibers Ipsilateral Horner S) ndrome Also supplies inferior cerebellar
27
Inferior cerebellar peduncle Ataxia, d)Smetria peduncle (part of cerebellum).
• 28
29 Anterior Lateral pons Lateral pontine syndrome.
inferior Facial nucleus Pa ralr~i\ of face, l lacrimation, Facial nucleus effects are
30
cerebellar l salivation, l taste from anterior specific to AICA lesions.
31 artery ~ of tongue "Facial droop means AICA's
. 32 Vestibular nuclei Vomiting, vertigo, nystagmus pooped."
33 Spinothalamic tract, spinal l pain and temperature sensation Also supplies middle and
trigeminal nucleus from conira lateral body, inferior cerebellar peduncles
34
ipsilatera I face (part of cerebellum).
35
Sympathetic fibers Ipsilatera I llorncr synd rome
36 Middle and inferior cerebellar Ataxia, dysmetria
37 peduncles
38 Basilar artery Pons, medulla, lower midbrain RAS spared, therefore preserved "Locked-in syndrome."
39 COilSCIOIISilCSS

40 Corticospinal and corticobulbar Quadriplegia; loss of' olnntary


trac t~ facial, mouth, and tongue
41 •

a
Lock
s
Suspend
8
End Bl ock
Item: 41 of 41 ~ 1 • M k -<:J 1>- Jil ~· !:';-~
QIO: 5 0 11 ..L ar Pre v ious Next Lab~lues Notes Calcula t o r
A
. - - - -
A

21 peduncles
22 Basilar artery Pons, medulla, lower midbrain RAS spared, therefore preserved "Locked-in syndrome."
23 eOJlSCIOIISileSS

24 Corticospinal and corticobulbar Quadriplegia; loss of' oluntary


tracts facial, mouth, and longue
25
mo,·ements
26
O cular cranial nerve nuclei, Loss of hori7ontal, but not vertical,
27 paramedian pontine reticular e\e mo\ ements

• 28 formation
29 Posterior O ecipitallobe [!]. Cont ralateral hemianopia with
30 cerebral macular sparing.
artery
31
. 32
33
34
35
36
37
38
39
40
41 •

a
Lock
s
Suspend
8
End Bl ock
Item: 32 of 41 ~ 1 • M k -<:J 1>- Jil ~· !:';-~
QIO: 2936 ..L ar Pre v ious Next Lab~lues Notes Calcula t o r
A A

21 A 32-year-old woman comes to the physician because of a 1-month history of lower extremity weakness and difficulty controlling hand movements.
She had similar symptoms 1 year ago that were more transient. MRI of the brain reveals plaques in the white matter of the cerebral hemispheres In
22
the area of descending motor and ascending sensory neurons.
23
24 Which of the following pathological findings is this patient most likely to have?
25 :
26 A. Album1nocytologic dissodation

27 B. Gliosis in the caudate nucleus


• 28
c. Hyperphosphorylated tau protein
29
D. Increased immunoglobulin in cerebrospinal fluid
30
31 E. Intracytoplasmic eosinophilic inclusions

. 32
33
34
35
36
37
38
39
40
41 •

a
Lock
s
Suspend
8
End Bl ock
Item: 32 of 41 ~ 1 • M k -<:J 1>- Jil ~· !:';-~
QIO: 2936 ..L ar Previous Next Lab~lues Notes Calculat or
A A

21
22 Th e correct a n swer i s D. 620/o chose this.

23 The clinical scenario, physical findings, and imaging point to multiple sclerosis (MS). MS presents with a host of neurologic problems because
demyelinating plaques can occur anywhere in the white matter. MS lesions are "separated in time and space." The patient's lesion from 1 year ago healed
24 because the myelin was able to regenerate. Depletion of myelin-producing ollgodendrocytes, which are present only in the white matter, is the hallmark
for this disease. Increased cerebrospina l fluid immunoglobulin levels, reflecting the presence of intrathecal humoral immune activation, appear In
25 80%-90% of patients with MS. IgG is the primarily increased globulin.
26 Multiple s erosis Cerebrosponal fluid Intrathecal administration Mye 1n White matte& Ohgodeudrocyte Immunoglobulin G Antibody Lesion Neurology
H omoral 'mmu 'ty Demyel"nating disease
27
A i s not correct. 7 % chose this .
• 28
Gulllaln-Barre syndrome involves the acute demyelination of peripheral nerves, sometimes preceded by a viral infection or gastroenteritis from
29 Campylobacter jejuni infection. The classic presentation is an acute ascending paralysis. Its hallmark is albuminocytologic dissociation (greatly Increased
protein concentration with only a modest increase in cell count) of the cerebrospinal fluid. Guillain-Barre syndrome does not involve the central nervous
30 system.
31 Campylobacter jejuni Cerebrospinal fluid Demyelinating disease Central nervous system Gastroenteritis Campylobacter Paralysis Protein Viral disease
Peripheral nervous system Nervous system Infection
32
B is not co rrect. 130/o chose this.
33
Poorly controlled hand movement could indica t e a lesion in the basal ganglia, as In Huntington disease, an autosomal dominant disease marked by chorea
34 and cognitive decline. However, the rest of the patient's symptoms do not point to this diagnosis. The pathogenesis of Huntington disease Involves
glutamate toxicity of the basal ganglia, nam ely the caudat e and putamen. Gliosis and neuronal depletion in these structures result in loss of motor
35 Inhibition, leading to chorea and athetoid m ovem ents. Unlike multiple sclerosis, Huntington disease does not t ypically present with clinical symptoms of
demyelinating disease.
36
Multiple sclerosis Gliosis Huntington's disease Basal ganglia Putamen Excitotoxicity Demyelinating disease Dominance (genetics) Chorea Glutamic acid Athetosis
37 Lesion Ganglion Caudate nucleus Autosome Pathogenesis Dementia
38 C i s n ot co rrect. 100/o ch ose this.
39 Neurofibrillary tangles are characteristic findings in Alzheimer disease. They are a product of hyperphosphorylat ed t au protein. Alzheimer disease Is the
most common cause of dementia in the elderly. This patient shows no signs of dementia, such as loss of short-t erm or long-term memory, Inability to
40 read, speech difficulties, or other signs of progressive decline in intellectual functioning.
41 Tau protein Neurofibr lary tangle Alzheimer's disease Protein Dementia Long-term memory Hyperphosphorylation Phosphorylation Tau

a
Lock
s
Suspend
8
End Bl ock
Item: 32 of 41 ~. I • M k <:] t> al ~· ~
QIO: 2936 .l. ar Previous Next lab 'Vfl1 ues Notes Calculator

21 E is not correct. 8% chose this.


Intracytoplasmic eosinophilic inclusion bodies describe Lewy bodies seen in Parkinson disease. Parkinson disease usually affects the elderly and is
22 characterized histologically as a loss of pigmentation in the substantia nigra . It is associated with four cardinal signs: resting tremor; bradykinesia,
cogwheel rigidity, and postural instability.
23
Substantia nigra Bradykinesia Parkinson' s disease lewy body Tremor Histology Balance disorder Inclusion bodies
24
25
Bottom Line:
26
Multiple sclerosis is a demyelinating disease of the central nervous system characterized by increased IgG levels in cerebrospinal fluid .
27 Multiple sclerosis Cerebrospinal fluid Central nervous system Demyelinating disease Immunoglobulin G Nervous system Myelin

• 28

29
30 l i j l ; f i i J I • l toryear:[ 2017 • ]
FI RST AI D FA CTS
31
32 FA17 p493.1
33
Multiple sclerosis Autoimmune inflammation and demyelination of CNS (brain and spinal cord). Patients can
34 present with optic neuritis (sudden loss of vision resulting in Ylarcus Gunn pupils), INO,
35 hemiparesis, hemisensory symptoms, bladder/bowel dysfunction. Symptoms may exacerbate with
36 increased body temperature (eg, hot bath, exercise). Relapsing and rem itting is most common
clinical course. Most often affects women in their 20s and 30s; more common in Caucasians
37
livi ng farther from equator. leek Aexion may precipitate sensation of electric shock run ning
38
down spine (Lhermitte phenomenon).
39 Charcot triad ofMS is a SIN:
40 • Scanning speech
41 • Intention tremor (also Inconti nence and Internuclear ophthalmoplegia)

6
lock
s
Suspend
0
End Block
Item: 32 of 41 ~ 1 • M k -<:J 1>- Jil ~· !:';-~
QIO: 2936 ..L ar Pre v ious Next Lab~lues Notes Calcula t o r
A A

21 FA17 p 4 93.1
22 Multiple sclerosis Autoimmune inAammation and dem)elination ofCNS (brain and spinal cord). Patients can
23 present with optic neuritis (sudden loss of vision resulting in Ylarcus Gunn pupils), !NO,
24 hemiparesis, hemisensory symptoms, bladder/bowel drsfunction. Symptoms may exacerbate with
25
increased body temperature (eg, hot bath, exercise). Relapsing and remitting is most common
clinical course. ~ lost often affects women in their 20s and 30s; more common in Caucasians
26
)i,·ing farther from equator. .\eck flexion may precipitate sensation of electric shock running
27 down spine (Lhermitte phenomenon).
• 28 Charcot triad of~ IS is a SI:\:
29 • Scanning speech
30
Intention tremor (also Incontinence and Internuclear ophthalmoplegia)
~ystagmus
31
FINDINGS t IgG level and myelin basic protein in C F'. Oligoclonal bands are diagnostic. MRI is gold
32
standard. Periventricular plaques rJ (areas of ol igodendrocyte loss and reactive gliosis) with
33 preservation of axons . .Multiple white matter lesions disseminated in space and time.
34
35
36
37
38
39 TREATMENT Slow progression with disease-modifying therapies (eg, ~ -interferon, glatiramer, natalizumab). Treat
40 acute flares with I steroids. S) mptomatic treatment for neurogenic bladder (catheterization,
41
muscarinic antagonists), spasticit) (baclofen, CASAR receptor agonists), pain (TCAs,
'
a
Lock
s
Suspend
8
End Bl ock
Item: 32 of 41 ~ 1 • M k -<:J 1>- Jil ~· !:';-~
QIO: 2936 ..L ar Pre v ious Next Lab~lues Notes Calcula t o r

21
A
FA17p500.1 A

Spinal cord lesions


22
AREAAFFECTED DISEASE CHARACTERISTICS
23
Poliomyelitis and Werdnig-Hoffmann Congenital degeneration of anterior horns of spinal
24
disease cord. LM ' lesions only. "Floppy baby" with marked

•• hypotonia and tongue fasciculations. Infantile type


25
26 has median age of death of 7 months. Autosomal
recessive inheritance.
27
Poliomyelitis - asymmetric weakness.
• 28
\Verclnig-1-Joffm ann disease - symmetric weakness.
29
Amyotrophic lateral sclerosis Combined U\II\ and L~ IN deficits with no sensor'•

•• • •
30 or bowel/bladder deficits (due to loss of cortica l and
31 spina I cord motor neurons, respectively).
32
Can be caused by defect in superoxidc dismulasc I.
Commonly presents with asymmetric limb weakness
33 (hands/feet), fasciculations, eventual atrophy. Fatal.
34 Commonly known as Lou G ehrig disease.
35 Treat mcnt: riluzole.
36 Complete occlusion of anterior Spares dorsal columns and Lissauer tract; mid-
37 spinal artery thoracic ASA territory is watershed area, as artery
of Adamkiewicz supplies ASA below - T8. Presents
38
with UM deficit below the lesion (corticospinal
39 tract), L 1 defi cit at the b ·el of the lesion (anterior
40 horn), and loss of pain and temperature sensation
below the lesion (spinothalamic tract).
41

a
Lock
s
Suspend
8
End Bl ock
Item: 32 of 41 ~ 1 • M k -<:J 1>- Jil ~· !:';-~
QIO: 2936 ..L ar Pre v ious Next Lab~lues Notes Calcula t o r
A A

21 Tabes dorsalis Caused by 3° syph ilis. Results from degeneration


22 (demyelination) of dorsal columns and roots
23
__. progressi\·e sensory ataxia (impaired
proprioception -... poor coordination).
24
Associated" ith Charcot joints, shooting pain, Arg) II
25 Robertson pupils.
26 Exam will demonstrate absence of DTRs and
27
Ef) Romberg sign.

• 28 Syringomyelia yrinx expands and damages anterior white


commissure of spinothalamic tract (2nd-order
29
neurons) - bilateral loss of pain and temperature
30
31
• sensation in cape-like distribution; seen \\ ith Chiari
!mal formation; can expand and affect other tracts.
32
33 , Vitamin 812 deficiency Subacute combined degeneration (SCO)-
dcmyclination of Spinocerebellar tracts, lateral
34
35
36
' Cauda equina syndrome
C orticospinal tracts, and Dorsal columns. Ataxic
gait, paresthesia, impaired position/vibration sense.

Unilatera l symptoms including radicular pain, absent


37 knee and ankle reA ex, loss of bladder and :m:~l
38 sphincter control. Can cause saddle anesthesia.
Treatment: emergent surgery and steroids.
39
Due to compression of spinal roots from L2 and
40 below, often caused by intra\·ertebral disk herniation
41 or tumors.

a
Lock
s
Suspend
8
End Bl ock
Item: 32 of 41 ~ 1 • M k -<:J 1>- Jil ~· !:';-~
QIO: 2936 ..L ar Pre v ious Next Lab~lues Notes Calcula t o r
A A

21 Treatment: emergent surgery and steroids.


22
Due to compression of spinal roots from L2 and
below, often caused by intnwertebral disk herniation
23
or tumors.
24
25 FA17 p 96.2
26 HLA subtypes associated with diseases
27 A3 Hemochromatosis
• 28 88 Addison disease, myasthenia grm is, CnHes
29 disease
30 827 Psoriatic arthritis, Ankylosing spondylitis, PAIR. Also known as seronegati,·e arthropathies.
31 IBD-associated arthritis, Reacti,·e arthritis
32 DQ2/ DQ8 Celiac disease I ale (8) too (2) much gluten at Dairy Queen.
33 DR2 l\Iultiple sclerosis, hay fever, SLE, l\lultiple hay pastures have dirt.
34 Goodpasture syndrome
35 DR3 Diabetes mellitus type I, SLE, Craves disease, 2-3, S-L-E
36 Hashimoto thyroiditis, Addison disease
37 DR4 Rheumatoid arthritis, diabetes mellitus type I, There are 4 walls in a "rheum" (room).
38
Addison disease

39 DRS Pernicious anemia - 'itamin B12 deficiency,


Hashimoto thyroiditis
40
41 •

a
Lock
s
Suspend
8
End Bl ock

You might also like